Population Health Exam 2

¡Supera tus tareas y exámenes ahora con Quizwiz!

6. A student is pursuing a graduate nursing education with preparation as a generalist, functioning at the micro-systems level, and assuming accountability for health care outcomes for a specific group of clients. Which educational preparation is this student receiving? a. Clinical nurse leader (CNL) b. Advanced public health nurse (APHN) c. Nurse practitioner (NP) d. Doctor of Nursing Practice (DNP)

ANS: A A clinical nurse leader is defined as a nurse who is a master's prepared generalist who functions at the micro-system level and assumes accountability for health care outcomes for a specific group of clients within a unit or area. An NP applies nursing knowledge with physical, psychosocial, and environmental assessment skills. An APHN has a minimum of a master's degree and has specialized courses related to public health. A DNP is the preferred educational preparation for specialty advanced practice nurses.

1. Which statement about disasters is true? a. They can be natural or human-made. b. They can be relieved without assistance. c. There is always injury and death when a disaster occurs. d. The timing of a disaster does not influence the types of injuries that will occur.

ANS: A A disaster is any natural or human-made incident that causes disruption, destruction, and/or devastation requiring external assistance. Although natural incidents like earthquakes of hurricanes trigger many disasters, predictable and preventable human-made factors can further affect the disaster. In the disaster response phase, the incident type and time predict subsequent injuries and illnesses.

16. A nurse considers an audience's ability to read, comprehend, and act on medical instructions while preparing health education materials. Which of the following factors is the nurse considering? a. Health literacy b. Resilience c. Social justice d. Health disparity

ANS: A A measure of a patient's ability to read, comprehend, and act on medical instructions is health literacy. Resilience has to do with the factors that allow people to resist the effects of vulnerability. Social justice describes justice with respect to the concepts of egalitarianism and equality. Health disparity refers to the wide variations in health services and health status among certain population groups.

16. It is projected that the incidence of diabetes will increase in the future and therefore creates a need for more diabetic health educators. Which of the following approaches was used to identify this need? a. Pre-active b. Reactive c. Inactive d. Interactive

ANS: A A pre-active approach projects a future need. Identifying the need for more diabetes health educators based on the projected incidence of diabetes is an example of the pre-active approach. The inactive approach defines the problem based on the existing health state of the population to be served. A reactive approach defines the problem based on past needs. An interactive approach describes the problem using past and present data to project future population needs.

25. A nurse is critiquing the program's objective to decrease the incidence of early childhood disease in Center County by providing immunization clinics in all schools between August and December of 2018." Which of the following critiques is accurate? a. It is accurate as written. b. The results are ambiguous. c. The action verb is not action oriented. d. The purpose is not stated.

ANS: A A program objective needs a strong, action-oriented verb, a statement of a single purpose, a statement of a single result, and a time frame for achieving the expected result. Thus, the objective is correct as written as it contains all these components. The results are clear—to decrease the incidence of early childhood disease in Center County. The action verb is action oriented—by providing immunization clinics in all schools. The purpose is stated—to decrease the incidence of early childhood disease in Cedar County.

10. What information is shared among agencies when they collaborate to implement a quality surveillance system? a. How to use algorithms to identify which events should be investigated? b. Who is to blame for a disease outbreak? c. What shelters will be used and by whom? d. How political action will be necessary to ensure public health?

ANS: A Collaboration promotes the development of plans and a directory of emergency responses. How to use algorithms is a key type of information that is shared. Blaming others is not part of collaboration. Determining what shelters will be used and by whom is not the priority for collaboration. How political action will be necessary to ensure public health is not a priority for collaboration.

22. A nurse is creating a program objective. Which of the following would be developed? a. Decrease the incidence of early childhood disease in Center County by providing immunization clinics in all schools between August and December of 2007. b. Provide condoms to sexually active teenagers in the local high school. c. Increase by 25% the number of school-age children wearing seat belts. d. Provide free diabetic screening every other month for 2 years.

ANS: A A program objective needs a strong, action-oriented verb, a statement of a single purpose, a statement of a single result, and a time frame for achieving the expected result—To decrease the incidence of early childhood disease in Center County by providing immunization clinics in all schools between August and December of 2007. To provide condoms to sexually active teenagers in the local high school does not include the statement of a result of this action or a time frame. To increase by 25% the number of school-age children wearing seat belts does not have a time frame for achieving the expected result. To provide free diabetic screening every other month for 2 years does not include the statement of a single result of this action or a specific time frame.

4. A nurse responding to a human-made disaster is responding to what likely source of the event? a. Structural collapse b. Communicable disease epidemics c. Mud slides d. Floods

ANS: A A structural collapse would be considered a human-made disaster. Communicable disease epidemics, mud slides, and floods are natural disasters.

16. A nurse is completing a disease investigation. Based on the clinical symptoms presented, it is assumed that the individual has measles. However, there has been no laboratory evidence documenting that this individual has the disease. Which of the following types of cases is the nurse investigating? a. A suspected case b. A confirmed case c. A prolonged case d. An identified case

ANS: A A suspected case is a clinically compatible case of illness without laboratory confirmation. A confirmed case is a clinically compatible case that is laboratory confirmed by isolation of the organism. A prolonged case and identified case are not types of cases discussed in the text.

1. Which statement about advanced public health nurses (APHNs) and nurse practitioners (NPs) is correct? a. They often work in similar settings. b. They have identical educational preparation. c. Their practices have become more similar during the past decade. d. The clinical nurse specialist role always involves prescriptive practice.

ANS: A APHNs and NPs often work in similar settings. APHNs and NPs have differing educational preparation depending on the specialty focus. The practices have continued to differ during the past decade. The clinical nurse specialist role typically does not involve prescriptive practice.

5. A nurse is providing education to a mother about the importance of having her infant immunized for measles, mumps, and rubella. Which of the following best describes the type of immunity that will be provided? a. Active b. Passive c. Natural d. Acquired

ANS: A Active immunity refers to the immunization of an individual by administration of an antigen (infectious agent or vaccine) and is usually characterized by the presence of an antibody produced by the individual host. Passive immunity refers to immunization through the transfer of a specific antibody from an immunized individual to a non-immunized individual. Natural immunity refers to species-determined, innate resistance to an infectious agent. Acquired immunity is the resistance acquired by a host as a result of previous natural exposure to an infectious agent.

8. A nurse is teaching a postpartum mother how to breastfeed her infant. The nurse notes that the mother is alert and agrees that breastfeeding is important to her and beneficial to her baby. The nurse outlines the expectations of breastfeeding for the mother and the baby. Considering the events of instruction, which of the following should the nurse do next? a. Ask the mother about her previous experience with breastfeeding. b. Demonstrate how to position the baby for breastfeeding. c. Show the mother a video about breastfeeding. d. Have the mother demonstrate breastfeeding.

ANS: A Asking the mother about her previous experience with breastfeeding identifies her educational needs. Using the TEACH mnemonic, the first thing the nurse should do is "Tune in": listen before starting to teach; the client's needs should direct the content. Thus, this should be done before the nurse does any demonstration, shows audiovisual resources, or requests return demonstrations.

10. A nurse is planning a health program. Which of the following is the most critical action that should be taken by the nursNe? R I G B.C M a. Assessing need of the community b. Considering alternative solutions or options c. Detailing the costs and resources needed d. Weighing each alternative

ANS: A Assessing need provides the reasons and rationale for the program plan. The need within the community must be assessed before alternatives can be proposed or weighed, or the details of the costs and resources can be determined.

7. A nurse is working in a temporary shelter for victims following a natural disaster. Which condition is the nurse most likely to personally encounter? a. Stress b. Communicable disease c. Depression d. Injuries requiring first aid

ANS: A Causes of stress can be the shock of the disaster itself, loss of personal possessions, fear of the unknown, living near total strangers, and boredom. Illnesses requiring first aid and communicable diseases are not the primary concern of the nurse working in the shelter. Immediately following the disaster, it is unlikely that the victims will immediately encounter depression because of the shock of the disaster.

4. A nurse is pursuing certification as an occupational health nurse. Which organization will provide this certification? a. American Board for Occupational Health Nurses (ABOHN) b. Occupational Safety and Health Education and Research Center c. National Institute for Occupational Safety and Health (NIOSH) d. Occupational Safety and Health Administration (OSHA)

ANS: A Certification in occupational health nursing is provided by the American Board for Occupational Health Nurses. The Occupational Safety and Health Education and Research Center supports master's and doctoral education with emphases in occupational health nursing, industrial hygiene, occupational medicine, and safety. NIOSH examines potential hazards of new work technologies and practices. OSHA educates employers about occupational health and safety.

10. . A nurse is caring for a migrant farmworker who has been working in the agricultural industry for the past 10 years. When questioned about environmental hazards, the client reports regular exposure to pesticides. Which of the following is the client at increased risk of developing? a. Cancer b. Memory loss c. Skin rashes d. Headaches

ANS: A Chronic exposure to pesticides may lead to cancer. Acute exposure to pesticides may cause memory loss, skin rashes, and headaches.

16. A public health worker displayed a competency in disaster medicine and public health. Which of the following describes what the worker has successfully accomplished? a. Communicating effectively with others b. Participating in the chain of command c. Using a set of standardized activities for every disaster d. Implementing a community assessment

ANS: A Communicating effectively with others in a disaster or public health emergency is one of the competencies demonstrated in disaster medicine and public health. Involvement in the chain of command, using a set of preplanned activities for every disaster, and implementing a community assessment are not part of these competencies.

11. A nurse wants to have a clear understanding of the overall health status of a community and to identify populations at risk. Which of the following actions should be taken by the nurse? a. Complete a community assessment. b. Perform a population needs assessment. c. Evaluate past programs in the community. d. Develop program resources for the community.

ANS: A Community assessment is a population-focused approach that views the entire community as the client. A population needs assessment focuses on the need for a service for a specific target population. Development of program resources occurs after the community assessment has been completed. Community assessment involves evaluation of current programs and resources in the community, not past programs.

23. A nurse is working with incarcerated adults who are being released from prison. Which of the following nursing intervNentRionsIwoGuldBb.e CmosMt appropriate to implement? a. Connect offenders with community-based mental health programs. b. Provide community supervision for mentally ill offenders. c. Advocate for increased prison time to decrease recidivism rates. d. Educate about available state resources.

ANS: A Connecting offenders with community-based mental health programs at the time of release from prison can decrease recidivism rates, as many incarcerated adults experience major psychiatric disorders. It would be impossible for the nurse to supervise all mentally ill offenders in the community. Increased prison time is not shown to decrease recidivism rates and education about available state resources is not the most important nursing intervention to provide for this population.

13. A nurse is teaching a group of clients newly diagnosed with diabetes how to give themselves injections. Which of the following formats would be most appropriate for the nurse to use? a. Demonstration b. Health fair c. Lecture d. Non-native language session

ANS: A Demonstration also includes return demonstration. Giving injections can best be learned by seeing the behavior being done. Health fairs target specific populations and are held in a variety of locations. Non-native language sessions are a way to adapt the health fair to a population that does not speak English. Lecture is a non-interactive method of teaching.

22. A nurse plans to address the increasing prevalence of depression in the community. Which of the following would be the best strategy to implement? a. Educate the community about how depression affects men. b. Recognize available community resources. c. Advocate for health policy changes. d. Understand how free clinics are able to provide mental health services to those without health insurance.

ANS: A Depression between men and women is recognized differently in the community. More women than men are classified as having depression. Men with depression often go unrecognized and underreported. Men tend to be stoic and do not verbalize how they feel, are reluctant to talk about health issues, and often do not have a positive relationship with their health care provider. Recognition is key in order to get those with depression the necessary services that they need. Once depression has been detected, these individuals can be referred to the appropriate resources. Health policy changes and the understanding of free clinics would not provide the most immediate solution to the community.

10. A nurse is applying the principle of distributive justice. Which of the following describes the benefits that will occur through application of this principle? a. Basic needs, material and social goods, liberties, rights, and entitlements b. Taxes, military service, location of incinerators or power plants c. Entitlement to equal rights and equal treatment d. The right to private property and personal assets

ANS: A Distributive justice requires that the distribution of benefits and burdens on a society be fair or equal. Entitlement to equal rights and equal treatment refers to egalitarianism. The right to private property and personal assets refers to libertarianism. Taxes, military service, and location of incinerators or power plants are not benefits associated with justice.

10. A nurse is examining a child in the early stages of HIV infection. Which of the following would the nurse expect to find? a. Failure to thrive and developmental delays b. Kaposi's sarcoma and developmental delays c. Toxoplasmosis and oral candidiasis d. Fatigue and shortness of breath

ANS: A Early symptoms of pediatric HIV infection include failure to thrive and developmental delays. Kaposi's sarcoma and oral candidiasis are common opportunistic diseases later in the disease process. Fatigue may be seen in the later stages and is commonly seen in hepatitis and TB.

2. Which statement about education is true? a. It emphasizes the provider of knowledge and skills. b. It emphasizes the recipient of knowledge and skills. c. It is a process of gaining knowledge and expertise. d. It results in behavioral change.

ANS: A Education is the establishment and arrangement of events to facilitate learning. Education emphasizes the provider's role in providing knowledge and skills. Learning emphasizes the recipient of knowledge and skills. Education does not necessarily result in change, as change is typically not easy for most people. Education is designed to effect changes in the knowledge, skills, and attitudes, not necessarily expertise.

19. An occupational health nurse is assessing slippery floors as a safety hazard in the work setting. Which of the following best describes the type of hazard the nurse is assessing? a. Enviromechanical b. Physical c. Chemical d. Psychosocial

ANS: A Enviromechanical agents are those that can potentially cause injury or illness in the workplace. They are related to the work process or to working conditions, and they can cause postural or other strains that can produce adverse health effects when certain tasks are performed repeatedly. Slippery floors are known as enviromechanical hazards. They can cause or potentiate accidents, injuries, strain, or discomfort.

13. A community health nurse is caring for an individual who is experiencing crisis poverty. Which of the following characteristics would this individual most likely display? a. Episodic homelessness b. Physical disability c. Mental disability d. Drug abuse

ANS: A Episodic homelessness is a characteristic of crisis poverty. Physical disability, mental disability, and drug abuse would be considered in persistent poverty.

9. An advanced practice nurse (APN) is pursuing professional certification. Who will the nurse need to contact to obtain certification? a. American Nurses Association b. National League for Nursing c. State board of nursing d. Department of Health and Human Services

ANS: A Examinations are administered through the ANCC, which is part of the American Nurses Association. The National League for Nursing and state board of nursing do not provide professional certification for advanced practice nurses.

4. Which of the following health problems ranks as one of the top five problems for farm workers aged 5 to 19 years? a. Dental problems b. Communicable diseases c. Gastrointestinal problems d. Stress

ANS: A Farmworkers of all ages consistently have more dental disease than the general population.

3. A nurse is providing care to populations who are from different socioeconomic and cultural backgrounds. Which of the following barriers should the nurse be aware of that may prevent full engagement in a relationship? a. Fears and misconceptions related to poverty b. Lack of tangible and emotional resources c. Lack of knowledge about people who live in poverty d. Perception that the poor don't need to be poor

ANS: A Fears and misconceptions of nurses related to poverty is a barrier that may prevent nurses from fully engaging in relationships with people who come from different socioeconomic and cultural backgrounds. It is important for the nurse to be a good listener, individualize care, and avoid making inappropriate assumptions about their needs. It is important to listen to the stories of their lives as well as examine social and cultural definitions and considerations related to poverty.

7. A nurse is referring to the federal income guidelines. Which of the following best describes why the nurse is completing this action? a. To determine financial eligibility for government assistance programs b. To calculate statistical occurrences in the community c. To identify those with inadequate resources for basic needs d. To provide guidelines for reform of social programs

ANS: A Federal income guidelines are used primarily to determine financial eligibility for government assistance programs. The Poverty Threshold Guidelines are used primarily for statistical purposes. The federal income guidelines do not identify those with inadequate resources for basic needs or provide guidelines for reform of social programs.

11. A public health nurse (PHN) is working with a community during the recovery phase of a disaster. Which of the following attributes would be most important for the nurse to possess? a. Flexibility b. Organization skills c. Personable d. Sense of humor

ANS: A Flexibility is key to a successful recovery operation. The role of the PHN in the recovery phase of a disaster is as varied as in the preparedness and response phases, but the PHN's connection to the community puts the PHN in an incredible position of knowledge and awareness on the interprofessional recovery team. The attributes of being organized, personable, and having a sense of humor are not as important as flexibility when dealing with this phase of the disaster.

1. Which statement regarding Florence Nightingale's ideas about ethics is correct? a. Nursing is a call to service, and the moral character of persons entering nursing is important. b. Ethical principles are based on the values of the individual nurse. c. Society will dictate the ethical principles to which nurses must adhere. d. Ethics are very important in times of war, such as in the Crimean War, when she set up public health centers.

ANS: A Florence Nightingale saw nursing as a call to service and viewed the moral character of persons entering nursing as important. Florence Nightingale did not set up public health centers. Florence Nightingale did not believe that nurses must adhere to society's view of ethical principles. Ethical beliefs are based on the values of the individual nurse, not ethical principles.

20. What causes food intoxication? a. Toxins produced by bacterial growth and chemical contaminants b. Bacterial, viral, or parasitic invasion of food c. Overcooking of meat and produce d. Adding too many spices or ingredients to food

ANS: A Food intoxication is caused by toxins produced by bacterial growth, chemical contaminants, and a variety of disease-producing substances found naturally in certain foods such as mushrooms and some seafood. Bacterial, viral, or parasitic invasion of food is not a cause of food intoxication. Food intoxication is not caused by overcooking meat or adding too many ingredients to food.

3. HIV transmission can occur through contact with what? a. Contaminated blood b. Insect bites c. Shared eating utensils d. Contaminated toilets

ANS: A HIV can be transmitted through exposure to blood. HIV is not transmitted by insect bites, sharing of eating utensils, or toilets.

13. Current research on hormone replacement therapy (HRT) has found which statement to be true? a. It does not prevent heart disease. b. It should be used to prevent osteoporosis. c. It is useful with complementary therapies. d. It causes breast cancer.

ANS: A HRT does not prevent heart disease. To prevent heart disease, women should avoid smoking, reduce fat and cholesterol intake, limit salt and alcohol, maintain a healthy weight, and be physically active. HRT should be used to prevent osteoporosis only among women who are unable to take non-estrogen medications. HRT is not useful with complementary therapies and does not cause breast cancer.

24. What is the best method for preventing health care-associated infections? a. Perform good hand washing before and after approaching every patient. b. Prevention is almost impossible due to the high infection rates in hospitals. c. Isolate every patient having surgery. d. Use contact isolation for every patient at risk.

ANS: A Hand washing is the best way to prevent infection. If good hand washing is in place, it is not necessary to isolate patients or implement contact isolation. Prevention is possible with the use of good hand washing.

1. What is the goal of providing education across the three levels of prevention? a. Enabling clients to attain optimal health b. Identifying and treating health problems early to eliminate disability c. Enabling populations to break into individuals d. Teaching people about Healthy People 2020

ANS: A Health education enables clients to attain optimal health, prevent health problems, and identify and treat health problems early to minimize disability. Elimination of disability may not be possible. Health education does not teach about the Healthy People 2020 document or break populations into individuals.

9. Which group has the highest rate of poverty? a. Children b. Teenagers c. Women d. Older adults

ANS: A In 2016, the highest rates of poverty occurred among children under the age of 18.

9. A public health nurse (PHN) is investigating an outbreak of salmonellosis in a community. Which type of surveillance system is being used? a. Active b. Passive c. Sentinel d. Special

ANS: A In the active system, a PHN begins to search for cases through contacts in the community. The nurse names the disease and gathers data about existing cases to try to determine the magnitude of the problem. In the passive system, case reports are sent to local health departments by health care providers. In the sentinel system, trends in commonly occurring diseases or key health indicators are monitored. Special systems are developed for collecting types of data and may be a combination of active, passive, and/or sentinel systems.

21. A nurse is uncomfortable discussing such topics as sexual behavior and sexual orientation when counseling clients and avoids this topic with clients. Which of the following is the most likely outcome of this avoidance? a. Potential risks and risky behaviors will not be identified. b. Transmission of sexually transmitted infections (STIs) will remain unchanged. c. Clients will develop a trusting relationship with health care providers. d. The nurse will be violating the laws in several states.

ANS: A It is important that nurses be able to discuss these topics to help prevent and control STIs. Without discussion of these topics, it is possible that clients will not be aware that they have an STI and may transmit it to others. Thus, the transmission of STDs may increase. A trusting relationship with a health care provider may or may not develop and is not dependent on whether these issues are discussed. There are not laws that mandate nurses to discuss sexual behavior and sexual orientation with clients.

7. A nurse is investigating the overall health status of a population. Which of the following indicators would most likely be used by the nurse? a. Life expectancy b. Health status c. Morbidity rate d. Prevalence

ANS: A Life expectancy is a measure that is often used to gauge the overall health of a population. Health status indicators are the quantitative or qualitative measures used to describe the level of well-being or illness present in a defined population or to describe related attributes or risk factors. Morbidity measures the rate of disability or illness. Prevalence is the proportion of the population that has a specific disease or illness.

10. A nurse is caring for a population that has experienced a health disparity. Which of the following best describes the outcome of a health disparity? a. Low childhood immunization rates b. High dropout rates c. Unemployment d. Income below poverty level

ANS: A Low childhood immunization rates signify a health disparity. High dropout rates, unemployment, and income below poverty level are social conditions that may lead to health disparities.

5. A nurse matches client needs, provider strengths and competence, and agency resources. Which of the following types of planning is being used? a. Strategic planning b. Program planning c. Management planning d. Process planning

ANS: A Matching client needs, provider strengths and competence, and agency resources is considered strategic planning. Program planning reflects the desire to implement a reality-based program that can be readily evaluated and can reduce the number of unexpected events that occur in a defined population. Management planning assists managers to determine whether the resources of the agency are used properly to actually implement the agency programs. Process planning is not a type of planning discussed by the text.

17. Which historical event caused dramatic changes in home care nursing? a. Passage of Medicare legislation b. Establishment of Visiting Nurse Associations c. Creation of the Montefiore Hospital Home Care Program d. Provision of funds by the Civil Works Administration

ANS: A Medicare introduced regulations for home care practice as well as for reimbursement mechanisms.

9. A nurse is assessing a client who has had an acute exposure to a pesticide. Which of the following symptoms is the client most likely to display? a. Memory loss b. Musculoskeletal problems c. Cancer d. Infertility

ANS: A Memory loss is a symptom that is displayed following an acute exposure to a pesticide. Musculoskeletal problems, cancer, and infertility are from chronic exposure.

8. A riverfront community builds a retaining wall to divert flood water away from the town. Which term describes this action? a. Mitigation b. A natural disaster c. Community resilience d. Rapid needs assessment

ANS: A Mitigation is defined as actions or measures that can either prevent the occurrence of a disaster or reduce the severity of the effects. A natural disaster, such as an earthquake or hurricane, causes disruption, destruction, and/or devastation requiring external assistance. Community resilience is defined as the sustained ability of a community to withstand and recover from adversity. A rapid needs assessment is based on the traditional model of community assessment where there is a rapid appraisal of a sector or region's population, social systems, and geophysical features.

15. A nurse is implementing a secondary prevention strategy. Which of the following activities would the nurse most likely implement? a. Monitoring for prenatal care b. Treatment for anemia c. HIV prevention d. Tetanus immunization

ANS: A Monitoring for prenatal care is an example of a secondary prevention strategy. Treatment for anemia is tertiary prevention, and HIV prevention and tetanus immunization are primary prevention strategies.

23. What is the purpose of the National Institute for Occupational Safety and Health (NIOSH)? a. Examining potential hazards of new work technologies and practices b. Educating employees about environmental risks and hazards c. Developing and maintain a database of work-related deaths d. Setting standards that regulate workers' exposure to potentially toxic substances

ANS: A NIOSH examines potential hazards of new work technologies and practices. Occupational Safety and Health Act (OSHA) educates employees about environmental risks and hazards, develops and maintains a database of work-related deaths, and sets standards that regulate workers' exposure to potentially toxic substances.

9. A nurse is applying the ethical principle of nonmaleficence. Which of the following describes the action that the nurse is taking? a. Administering medications using the "five rights" b. Allowing clients to be active participants in their care c. Providing patient privacy when delivering care d. Referring a client to a physical therapist

ANS: A Nonmaleficence requires that one do no harm. It requires that health care professionals act according to the standards of due care, always seeking to produce the least amount of harm possible. Providing privacy when delivering care demonstrates the client's right to privacy. Allowing clients to be active participants in their care refers to the ethical principle of the right to autonomy. Referring a client to a physical therapist demonstrates the nursing role of referral agent.

13. A nurse is assisting an employer who has hired an individual who has been recently diagnosed with HIV. Which of the following interventions would be most appropriate for the nurse to implement? a. Educate about how to reduce the risk of breaching the employee's confidentiality. b. Explain how to inform coworkers about avoiding HIV transmission. c. Facilitate obtaining medical insurance coverage for the HIV-infected employee. d. Describe the early signs and symptoms of HIV infection.

ANS: A Nurses frequently work in the education role, and employers may need assistance in dealing with HIV-infected employees. Disclosing a worker's infection to other workers, terminating employment, and isolating an infected worker are examples of situations that have led to litigation between employees and employers. Thus, the priority will be to protect the employer from litigation.

18. An advanced public health nurse (APHN) has been working with multiple agencies to establish a nursing center in the community. What is the purpose of creating a nursing center? a. To provide opportunities for collaborative relationships b. To offer specialized care for certain medical conditions c. To promote the autonomy of nurse practitioners (NPs) d. To focus on primary prevention for the community

ANS: A Nursing centers provide opportunities for collaborative practice among APHNs, NPs, baccalaureate-prepared nurses, and community members. Nursing centers address the health needs of vulnerable populations. Their central mission is community development; public involvement; interprofessional practice; and health promotion and disease prevention supported by the principles of primary health care.

11. A community health nurse is writing an objective. Which of the following would be most appropriate for the nurse to write? a. Each member of the family will give an insulin injection to the client with accurate dosage 100% of the time for 10 consecutive trials. b. The client will perform a blood sugar test on herself with an accurate blood sugar reading. c. The community will take their children to receive immunizations within 1 month of the immunization due date. d. Fifty percent of the eligible women seen in the clinic will return for their scheduled mammogram appointment.

ANS: A Objectives are specific, short-term criteria that need to be met as steps toward achieving the long-term goal. They are written as statements of an intended outcome or expected change in behaviors and should be defined in measurable terms. The objective "each member of the family will give an insulin injection to the client with accurate dosage 100% of the time for 10 consecutive trials" contains the components of a written objective. "The client will perform a blood sugar test on herself with an accurate blood sugar reading" does not provide any measurable terms. "The community will take their children to receive immunizations within 1 month of the immunization due date" does not provide a measurable percentage of the members of the community. "Fifty percent of the eligible women seen in the clinic will return for their scheduled mammogram appointment" focuses on a long-term goal, rather than a short-term objective and does not provide a time frame for when this would be measured.

4. A public health nurse (PHN) is reviewing Healthy People 2020 to determine where to prioritize programming for the county health department. Based on Healthy People 2020, which of the following areas would the nurse most likely plan to implement programming? a. Reduce the rate of HIV transmission among adults and adolescents. b. Eliminate sexually transmitted diseases (STDs) from developed countries. c. Reduce deaths from gonorrhea. d. Increase awareness about HIV in the LGBTQ population.

ANS: A One of the Healthy People 2020 objectives is reducing the number of cases of HIV infection among adults and adolescents. Eliminating STDs from developed countries, reducing deaths from gonorrhea, and increasing awareness about HIV among the LGBTQ population are not addressed by Healthy People 2020.

3. Which is a feature of public health surveillance? a. Sharing of the results with others b. Defining public health policy c. Evaluating interventions d. Planning national programs

ANS: A One of the features of public health surveillance is sharing of the results with others. Defining public health policy, evaluating interventions, and planning national programs are all purposes of surveillance, not features.

21. A client diagnosed with diabetes reports to the nurse that she has been learning more about controlling her blood sugars by reading information found on the Internet. Which of the following statements by the nurse would be appropriate? a. "Looking at the date the content was posted on the website is important." b. "The Internet is an unreliable source of information and should not be used." c. "The best sources of information are found on pharmaceutical websites." d. "Your physician will provide you better information than the Internet."

ANS: A One of the ways to assess the reliability and validity of Internet sources is to look at its currency, including the dates when the content was posted and updated. The Internet has reliable information, but the quality of the information must be assessed. Pharmaceutical websites may have biased information. Physicians are another source of information, but this may or may not be better than what is found on the Internet.

14. Pelvic inflammatory disease (PID) is a common complication of which sexually transmitted infection (STI)? a. Gonorrhea b. Syphilis c. Chlamydia d. Herpes

ANS: A PID is a common complication of gonorrhea. PID is not a common complication of syphilis, chlamydia, or herpes.

8. Which of the following is most likely to live in poverty? a. Those who work in high-risk jobs. b. Those who have adequate nutrition. c. Those who effectively manage stress d. Those who live-in single-family homes.

ANS: A People who are poor are more likely to live in hazardous environments that are overcrowded and have inadequate sanitation, work in high-risk jobs, have less nutritious diets, and have multiple stressors.

11. A nurse would identify which as a point source of air pollution? a. A smoke stack b. The number of cars and trucks c. The amount of fossil fuel consumed in a community d. Ground ozone levels

ANS: A Point sources of pollution are identifiable sources of air pollution, such as a smoke stack. Nonpoint sources come from more diffuse exposures, such as from cars and trucks. The amount of fossil fuel that is consumed and ground ozone levels do not identify sources of air pollution.

18. A nurse is presenting information to the county health department about potential bioterrorism threats. Which of the following agents would the nurse discuss in this presentation? a. Smallpox b. West Nile Virus c. Severe acute respiratory syndrome (SARS) d. Novel influenza A (H1N1)

ANS: A Susceptibility to smallpox is 100% in the unvaccinated and fatality rate is estimated at 20% to 40% or higher. The agents of highest concern are anthrax, plague, smallpox, botulism, tularemia, and selected hemorrhagic viruses. West Nile Virus, SARS, and H1N1 are not viruses that would be used for bioterrorism.

1. Why is it important for nurses to understand the premises of environmental health? a. Nurses should be able to assess risks and advocate for policies that support healthy environments. b. Toxicologists often consult nurses about environmental pollutants. c. Pollutant exposures such as lead are reported by nurses to the Environmental Protection Agency (EPA). d. Many Americans live in areas that do not meet current national air quality standards.

ANS: A Potential risks to health are concerns for professional nurses. It is the responsibility of the nurse to understand as much as possible about these risks: how to assess them, how to eliminate/reduce them, how to communicate and educate about them, and how to advocate for policies that support healthy environments. Toxicologists do not often consult nurses about environmental pollutants. Pollutant exposures are not routinely reported by nurses. Although there may be problems with air quality standards in the United States, this would not be the primary reason why nurses should understand environmental health.

12. A nurse is completing an exposure history using the mnemonic I PREPARE. What data would a nurse collect when asking questions about the first "P"? a. Present work b. Potential exposures c. Personal protective equipment use d. Past work

ANS: A Present work is the first "P." Potential exposures are part of the "I"—investigate potential exposures. Personal protective equipment use is not part of the mnemonic. Past work is the second "P."

10. A forensic nurse implements a program aimed at preventing domestic abuse. Which of the following levels of prevention is being used? a. Primary prevention b. Secondary prevention c. Tertiary prevention d. Assessment

ANS: A Primary prevention focuses on preventing violence and injuries before they occur—program focused on preventing domestic abuse. Secondary prevention occurs following the occurrence of injuries and crime. This includes direct care provided to victims and perpetrators as well as the collection of evidence. If disability, incarceration, or death occurs, tertiary prevention is applied in settings appropriate to address rehabilitation or identify factors that have put individuals at risk.

25. A nurse promotes the use of universal precautions by all health care workers. Which of the following best describes the action that was taken by the nurse? a. Primary prevention b. Secondary prevention c. Tertiary prevention d. Health care-associated infection

ANS: A Primary prevention refers to those interventions aimed at preventing the occurrence of disease, injury, or disability. Secondary prevention seeks to prevent the spread of infection and/or disease once it occurs. Tertiary prevention reduces complications through treatment and rehabilitation. Health care-associated infections are prevented by good hand washing practices.

6. The nurse is doing a walk through to identify workplace hazards. Which of the following levels of prevention is being implemented? a. Primary prevention b. Secondary prevention c. Tertiary prevention d. Assessment

ANS: A Primary prevention refers to those interventions aimed at preventing the occurrence of disease, injury, or disability. A walk through is part of an assessment, which is part of primary prevention. Tertiary prevention includes those interventions aimed at disability limitation and rehabilitation from disease, injury, or disability. Secondary prevention occurs after a disease process has already begun.

12. An advanced public health nurse (APHN) is offering classes on breast self-exam. Which of the following levels of prevention is being implemented? a. Primary prevention b. Secondary prevention c. Tertiary prevention d. Assessment

ANS: A Primary prevention refers to those interventions aimed at preventing the occurrence of disease, injury, or disability. Classes on breast self-exam focus on preventing the occurrence of breast cancer. Secondary prevention refers to those interventions aimed at early detection and screening. Tertiary prevention refers to those interventions aimed at treatment and rehabilitation. Assessment is not a level of prevention.

20. A nurse screens blood product, donor organs, and tissues for the hepatitis C infection. Which of the following best describes this nursing action? a. Primary prevention b. Secondary prevention c. Tertiary prevention d. Health promotion

ANS: A Primary prevention refers to those interventions aimed at preventing the occurrence of disease, injury, or disability. Screening blood products, organs, and tissues for infection protects the population from exposure to hepatitis C, which prevents them from contracting the disease. Secondary prevention includes screening for diseases to ensure their early identification, treatment, and follow-up with contact to prevent further spread. Tertiary prevention focuses on chronic care and rehabilitation. Health promotion focuses on the primary prevention activities to promote health and prevent disease.

24. With the involvement of the local government and health department, a community-wide program is planned to make all public businesses smoke free, thus eliminating exposure to second-hand smoke. Which of the following levels of prevention of program planning and evaluation is being applied? a. Primary prevention b. Secondary prevention c. Tertiary prevention d. Assessment

ANS: A Primary prevention refers to those interventions aimed at preventing the occurrence of disease, injury, or disability. Secondary prevention refers to screening and early detection. Tertiary prevention refers to rehabilitation and treatment. Assessment is not a level of prevention.

9. A nurse is providing information about safe sex practices. Which of the following levels of prevention is being implemented? a. Primary prevention b. Secondary prevention c. Tertiary prevention d. Assessment

ANS: A Primary prevention refers to those interventions aimed at preventing the occurrence of disease, injury, or disability—providing information about safe sex practices. Secondary prevention focuses on early detection and screening. Tertiary prevention focuses on treatment and rehabilitation. Assessment is not a level of prevention.

21. A forensic nurse is applying the concept of distributive justice. Which of the following questions would be most appropriate for the nurse to ask? a. How will limited funds be spent? b. Who will benefit the most from the services? c. How will an additional service benefit the population? d. How will the privacy of the population be respected?

ANS: A Principles of distributive justice address questions in deciding what goods will be distributed to which persons in what proportions. This is best addressed by asking "How will limited funds be spent?" The principle of beneficence is addressed through "Who will benefit the most from the services?" and "How will an additional service benefit the population?" The principle of confidentiality is addressed through the question "How will the privacy of the population be respected?"

8. A nurse is employed in home health nursing practice. Which of the following terms best describes the care that is being provided? a. Intermittent nursing care b. Custodial care c. Family caregiving d. Palliative care

ANS: A Provision of intermittent skilled visits is a component of home health care nursing. Custodial care refers to the care given in long-term care facilities. Family caregiving may be an important role of keeping the client at home, but it is not a necessary component of home care. Palliative care is an extended continuum of chronic serious illness to acute serious illness during which stabilization and exacerbations may occur.

16. Which type of hepatitis would likely be found where sanitation is inadequate? a. A b. B c. C d. D

ANS: A Public Health Nursing 10th Edition Stanhope Test Bank Hepatitis A would likely be found where sanitation is inadequate. Hepatitis B and C are spread through blood and body fluids. Hepatitis D can only exist in people who are already infected with Hepatitis B.

26. A nurse uses the leadership behavior of reflecting when providing education to community members about how to lead healthy lifestyles. Which of the following actions is most likely being taken by the nurse? a. Providing feedback on how behavior appears to others b. Introducing new topics to the group c. Verifying information through questions and restatement d. Highlighting important points from the discussion

ANS: A Reflecting involves giving feedback on how behavior appears to others. Advising introduces new topics. Clarifying verifies new information. Summarizing highlights the important points.

3. Resilience refers to what characteristic? a. Resistance of certain groups to risk factors b. Increased susceptibility to cumulative risk factors among vulnerable groups c. Variability in the effects of stressors according to socioeconomic status d. Increased sensitivity of the very young and the very old to risk factors

ANS: A Resilience refers to how vulnerable populations can resist or overcome the effects of the vulnerability. These populations do not succumb to the health risks that impinge on them.

1. What has been the focus of the majority of scientific research in forensic nursing? a. Role of the forensic nurse and client population b. Sexual assault team response c. Client satisfaction with forensic nurses d. Collaboration with other disciplines

ANS: A Scientific research in forensic nursing is in its early stages. Currently, research is largely a description of the forensic nursing role and client population.

10. A nurse is implementing secondary prevention strategies in the community. Which of the following best describes what the nurse is doing? a. Screening children for lead poisoning b. Working with communities on emergency preparedness plans c. Developing social networking interventions to modify community norms d. Educating patients with strokes at rehab centers to help optimize their functioning

ANS: A Screenings are secondary prevention activities. Secondary prevention focuses on early detection and screening. Tertiary prevention focuses on treatment and rehabilitation—educating patients with strokes. Primary prevention refers to those interventions aimed at preventing the occurrence of disease, injury, or disability—working with communities on emergency preparedness plans and developing social networking interventions.

20. A nurse is performing a population-focused intervention. Which of the following activities is the nurse most likely completing? a. Setting up a health fair with nursing students at a local university b. Teaching a new mother how to breastfeed c. Completing a physical examination on a preschool child d. Contacting local officials about the rise in crime in one area of the city

ANS: A Setting up a health fair with nursing students at a local university is an example of a population-focused intervention. Population-focused interventions address the needs of the community rather than only the needs of individuals. Teaching a new mother how to breastfeed and doing physical examinations on each preschool child are examples of individual-focused interventions. Contacting local officials about the rise in crime is an example of a system-focused intervention.

18. A nurse is polite and non-confrontational when working with the family of a migrant worker. Which of the following concepts is the nurse demonstrating? a. Simpatía b. Respeto c. Dignidad d. Personalismo

ANS: A Simpatía means having polite, non-confrontational relationships with others. Respeto means respect, dignidad means dignity, and personalismo is relating to the individual.

7. A nurse is providing a skilled nursing service. Which of the following best describes the intervention the nurse is performing? a. Teaching the client and family about medication administration b. Consulting with other nurses about the care of a client in the home c. Coordinating services for maximum benefit at minimal cost d. Reviewing nursing assistant charting at the end of each day

ANS: A Skilled nursing service is the Medicare term that describes the duties of the registered nurse and refers to the requirement of nursing judgment. Those services involve assessment, teaching, and selected procedures. Teaching the client and family to implement a therapeutic plan, such as taking medications, is an example of a skilled nursing service. Consultation, coordination, and review of documentation do not fit the definition of a "skilled nursing service."

30. Which Core Competency of communication is used by nurses engaged in group work? a. Soliciting input from individuals and organizations b. Using simple language when presenting information c. Asking the group to develop the program of interest d. Presenting material to lay audiences

ANS: A Soliciting input from individuals and organizations is a communication competency. Various strategies should be used to present the information. The group should assist in determining the topic but not developing the program. The material should be presented to professional and lay audiences.

9. A nurse is providing emergency care to an assault victim. Which of the following would be the most appropriate action for the nurse to take? a. Support the privacy of the victim. b. Assess the socioeconomic status of the victim. c. Question the victim about the details of the assault. d. Educate the victim about violence prevention.

ANS: A Supporting the privacy of the assault victim is the most appropriate action for the nurse to take when providing care. When providing emergency care, it is not the most appropriate time to assess the client's socioeconomic status or educate the victim about violence prevention. The forensic nurse will need to document the victim's account of the assault; however, there will be law enforcement involved to collect this data.

9. A nurse counsels a client to have the enzyme-linked immunosorbent assay (EIA) test. Which of the following best describes the rationale for this test? a. To indicate the presence of the antibody to HIV b. To reveal whether the client has AIDS c. To isolate the HIV virus d. To confirm HIV after having a positive Western blot

ANS: A The EIA is used to indicate the presence of the antibody to HIV. To minimize false positive results, the Western blot is used as a confirmatory test to verify the results. The EIA does not isolate the virus, nor does it reveal whether the individual has symptomatic AIDS.

24. An occupational health nurse is assessing the need for safety data sheets (SDSs) at the worksite. Which of the following items would require an SDS? a. Hand soap b. Eye goggles c. Aprons d. Earplugs

ANS: A The Hazard Communication Standard requires that all worksites with hazardous substances inventory their toxic agents, label them, and provide information sheets, called MSDSs, for each agent. Hand soap is a toxic agent. Eye goggles, aprons, and earplugs are not toxic agents and therefore, do not need an SDS.

15. A nurse is working with a family who is unable to find adequate health care coverage for the children in the family. Which of the following pieces of legislation will assist in provision of health care coverage for these children? a. Social Security Act b. Balanced Budget Act c. Health Insurance Portability and Accountability Act (HIPAA) d. Affordable Care Act (ACA)

ANS: A The Social Security Act provides funds to insure currently uninsured children. The Balanced Budget Act shifted payment in home health care. The HIPAA was intended to help people keep their health insurance when moving from one place to another. The ACA of 2010 provides the opportunity for all to purchase health insurance.

13. Which federal program created support for older and poor Americans? a. Social Security Act b. Medicare Amendment c. Medicaid Amendment d. Hill-Burton Act

ANS: A The Social Security Act sought direct payments to eligible individuals to ensure a minimum level of support for people at risk of problems from inadequate financial resources. The Social Security Act encompasses the Medicare and Medicaid Amendment. The Hill-Burton Act provided funding to assist with building hospitals.

8. What is the extent of the economic burden of injuries in the United States? a. It is extensive, including medical treatment and lost productivity. b. It is minimal, as limited funds are spent treating injuries. c. It varies depending on the extent of the injury. d. It has been reduced through risk reduction strategies.

ANS: A The economic burden of injuries in the United States is extensive, especially when considering the costs of medical care and treatment, productivity losses (lost wages, accompanying fringe benefits), and lost ability to perform normal household responsibilities.

16. A nurse is working with a group of clients diagnosed with diabetes and is teaching a class about avoiding the long-term effects of diabetes. The nurse begins the class by reviewing the basic physiology of diabetes, which was taught the week before. Which of the following principles of effective education is the nurse using? a. Stimulating recall of prior learning b. Gaining attention c. Presenting the material d. Providing learning guidance

ANS: A The educator should have the learners recall previous knowledge related to the topic of interest. The principle of gaining attention should have been applied as the program started the week before. Recall should be done before new material is presented and before learning guidance is provided.

4. Which event has the potential to weaken existing public health programs? a. The shift in funding to support bioterrorism response efforts b. The outbreaks of H1N1 influenza c. The number of citizens with human immunodeficiency virus (HIV) d. The lack of PHNs

ANS: A The events of the twenty-first century have shifted funding to bioterrorism programs. This shift of funding has the potential to weaken existing important public health programs.

5. A nurse has referred a home care client to hospice care. Which of the following best explains the rationale for this referral? a. To provide comfort and peace at a difficult time b. To reduce the cost of care for the terminally ill c. To keep clients at home until the end of life d. To provide care for terminally ill clients expected to live longer than 6 months

ANS: A The focus of hospice care is comfort, peace, and a sense of dignity at a very difficult time. Comprehensive services emphasize continuity oOf care. Hospice care is provided for those with a life expectancy of less than 6 months. Hospice is the only Medicare benefit that includes medications, medical equipment, 24 hours/7 days a week access to care, and support for family members after death. Hospice care does not reduce the cost of care.

15. What was the purpose of the Centers for Disease Control and Prevention (CDC) creating the first list of standard case definitions for notifiable diseases? a. Some diseases were underreported, and others were overreported. b. There was no central office to collect data. c. Some people refused to notify officials when diseases were seen. d. No one knew where to report the information that was collected.

ANS: A The initial work was done by the CDC because diseases were overreported or underreported. Before this time, state and local health departments used many different criteria for identifying cases of reportable diseases. There was a central office to collect the data and providers knew that this information needed to be reported, but what was being reported by departments was different.

20. A nurse is providing care for a Mexican client who has sustained a farm injury. If a major decision about the client's treatment needs to be made, who would most likely make that decision? a. The male head of the family b. The wife/mother c. The person needing services d. The crew leader

ANS: A The male is considered to be the major decision maker in the family, whereas the female is considered the caretaker.

1. The process of program management is similar to: a. the nursing process. b. the medical model of care delivery. c. the requirements for licensure as a registered nurse. d. summative and formative evaluation.

ANS: A The nursing process has the same basic steps as the process of program management. Like the nursing process, the process of program management consists of a rational decision-making system designed to help nurses know when to make a decision to develop a program, where they want to be at the end of the program, how to decide what encompasses a successful program, how to develop a plan to go forward so they will know where they want to be, how to know that they are getting there (formative evaluation), and what to measure to know that the program has successful outcomes (summative evaluation).

10. An advanced practice nurse (APN) has obtained certification. Which of the following provides the rationale for receiving certification? a. Assures competence at an advanced level. b. Verifies minimal competency levels of practice. c. Creates a network for nurses to collaborate. d. Making becoming a nurse a challenging process.

ANS: A The purpose of certification is to assure the public that nurses who claim to be competent at an advanced level have had their credentials verified through examination. Licensure verifies minimal competency levels of practice, not certification. Certification does not regulate the ability to become a nurse; that occurs with licensure. Certification does not create a network of nurses for the purpose of collaboration.

13. A nurse is conducting a community assessment in a healthy community. Which of the following findings should the nurse anticipate? a. Low crime rates b. High rates of injury c. Significant health disparities d. Low mortality rates

ANS: A The rate of injury and crime is lower in communities that promote good health. Where inequities of resources and education exist, health disparity, violence, and other crimes rise and accidental injuries occur more often.

17. During a class on newborn care given at a local health department, a nursing student asks the participants to practice with baby dolls. Which of the following steps of promoting effective education is the student using? a. Eliciting performance b. Assessing performance c. Enhancing retention and transfer of knowledge d. Gaining attention

ANS: A The step of eliciting performance includes encouraging the learners to demonstrate what they have learned. Gaining attention occurs at the beginning of the program. Assessing performance occurs at the end of the program. Enhancing retention and transfer of knowledge occurs as the learners are asked to apply the information to their lives and situations (not to the care of the dolls).

14. A nurse is triaging victims followIng a disaster. Which of the following describes the action the nurse would take? a. Allocation of treatment based on the victim's potential for survival b. Assignment of tasks to the appropriate disaster response personnel c. Determination of the level of disaster and agency involvement d. Participation in community preparedness for the disaster response

ANS: A The triage process in disaster involves separating the casualties and allocating treatment based on the victim's potential for survival. Assignment of tasks to the appropriate disaster response personnel, determination of the level of disaster and agency involvement, and participation in community preparedness for the disaster response are not part of the triage process.

1. What is implied by the Web of Causation model? a. Variables interact resulting in higher probability of illness. b. One disease causes another, especially in vulnerable populations. c. The greater the poverty, the more likely people are to have diseases. d. Immunizations are necessary because vulnerable populations spread disease.

ANS: A The web of causation model implies that not only are there more variables, but also the variables interact, resulting in higher probability of illness. The relative risk for poor health is greater for vulnerable populations.

17. A nurse is working with a client who is part of the Mexican culture and is experiencing a folk illness. Which of the following actions by the client should the nurse anticipate? a. Prefer to seek care with a folk healer. b. Consult the female leader of the family. c. Rely on Western medicine to cure the ailment. d. Become isolated and not seek care.

ANS: A There are four common folk illnesses that a nurse may encounter with the Mexican client: (1) mal de ojo (evil eye), (2) susto (fright), (3) empacho (indigestion), and (4) caida de mollera (fallen fontanel). When faced with one of these illnesses, the traditional Mexican individual would prefer to seek care with a folk healer. Healers known as curanderos, herbalists, and espiritualistas are often sources of health care for folk illnesses.

16. A nurse is providing care using the idea of "servicing citizens, not customers." Which of the following ideas is being applied by the nurse? a. Ethical tenets of policy development b. Basic concepts of the feminist theory c. Underlying premise of virtue ethics d. Components of distributive justice

ANS: A There are three tenets of both policy and ethics. The approach is based on the voice of the community as the foundation on which policy is developed. The basic concept of feminist theory allows us to think critically about connections among gender, disadvantage, and health as well as the distribution of power in public health processes. The goal of virtue ethics is to enable persons to flourish as human beings. Distributive justice requires that there be a fair distribution of the benefits and burdens in society based on the needs and contributions of its members.

13. According to Leininger and Watson, what is the moral ideal of nursing? a. Caring b. Advocacy c. Responsibility d. Accountability

ANS: A This conceptualization occurred as a response to the technological advances in health care science and the desire of nurses to differentiate nursing practice from medical practice. Advocacy, responsibility, and accountability are not part of the moral idea of nursing proposed by Leininger and Watson.

14. The community health nurse is caring for a client who is living in persistent poverty. Which of the following best describes this client? a. Severe mental illness b. Transient homelessness c. Victim of domestic violence d. Temporary housing

ANS: A Those who are in persistent poverty typically are older, with physical and mental disabilities, alcohol and other drug abuse, severOe mental illness, chronic health problems, and significant family difficulties. Transient homeless persons, victims of domestic violence, and persons with temporary housing would be considered in crisis poverty.

16. A public health nurse (PHN) is involved in disaster planning. Which of the following best describes why this is important? a. To be most effective as a disaster responder b. To anticipate the potential disasters that may occur c. To educate the public about their responsibilities if a disaster occurs d. To demonstrate the importance of community partnerships

ANS: A To be most effective as a disaster responder, the PHN needs to be part of the team before an emergency occurs. PHNs can be involved in educating the public, working with community partnerships, and anticipating potential disasters; however, the nurse must be part of the team before the emergency to accomplish these things.

16. A nurse is implementing a tertiary prevention strategy. Which of the following activities would the nurse most likely implement? a. Diabetes screening b. Rehabilitation for musculoskeletal injury c. Birth control d. Pregnancy testing

ANS: B Rehabilitation for musculoskeletal injury is an example of tertiary prevention. Diabetes screening and pregnancy testing are secondary prevention strategies, and birth control is a primary prevention.

5. What term is used to refer to the basic science applied to understanding the health effects associated with chemical exposures? a. Toxicology b. Pharmacology c. Chemistry d. Environmental epidemiology

ANS: A Toxicology is the study of the health effects associated with chemical exposures. Pharmacology is the branch of medicine concerned with the multiple aspects of drugs. Chemistry is the branch of science with deals with the way that matter is composed. Environmental epidemiology is concerned with the discovery of environmental exposures that cause or protect against illness or disease.

4. The nurse is caring for an elderly client who has written advanced medical directives. Which of the following best describes what this means? a. The client has made decisions regarding treatment if incapacitated. b. The client has a living will. c. The client has a Do Not Resuscitate order. d. The client's family has decided on a medical treatment regimen.

ANS: A Two parts of advance directives are the living will and a durable power of attorney. An advance directive indicates treatment choices if clients become incapacitated. A living will allows the client to express the wishes regarding the use of medical treatments in the event of a terminal illness. A Do Not Resuscitate order is a specific order from a physician not to use cardiopulmonary resuscitation.

10. A nurse is implementing Wagner's Chronic Care Model (CCM). Which of the following actions would most likely be taken by the nurse? a. Educate a community group about hypertension control. b. Create a budget for chronic disease management. c. Administer immunizations to community members. d. Conduct depression screenings in the community.

ANS: A Use of electronic health records, provider reminders for key evidence-based care components, interprofessional teams communicating regularly, and community health classes to educate people with chronic diseases are various ways the CCM is being implemented. Creating a budget is not a way to use CCM. Administration of immunizations and conducting depression screenings do not address the management of the most common and costly chronic diseases: heart disease, diabetes, stroke, cancer, and arthritis.

8. A nurse is teaching members of the community about vertical transmission of a disease. Which route would the nurse most likely discuss? a. Breast milk b. Sexual contact c. Mosquito bites d. Contaminated food

ANS: A Vertical transmission is the passing of infection from parent to offspring via sperm, placenta, milk, or contact in the vaginal canal at birth. Transmission through sexual contact is horizontal transmission. Transmission from mosquito bites is vector transmission. Transmission from contaminated food is common vehicle transmission.

20. A nurse vocalizes a concern that a group of .refugees in the community is not receiving equal access to health care services when compared to the other residents in the community. Which of the following concepts is being applied? a. Victimization b. Perceptivity c. Questioning d. Beneficence

ANS: A Victimization occurs when individuals or groups are treated unfairly; generally, when there is an imbalance of power. Perceptivity, as an element of intuitiveness, is one tool the forensic nurse uses to investigate injury; it involves an increased awareness of human behavior and environment that is interpreted by knowledge and lived experience in making expert decisions. Questioning has led to a greater understanding of what most would consider the unexplainable and unimaginable. Beneficence is an ethical principle that relates to caring.

2. Which of the following is a characteristic of a vulnerable population? a. Have worse health outcomes and an increased sensitivity to risk factors than the general population. b. Have a single risk factor but experience worse health outcomes than the general population. c. Have multiple risk factors but equal health outcomes to the general population. d. Have worse outcomes with better access to health care than the general population.

ANS: A Vulnerable population groups are more sensitive to risk factors and have worse health outcomes. Vulnerable populations experience multiple risk factors. Vulnerable populations have worse health outcomes than the general population. Vulnerable populations have more problems accessing health care than the general population.

12. An occupational health nurse is educating employees about work-related hazards. Which of the following hazards would the nurse most likely discuss in the presentation? a. Workplace stress leading to hypertension and cardiovascular disease b. Asbestos, plastics, lead, and solvents leading to dermatitis c. Cement dust and metals leading to bronchitis d. Hormones and nitroglycerine leading to reproductive effects

ANS: A Work-related stress or burnout has been defined as an important problem for many individuals. Responses to negative interpersonal relationships, particularly those with authority figures in the workplace, are often the cause of vague health symptoms and increased absenteeism.

2. Which organizations offer volunteer opportunities in disaster work for nurses? (Select all that apply.) a. American Red Cross (ARC) b. Citizens Corps c. American Nurses Association d. National Public Health Training Centers e. Sigma Theta Tau International

ANS: A, B ARC and Citizens Corps offer volunteer opportunities. The American Nurses Association and National Public Health Training Centers offer education and training opportunities. Sigma Theta Tau International is an honor society for nurses.

1. After a need and client demand for a program have been determined through the needs assessment process, the next step in the development of the program is to choose a procedural method that will assist the nurse in planning the program to be offered. Which are possible planning methods? (Select all that apply.) a. Program planning method (PPM) b. Multi-attribute utility technique (MAUT) c. Sustainability method d. Effectiveness method e. Population needs assessment

ANS: A, B PPM and MAUT are types of planning methods. The sustainability and effectiveness methods are parts of the PPM. Sustainability is an evaluation method to determine if there are resources available to continue the program. Effectiveness is an evaluation method to determine if the program is able to meet its objectives and to evaluate the results of program efforts. A population needs assessment is one of the first steps in program planning but does not describe the entire process of program planning.

22. The community leaders in a lesser-developed country decide not to tell the citizens of a small village about a chemical spill at a major industrial facility that could produce harmful effects. Which of the following principles is being violated? a. Morality b. Advocacy c. Caring d. Virtue

ANS: B Advocacy requires that the community be properly informed, and this was violated in the above scenario. Morality is shared and generational societal norms about what constitutes right or wrong conduct. Caring represents the essence of nursing. Virtue demonstrates behavior showing high moral standards.

2. A nurse is caring for an elderly female population. Which of the following considerations should be made? (Select all that apply.) a. Postmenopausal women are more likely than men to experience hypertension. b. Women use more health services than men. c. Women report lower rates of disability than men. d. Women and men have similar socialization patterns. e. Postmenopausal women are less likely than men to experience depression.

ANS: A, B Postmenopausal women are more likely than men to experience hypertension; prior to menopause, the rate is higher among men. Women use more health services than men. Women report higher rates of disability than men. Women and men have different socialization, expectations, and lifestyles.

2. A nurse would like to obtain certification as a forensic nurse. Which specialty certifications are available for the forensic nurse? (Select all that apply.) a. Legal Nurse Consultant b. Death Investigator Public Health Nursing 10th Edition Stanhope Test Bank NURSINGTB.COM c. Adolescent and adult SANE A d. Forensic Consultant e. Forensic nurse

ANS: A, B, C Certification is available as an adolescent and adult SANE A, Legal Nurse Consultant, and Death Investigator. Forensic Consultant and forensic nurse are not certifications that are available.

1. A nurse is using public surveillance. Which of the following describes the actions involved? (Select all that apply.) a. Estimating the magnitude of an influenza outbreak b. Determining the geographical distribution of gonorrhea cases c. Detecting an epidemic of whooping cough d. Reducing the prevalence of obesity e. Determining the incidence of cardiovascular disease

ANS: A, B, C Estimating the magnitude of the problem, determining geographic distribution of an illness, and detecting epidemics are all uses of public surveillance. Reduction of the prevalence of obesity and determining the incidence of cardiovascular disease are not how public health surveillance is used.

1. A public health nurse (PHN) working with a family living in poverty is concerned about their exposure to environmental hazards. Which of the following factors should the nurse identify as possibly putting the family at risk? (Select all that apply.) a. Limited funds to pay for health care b. Poor nutrition c. Homes located closer to hazardous waste sites d. Less education e. Unsafe working conditions

ANS: A, B, C Families living in poverty are more likely to experience environmental justice issues such as disproportionate environmental exposures. Substandard housing, living closer to hazardous waste sites, working in more hazardous jobs, poorer nutrition, and less access to quality health care all contribute to this issue. Although limited education is related to poverty, it is not discussed as causing an increase in environmental exposure.

1. Which functions are shared by advanced public health nurses (APHNs) and nurse practitioners (NPs)? (Select all that apply.) a. Leadership b. Research c. Comprehensive assessment d. Systems focus e. Community assessment

ANS: A, B, C Leadership, research, and comprehensive assessment are functions that are completed by both APHNs and NPs. The APHN is more systems focused and would conduct a community assessment, whereas the NP has an individual/family focus and would conduct an individual health assessment.

2. Which factors cause stress for advanced practice nurses (APNS)? (Select all that apply.) a. Legal issues b. Professional isolation c. Conflicting expectations d. Role confusion e. Advanced education

ANS: A, B, C Legal issues, professional isolation, conflicting expectations, liability, collaborative practice, and professional responsibilities all may cause stress for advanced practice nurses. Role confusion and advanced education are not factors causing stress for advanced practice nurses.

2. A nurse "sets the stage" when assessing members of vulnerable population groups. Which of the following interventions would be completed? (Select all that apply.) a. Creating a comfortable, non-threatening environment b. Providing culturally and linguistically competent assessment c. Collaborating with others as appropriate d. Providing financial and legal advice e. Developing a free clinic in a low-income neighborhood

ANS: A, B, C Nurses set the stage by creating a comfortable, non-threatening environment, providing culturally and linguistically competent assessment, and collaborating with others as appropriate. A nurse should not provide financial or legal advice. Developing a free clinic does not "set the stage."

1. Which of the following factors are cited as obstacles to improving men's health? (Select all that apply.) a. Reluctance to consult their primary care provider b. Emphasis of physical strength and competitiveness on masculinity c. Lack of a primary care provider d. Lack of access to health care by African American and Hispanic men e. Emphasis on the male role as the head of the family

ANS: A, B, C Obstacles to improving men's health include reluctance to consult their primary care provider, the concept that masculinity emphasizes physical strength and competitiveness, and the fact that many men do not have a primary care provider.

1. Which populations are at greatest risk for disruption after a disaster? (Select all that apply.) a. Single-parent families b. Children c. Substance abusers d. Middle-class families e. Young adults

ANS: A, B, C Single-parent families, children, and substance abusers are all at greater risk for disruption after disaster than a middle-class family or young adults.

1. Which are examples of federal agencies? (Select all that apply.) a. The U.S. Department of Health and Human Services (USDHHS) b. The Centers for Disease Control and Prevention (CDC) c. The Health Resources and Services Administration (HRSA) d. The Pentagon e. The Congress

ANS: A, B, C The USDHHS, the CDC, and the HRSA are all federal agencies. The Pentagon and the Congress are not federal agencies.

1. A nurse is coordinating care through a high-intensity transitional care program. Which of the following best describes the populations that the nurse is working with? (Select all that apply.) a. Adults with cognitive impairments b. Women with high-risk pregnancies c. Older adults with heart failure d. People who are moving from one state to another e. Men who have just experienced divorce

ANS: A, B, C Transitional care ensures the coordination and continuity of health care as clients transfer between different locations and different levels of care in the same location. High-intensity transitional care programs are designed for populations who have complex or high-risk health problems.

16. What is considered a nonpoint source of pollution? a. Hazardous waste site b. Animal waste from wildlife c. Chlorine poured down a well d. Stagnant water

ANS: B Nonpoint sources come from more diffuse exposures to pollution. Animal waste is the only diffuse exposure given. The others are considered point sources, point sources are individual, identifiable sources such as smoke stacks.

3. Which illegal and unethical activities have occurred in health care systems in the past? (Select all that apply.) a. Inappropriate use of home health services b. Inaccurate billing for services c. Excessive administrative staff d. "Kickbacks" for referrals e. Primary care services provided in the home

ANS: A, B, C, D Examples of Medicare fraud and abuse in home health and hospice include inappropriate use of services, excessive payments to administrative staff or owners, "kickbacks" for referrals, and billing for visits and/or medical supplies that are not authorized or provided. The home care nurse must abide by established federal regulations when delivering care to clients.

1. Which factors are challenges of the migrant lifestyle? (Select all that apply.) a. Leaving one's home every year b. Traveling c. Experiencing uncertainty regarding work and housing d. Lack of resources e. Middle income wages

ANS: A, B, C, D Leaving one's home every year, traveling, experiencing uncertainty regarding work and housing, and lack of resources are some of the challenges of the migrant lifestyle. The specifics for payment differ depending on the location and type of work. Reports of average income for farmworkers have varied.

3. A public health nurse (PHN) collaborates with agencies to address the problem of obesity in the community. Who would be the most appropriate people to include in this collaborative partnership? (Select all that apply.) a. Local health department staff members b. Restaurant owners c. Parks and recreation department staff members d. Church representatives e. Appliance store managers

ANS: A, B, C, D PHNs practice in partnership with each other at the local, state, and federal levels and with other public health staff, other governmental agencies, and the community to safeguard the public's health and to improve the community's health status. All of these people would provide valuable insight into the problem of obesity in the community.

1. An occupational health nurse is working in an occupational health and safety program. Which of the following services is the nurse most likely to provide? (Select all that apply.) a. Health/medical surveillance b. Health screening c. Case management d. Job task analysis e. Counseling

ANS: A, B, C, D The services provided by onsite occupational health programs range from those focused only on work-related health and safety problems to a wide scope of services that includes primary health care. An occupational health and safety program may include health/medical surveillance, health screening, case management, and job task analysis.

2. A staff nurse has demonstrNateUdRthSeIaNbiGlitTyBto.bCe OcoMmpetent as a participant in surveillance and investigation activities. Which abilities would you expect that nurse to possess? (Select all that apply.) a. Identifying appropriate data sources b. Communicating effectively using written reports c. Collaborating with community agencies d. Creating an appropriate budget e. Correctly interpreting laboratory findings

ANS: A, B, C, D The staff nurse must have analytic assessment skills, communication skills, and community dimensions of practice to be a participant in surveillance and investigation. The ability to interpret laboratory findings is not a core competency.

1. Which elements of surveillance does a nurse use? (Select all that apply.) a. Mortality registration b. Epidemic field investigation c. Laboratory reporting d. Individual case investigation e. Application of research

ANS: A, B, C, D There are 10 basic elements of surveillance. Mortality registration, epidemic field investigation, laboratory reporting, and individual case investigation are among them. Application of research is not part of the 10 basic elements of surveillance.

1. Which are current trends in providing care for vulnerable populations? (Select all that apply.) a. Community-based care and interorganizational partnerships b. Outreach and case finding c. Elimination of disparities d. Culturally and linguistically appropriate care e. Increased incidence of acute illnesses

ANS: A, B, D Community-based care and interorganizational partnerships, outreach and case finding, and provision of culturally and linguistically appropriate care are all trends to improve care among vulnerable populations. Elimination of disparities is not a trend related to caring for vulnerable populations. Increased incidence of acute illness is not a trend of care provision for vulnerable populations.

1. Which of the following are ethical tenets that underlie the core function of assessment? (Select all that apply.) a. Competency: the persons assigned to develop community knowledge are prepared to collect data on groups and populations. b. Moral character: the persons selected to develop, assess, and disseminate community knowledge possess integrity. c. Service to others over self: a necessary condition of what is "good" or "right" policy. d. "Do no harm": disseminating appropriate information about groups and populations is morally necessary and sufficient. e. Providers of public health services should be competent and available.

ANS: A, B, D Service to others over self is an ethical tenet of policy development. Providers of public health services should be competent and available is an ethical tenet of assurance. Competency, moral character, and "do no harm" are the ethical tenets of assessment.

1. A nurse is using basic educational principles when conducting an effective educational program. Which of the following principles would the nurse most likely be using? (Select all that apply.) a. Use a clear, succinct style. b. Use an active voice. c. Refer to Internet sources. d. Use aids to highlight key points. e. Repeat information several times.

ANS: A, B, D Using a clear, succinct style, active voice, and using aids to highlight key points are all basic educational principles that should be used by the nurse. Reliable sources must be used, and information found on the Internet is not always reliable. Repeating information is not an education principle.

1. The nurse is counseling a female who has recently tested positive for HIV. Which of the following will the nurse educate that the client is responsible for? (Select all that apply.) a. Have regular medical evaluations and follow-ups. b. Donate blood and plasma to others who are positive for the disease. c. Inform health care providers about the HIV infection. d. Consider the risk of perinatal transmission. e. Disclose her HIV infection to her employer

ANS: A, C, D A person who is infected with HIV should have regular medical evaluations and follow-up appointments, not donate blood or plasma, inform health care providers about the HIV infection, and consider the risk of perinatal transmission and follow-up with contraceptive use.

2. An occupational health nurse has become a member of the American Association of Occupational Health Nurses (AAOHN). Why would a nurse join this organization? (Select all that apply.) a. To promote the health and safety of workers b. To lobby in Congress for safer work places c. To advance the profession by supporting research d. To promote and provide continuing education in the specialty e. To obtain certification as a specialist in occupational health nursing

ANS: A, C, D The AAOHN is the professional organization for occupational health nurses. It supports the work of the occupational health nurse and advances the specialty by: promoting the health and safety of workers, defining the scope of practice and setting the standards of occupational health nursing practice, developing the code of ethics for occupational health nurses with interpretive statements, promoting and providing continuing education in the specialty, advancing the profession through supporting research, and responding to and influencing public policy issues related to occupational health and safety. A nurse joining the AAOHN would most likely not go to lobby in Congress; rather, AAOHN provides a lobbyist to address pertinent issues. Joining the AAOHN will not assist the nurse in obtaining specialty certification.

3. A nurse is planning to implement Wagner's Chronic Care Model (CCM). Which of the following essential elements would the nurse consider? (Select all that apply.) a. Clinical information systems b. Evidence-based change c. Self-management support d. Community e. Clinical practice guidelines

ANS: A, C, D The essential elements of CCM are: the community, the health system, self-management support, delivery system design, decision support, and clinical information systems. Evidence-based change concepts are found under each of the essential elements and help to foster productive interactions between clients and providers.

5. A nurse is working with an older person following a disaster. Which of the following emotions of the client should be anticipated by the nurse? a. Anger b. Fear of loss of independence c. Violence d. Regression

ANS: B A common reaction of older persons experiencing disaster may be fear of loss of independence. Young children may respond with regression. The community in general may respond will feelings of anger, sorrow, guilt, and perceived blame for the disaster or the outcomes of the disaster. Violence should not be anticipated with any of the populations discussed in the text.

3. A nurse is working with a client whose principal employment is in agriculture on a seasonal basis and has established temporary housing in the area while employed. Which of the following terms best describes this laborer? a. Seasonal farmworker b. Migrant farmworker c. Transient worker d. Share cropper

ANS: B A migrant farmworker is a laborer whose principal employment is in agriculture on a seasonal basis and, for the purposes of such employment, establishes a temporary abode. A seasonal farmworker returns to his permanent residence, works in agriculture for at least 25 days or parts of days, and does not work year-round only in agriculture. A transient worker moves from place to place to find work. A share cropper is a tenant farmer who uses part of the crop to pay for rent.

8. A nurse is providing services for uninsured women. Which of the following best describes the role that the nurse is using? a. Role model b. Primary caregiver c. Outreach worker d. Case manager

ANS: B A nurse providing services for uninsured women is acting in the role of primary caregiver. The public health function of a nurse who assists a client in identifying the services needed the most at the least cost is a case manager. A nurse serves as a role model for the client by being a person for the client to look up to. As an outreach worker, the nurse is able to gain acceptance with the population served.

17. The SARS (severe acute respiratory syndrome) outbreak was an epidemic that spread over several countries. Which of the following terms describes what happened? a. Hyperendemic b. Pandemic c. Mixed outbreak d. Holoendemic

ANS: B A pandemic refers to the epidemic spread of the problem over several countries or continents. If a problem is considered hyperendemic, there is a persistently (usually) high number of cases. A mixed outbreak is a common source followed by secondary exposures related to person-to-person contact. Holoendemic implies a highly prevalent problem found in a population commonly acquired early in life.

12. A nurse is working with multiple vulnerable groups. Which of the following would be most sensitive to the adverse effects of vulnerability? a. Pregnant teenager living with her parents for financial support b. Poor, older woman with no means of transportation c. 2-year-old boy of underinsured parents d. Recently unemployed father of five

ANS: B A poor, older woman with no means of transportation has a combination of risk factors. A pregnant teenager, 2-year-old boy, and recently unemployed father are only displaying one risk factor, not multiple risk factors as the older woman displays.

20. A set of actions one undertakes on behalf of another is identified by what term? a. Social justice b. Advocacy c. Resilience d. Risk

ANS: B A set of actions one undertakes on behalf of another is advocacy. Social justice describes justice with respect to the concepts of egalitarianism and equality. Resilience refers to how vulnerable populations are able to resist or overcome the effects of the vulnerability. Risk describes that some people have a higher probability of illness than others.

30. A nurse completes a cost-accounting study on a program that targets obese adults, helping them to adopt healthier lifestyle choices. The nurse would like to compare the cost of the program to the cost savings in the health care system by preventing the occurrence of chronic illnesses. Which of the following types of studies would be most appropriate to use in this situation? a. Cost-accounting study b. Cost-benefit study c. Cost-effectiveness study d. Cost-efficiency study

ANS: B A study used to assess the desirability of a program by examining costs and benefits is a cost-benefit study. Cost-accounting studies are performed to find the actual cost of a program. A study designed to measure the quality of a program as it relates to cost is a cost-effectiveness study. A cost-efficiency study is designed to examine the actual cost of performing program services and to focus on productivity versus cost.

1. Who is considered to be the first industrial nurse? a. Betty Moulder b. Ada Mayo Stewart c. Lillian Wald d. Florence Nightingale

ANS: B Ada Mayo Stewart was hired in 1885 by the Vermont Marble Company; she is often considered the first industrial nurse. Betty Moulder provided care of ailing coal miners and their families. Lillian Wald is known as the founder of public health nursing. Florence Nightingale is one of the pioneers of the nursing profession.

15. A nurse considering working as a public health nurse (PHN) refers to the American Public Health Association (APHA) Public Health Nursing Section to determine what level of educational preparation has been recommended. Which of the following best describes educational preparation? a. Associate degree b. Baccalaureate degree c. Master's degree d. Certification

ANS: B All PHNs should have a background in the social and behavioral sciences, epidemiology, environmental health, current treatment modalities, and health care delivery options to fully understand health policy, research, and treatment choices and to translate this knowledge into the promotion of healthy populations.

4. A nurse is investigating the role of the agent in the cause of an illness. Which of the following best describes what the nurse is examining? a. Host resilience b. Virus c. Infectiousness d. Insect bite

ANS: B An agent is described by its ability to cause disease and the nature and the severity of the disease. The four major categories of agents are: (1) bacteria, (2) parasites, (3) fungi, and (4) viruses. Host resilience is a host factor. Infectiousness is the measure of the potential ability of an infected host to transmit the infection to other hosts. Environmental factors facilitate the transmission of an infectious agent from an infected host to other susceptible hosts, such as an insect bite.

3. A nurse is working in an occupational health setting. Which of the following roles will the nurse most likely have? a. Administrator b. Clinicians/practitioners c. Consultant d. Educator

ANS: B An occupational health nurse could be employed in any of these roles. However, most occupational health nurses work as nurse clinicians/practitioners.

15. A nurse is volunteering on a disaster medical assistant team as a first responder. Which of the following tasks would the nurse be prepared to perform? a. Set up immunization clinics. b. Assist in triaging disaster victims. c. Provide all the medical care for disaster victims. d. Complete a needs assessment of the community.

ANS: B As a first responder arrives on the scene, the plans for triaging of disaster victims should begin immediately. Setting up immunization clinics would not be a priority following a disaster. In responding to a disaster, it may be impossible to provide all of the medical care for disaster victims. It would not be appropriate to start with completing a needs assessment of the community following a disaster; this should be done before the disaster occurs.

5. A public health nurse (PHN) discusses with a client which services are appropriate to meet her needs. Which of the following best describes the role that the nurse is using? a. Case manager b. Advocate c. Counselor d. Role model

ANS: B As an advocate, the PHN collects, monitors, and analyzes data and works with the client to identify and prioritize needed services, whether the client is an individual, a family, a community, or a population. A PHN discusses with a client which services are appropriate to meet her needs. In this case, the nurse's public health function is that of an advocate. Case managers assist clients in identifying services they need the most at the least cost. As a counselor, the nurse is a person for the client to talk with to discuss feelings and concerns. A nurse serves as a role model for the client by being a person for the client to look up to.

7. A registered nurse (RN) would like to become a nurse practitioner (NP). Which level of education is required? a. Bachelor of science in nursing degree b. Master's or higher degree in nursing and NP education c. Bachelor of science in nursing degree and completion of a formal NP program d. Master's degree with a specialty in public health nursing

ANS: B As of 1992, the American Nursing Credentialing Center (ANCC) required a master's degree or higher for NP certification. The practice doctorate is encouraged for NP certification but is not required.

18. A nurse believes that all Americans should receive basic health care services. Which of the following core functions supports this belief? a. Assessment b. Assurance c. Policy development d. Advocacy

ANS: B Assurance purports that all persons should receive essential personal health services. Assessment refers to systematically collecting data on the population, monitoring the population's health status, and making information available about the health of the community. Policy development refers to the need to provide leadership in developing policies that support the health of the population, including the use of the scientific knowledge base in making decisions about policy. Advocacy embodies an ethical focus grounded in quality of life.

7. Which factor may limit access to care for migrant farmworkers? a. Unwillingness to seek affordable health care b. Lack of knowledge regarding services c. Availability of 24-hour health care services d. Adequate transportation

ANS: B Because of isolation, migrant farmworkers lack usual sources of information regarding available services. Migrant workers typically are unable to afford health care, have limited services available, and may not have adequate transportation to get to health care services.

19. A nurse is educating migrant women to serve as links between the existing health care agencies and the migrant farmworker family. Which of the following best describes the role of these women? a. Migrant head start workers b. Outreach care workers c. Physician assistants d. Nurse aides

ANS: B Because these outreach workers are members of the migrant community, they are trusted and know the culture and the language. Physician assistants and nurse aides do not serve in this lay person role in the community.

5. Which factor influences the growing number of poor persons in the United States? a. Increase in company earnings. b. Changes in the labor force. c. Better access to quality education. d. Adequate welfare benefits.

ANS: B Changes in the labor force affect the growing number of poor persons in the United States. Decreased company earnings, decreased access to education and job skills, and inadequate welfare benefits also influence the growing number of poor persons.

2. Why is it important for a nurse to have knowledge of environmental health and the effects of chemical, biological, and radiological materials? a. They are a major cause of global warming. b. They are often found in the air, water, and products we use. c. They are frequently linked to the development of chronic illnesses. d. They are products that nurses work with daily.

ANS: B Chemical, biological, and radiological pollutants are often found in the air we breathe, the water we drink, and the products we use. These are not a major cause of global warming. They are not frequently linked to the development of chronic illnesses. These are not products that nurses work with daily.

7. When researching pesticides, the nurse looks at the "family" of the chemical. What similarities are found among chemicals that have been placed in the same family? a. Route of entry into the body b. Actions and associated risks c. Effects that they have on the body d. Potency and toxicity

ANS: B Chemicals are grouped so it's possible to understand the actions and risks associated with each group. Although some common health risks exist within these families of chemicals, the possible health risks for each chemical should be evaluated individually when a potential human exposure exists.

4. A nurse is coordinating care to ensure a comfortable and peaceful death. Which of the following factors should be an important consideration for the nurse? a. The unique stress dying patients often experience b. The cultural values, expectations, and preferences of the family c. The communication style of each family member d. The abilities of the nurse and health care team

ANS: B Cultures vary in their beliefs and responses to death. Nurses should know the differences in cultural responses so that they can effectively help people in their time of need. It will be important for the nurse to take cues from the client and the loved ones regarding their needs. The stress experienced by dying patients is different based on the client's culture.

4. A nurse is conducting disease surveillance. Which of the following describes the goal for this action? a. Eradicating a disease before it starts b. Establishing a baseline rate of disease occurrence c. Targeting populations for triage d. Reducing the incidence of heart disease in a community

ANS: B Disease surveillance helps establish baseline rates of disease occurrence and patterns of spread to make it possible to initiate a rapid response to an outbreak. Disease surveillance is unable to eradicate a disease before it starts, to target populations for triage, or to reduce the incidence of heart disease in a community. However, disease surveillance does make it possible to initiate a rapid response to an outbreak of a disease or an event that can cause a health problem.

16. A nurse was employed as a forensic nurse during the 1950s. Which of the following tasks would the nurse most likely have been implementing? a. Working in a sexual assault clinic b. Identifying indicators of assault c. Implementing protocols for sexual assault care d. Receiving sexual assault nurse examiners (SANE) training

ANS: B During the 1940s through the 1960s, nurses became aware of their role in identifying indicators of assault. The first nurse-run sexual assault clinic opened in 1974, with subsequent development of the first protocols for sexual assault care. SANE programs appeared in the 1990s.

12. A nurse is working in a community that is experiencing the honeymoon phase after a disaster. Which of the following is characteristic of this phase? a. First responders work tirelessly to save others. b. Survivors share their stories. c. Medical personnel experience exhaustion. d. Community organizations rebuild the community.

ANS: B During the honeymoon phase, survivors rejoice for their survival and may share their experiences and stories. First responders work tirelessly to save others during the heroic phase. Medical personnel experience exhaustion during the disillusionment phase. The community begins to rebuild during the reconstruction phase.

11. A nurse believes everyone is entitled to equal rights and equal treatment in society. Which of the following principles is being applied? a. Distributive or social justice b. Egalitarianism c. Libertarian view of justice d. Communitarianism

ANS: B Egalitarianism is defined as the view that everyone is entitled to equal rights and equal treatment in society. Distributive justice requires that the distribution of benefits and burdens on a society be fair or equal. The libertarian view of justice holds that the right to private property is the most important right. Communitarianism views that individual rights need to be balanced with social responsibilities; individuals do not live in isolation but are shaped by the values and culture of their communities.

4. What term is used to describe an orderly process that considers ethical principles, client values, and professional obligations? a. Moral distress b. Ethical decision making c. A value d. A code of ethics

ANS: B Ethical decision making is defined as an orderly process that considers ethical principles, client values, and professional obligations. Moral distress is an uncomfortable state of self when one is unable to act ethically. Values are beliefs about the worth or importance of what is right or esteemed. A code of ethics is a moral standard that delineates a profession's values, goals, and obligations.

3. A nurse who was working in the 1960s used the code of ethics to guide making an ethical decision. Which code of ethics would have been used? a. Nightingale Pledge b. Code for Professional Nurses c. Code of Ethics for Nurses with Interpretive Statements d. International Council of Nurses (ICN) Code of Ethics for Nurses

ANS: B Florence Nightingale lived in the 1800s. The Code for Professional Nurses was adopted in 1950, the Code of Ethics for Nurses with Interpretive Statements was adopted in 2001, and the ICN Code of Ethics for Nurses was adopted in 2000.

6. A forensic nurse is demonstrating the role of advocate. Which of the following best describes the intervention being performed by the nurse? a. Partner with public health professionals to implement programming. b. Promote programs that prevent injuries. c. Investigate injuries in the community. d. Provide holistic care to victims of violence.

ANS: B Forensic nurses can demonstrate the role of advocate by promoting programs that prevent injuries. Partnering with public health professionals to implement programming demonstrates collaboration. Investigating injuries in the community demonstrates the role of epidemiologist. Providing holistic care to victims of violence demonstrates the role of caregiver.

22. A forensic nurse plans to collaborate with another professional to provide care to the client. Which of the following professionals would the nurse be most likely to collaborate with? a. Social worker b. Corrections officer c. Clinic manager d. Dietician

ANS: B Forensic nurses collaborate with corrections officers and law enforcement, epidemiologists, emergency department providers, psychiatric practitioners, and public health nurses.

29. A nurse has provided an organization a philanthropic contribution. Which of the following best describes what the nurse has provided? a. Contract b. Gift c. Grant d. Loan

ANS: B Gifts are philanthropic contributions. Contracts are awarded for the performance of a specific task or service, usually to meet guidelines specified by the organization making the award. Grants are awards to nUonpSrofNit orTganizatOions to allow the recipient to implement activities of their own design that address the interests of the funding agency. A loan is money that needs to be paid back to the lender.

14. A public health nurse (PHN) understands that the emergence of new infectious diseases is influenced by the what? a. Increased availability of immunizations b. Globalization of food supplies c. Decreased use of child care facilities d. Creation of sanitation systems in third world countries

ANS: B Globalization of food supplies is one of the many factors that can influence the emergence of infectious diseases. The increased use of childcare facilities could contribute to the emergence of new infectious diseases. The lack of sanitation systems in third world countries could contribute to the emergence of new infectious diseases. The increased availability of immunizations should decrease the emergence of diseases.

10. A nurse is developing a goal for a client who is learning how to care for an ostomy. Which of the following would be most appropriate to develop? a. The client will look at his stoma without disgust each time his ostomy bag comes off. b. The client will be able to independently take care of his ostomy bag within 3 months. c. The client will gather all ostomy supplies correctly each time his ostomy bag needs to be changed. d. The client will successfully describe to the nurse how to care for his ostomy when he is asked.

ANS: B Goals are broad, long-term expected outcomes. The correct answer describes something that will happen over a long period of time. The other choices describe objectives.

5. A nurse is examining the route of HIV transmission for a newly diagnosed HIV client. Which of the following would most likely be discovered by the nurse? a. Having contact with an HIV-positive individual who is coughing b. An infant receiving breast milk from an HIV-positive mother c. Receiving a mosquito bite while in Africa d. Being near an HIV-positive individual who is sneezing

ANS: B HIV can be transmitted through breast milk. HIV is not transmitted by coughing, sneezing, or mosquito bites.

8. In comparison with HIV infection in adults, what is true concerning HIV infection in infants and children? a. They present with the same signs and symptoms. b. They have a shorter incubation period. c. They experience a longer survival period. d. They are diagnosed using the same tests.

ANS: B HIV infection in infants and children has a shorter incubation period. The physical signs and symptoms in children are different and include failure to thrive, unexplained persistent diarrhea, developmental delays, and bacterial infections such as tuberculosis (TB) and severe pneumonia. Detection is made through different tests in infants of infected mothers than from those who are over 18 months. They do not have a longer survival period.

17. A forensic nurse is implementing Hammer's definition of caring. Which of the following actions would the nurse most likely perform? a. Utilizing intuition when assessing a client b. Providing self-care and care for the client c. Assuring that the client's basic needs are met d. Assessing the biopsychosocial and spiritual characteristics of the client

ANS: B Hammer recognized that in forensic nursing practice the caring component must address caring for oneself as well as caring for clients. A sense of self-worth must be cultivated and nurtured to the degree that caring is conscientious, objective, and demanding of the scientific truth. Caring must also be curious and questioning; it should be directed toward colleagues; and finally, caring should be about firm and valued concepts that inform practice.

3. A nurse uses Healthy People 2020 as a guide when planning health education in the community. Which of the following actions would be taken by the nurse? a. Focus on avoiding cigarette smoking and using alcohol in moderation. b. Educate clients using primary and secondary levels of prevention. c. Use Bloom's taxonomy when planning educational objectives. d. Design health fairs aimed at individuals.

ANS: B Healthy People 2020 focuses on implementing health promotion in priority areas using primary and secondary prevention. Understanding the three learning domains is crucial in providing effective health care education. Health fairs targeted at specific populations can provide a venue for providing primary and secondary prevention.

3. What is immunity a characteristic of? a. An agent factor b. A host factor c. An environmental factor d. An epidemiologic triad

ANS: B Immunity refers to species-determined resistance to an infectious agent and is determined by the characteristics of the host. The agent is the infection that is causing the infection/disease. Environmental factors may influence the susceptibility of the host. The epidemiologic triad involves the interaction of the host, agent, and environment.

3. A provider and a client consider the possibilities of solving a problem using one of the solutions identified. Which of the following phases of the program planning process is being used? a. Conceptualizing b. Detailing c. Implementing d. Evaluating

ANS: B In the detailing phase, the provider details the costs, resources, and program activities needed to choose one of the solutions from the conceptualizing phase. In the conceptualizing stage, options for solving the problem are created and several solutions are considered. In the evaluating phase, each alternative is weighed to judge the costs, benefits, and acceptance of the idea to the client population, community, and providers. The implementing phase requires obtaining and managing the resources required to operationalize the program in a way that is consistent with the plan.

17. A nurse practitioner (NP) has recently been hired by a public health department. Which of the following best describes what the NP will most likely be doing in this role? a. Disease prevention activities b. Deciding on a treatment plan c. Identifying needs of populations at risk d. Identifying health needs of the community

ANS: B In the role of a clinician, the NP diagnoses actual or potential health problems; decides on treatment plans jointly with clients; intervenes to promote health, to protect against disease, to treat illness, to manage chronic disease, and to limit disability; and evaluates with the client and other primary care team members about how effective and comprehensive the nursing intervention may be in providing continuity of care. Disease prevention activities, identifying needs of populations at risk, and identifying health needs of the community are roles of an advanced public health nurse (APHN).

20. A nurse is working with an individual who has a low literacy level. Which of the following barriers to learning should the nurse anticipate? a. Asking for additional clarification of materials b. Requesting to read the information later c. Having a high level of anxiety d. Being overly dependent on others

ANS: B Individuals with a limited literacy may have a limited vocabulary and general knowledge and do not ask for clarification. They may have a low motivation to engage in learning or may drop subtle clues that they cannot read by stating they will look at information later or take it home. The level of anxiety may vary among these individuals; it is more common for these individuals to request to read it later.

14. A nurse wants to find more information about indoor air quality. Which of the following websites would be most helpful? a. The National Institutes of Health (NIH) b. The American Lung Association c. "Right to Know" NURSINGTB.COM d. The Occupational Safety and Health Administration (OSHA)

ANS: B Indoor air quality is a growing public health concern in office buildings, schools, and homes and is reflected in the alarming rise in asthma incidence in the United States, particularly among children. Sources of information about indoor air quality include the EPA and the American Lung Association. The NIH is part of the U.S. Department of Health and Human Services (USDHHS) and is responsible for health and biomedical research. "Right to Know" is part of a workplace safety legislation. OSHA regulates safety in factories and businesses.

9. A nurse has evaluated the learning needs of a community support group. Which of the following steps should the nurse take first when developing an educational program for them? a. Consider any potential barriers to learning. b. Establish goals and objectives for the program. c. Select appropriate materials for the program. d. Assess the dynamics of the group.

ANS: B Instructional objectives need to be evaluated before a teaching program is designed. The five steps of the educational process are: (1) identifying educational needs, (2) establishing educational goals and objectives, (3) selecting appropriate educational methods, (4) implementing the educational plan, and (5) evaluating the educational process.

8. For obtaining information about services needed to meet community needs, business leaders within a community were interviewed. Which of the following describes this method of obtaining needs assessment data? a. Community forum b. Key informant interview c. Survey of existing agencies d. Indicators approach

ANS: B Interviewing business leaders within a community is an example of key informant interviews. Community forum is defined as a community, group, organization, or open meeting. Survey of existing agencies provides estimates of client populations via services used at similar community agencies. The indicators approach uses existing data to determine problems.

2. The nurse practitioner (NP) movement began at the University of Colorado with nurses providing what type of care? a. Maternity b. Well-child c. Nursing home d. Diabetes

ANS: B It was determined that educating community-oriented nurses could decrease morbidity among medically deprived children. Nursing practice for pediatric NPs included the identification, assessment, and management of common acute and chronic health problems, with appropriate referral of more complex problems to physicians.

13. A nurse is working in a health department when a patient arrives who has been traveling to South America and has been diagnosed with malaria. Which of the following considerations should be made by the nurse? a. The Morbidity and Mortality Weekly Report should be consulted to investigate the rate of malaria in the United States. b. This is a disease that must be reported to the state health department. c. The nurse should take precautions to wear a mask and gown to avoid exposure. d. The patient is very ill and should be sent to the hospital immediately.

ANS: B Malaria is on the list of infectious diseases notifiable at the national level. The Morbidity and Mortality Weekly Report is published weekly with the rates of disease; however, this would not be the first consideration by the nurse. Malaria is not spread by direct contact, rather from a bite from an infected mosquito. Malaria begins with flu-like symptoms, or the client may have very few symptoms.

11. In which role are the most differences seen between the advanced public health nurse (APHN) and the nurse practitioner (NP)? a. Administrator b. Clinician c. Consultant d. Educator

ANS: B Many differences are seen between the APHN and the NP in the clinician role. APHNs are focused on the needs of the overall community, whereas NPs are focused on the care of individuals. APHNs and NPs both provide education within a nursing framework and as a professional nurse educator. Both may have administrative roles within the agency setting. Both may serve as consultants to help with problem solving for an individual, family, or community to improve health care delivery.

4. A nurse is working with a vulnerable group experiencing multiple risk factors. Which of the following best describes this group? a. Smokers who use chewing tobacco as well as cigarettes. b. Substance abusers who test positive for HIV. c. Persons with limited access to care because they live in a rural area. d. New mothers needing information about baby and child care.

ANS: B Multiple risk factors are present in substance abusers, including contracting HIV and hepatitis B virus. Homelessness is another risk factor. Vulnerable populations of concern to nurses are persons who are poor or homeless, have special needs, pregnant teens, migrant workers and immigrants, individuals with mental health problems, people who abuse addictive substances, persons who have been incarcerated, people with communicable diseases and those who are at risk, and persons who are HIV positive or have hepatitis B virus or STDs.

8. A nurse practitioner reports a case of gonorrhea to the local health department. Which type of surveillance system is being used? a. Active b. Passive c. Sentinel d. Special

ANS: B NURSINGTB.COM In the passive system, case reports are sent to local health departments by health care providers. In the active system, the public health nurse may begin a search for cases through contacts with local health providers and health care agencies. In the sentinel system, trends in commonly occurring diseases or key health indicators are monitored. Special systems are developed for collecting types of data and may be a combination of active, passive, and/or sentinel systems.

14. Which population is at the greatest risk for experiencing work-related accidents with subsequent injuries? a. Workers of child-bearing age b. Workers with less thanN1 yeRar oIf exGperBie.ncCe M c. Workers with diminished sensory abilities d. Workers with chronic illnesses

ANS: B New workers with less than 1 year of experience on the current job are at the greatest risk for experiencing work-related accidents with subsequent injuries. The highest percentages were in mining; agriculture, forestry, and fishing; construction; and wholesale and retail trade.

8. Which ethical principle requires "doing no harm?" a. Respect for autonomy b. Nonmaleficence c. Beneficence d. Distributive justice

ANS: B Nonmaleficence refers to doing no harm. Respect for autonomy requires that individuals be permitted to choose those actions and goals that fulfill their life plans unless those choices result in harm to another. Beneficence requires that we do good. Distributive justice requires that there be a fair distribution of the benefits and burdens in society based on the needs and contributions of its members.

14. A nurse is educating a client about reducing risk factors for the development of colorectal cancer. Which of the following lifestyle changes should the nurse recommend? a. Increase in consumption of dairy products b. Reduction in consumption of processed meats c. Decrease in exposure to the sun d. Increase in sedentary lifestyle

ANS: B Obesity, physical inactivity, smoking, heavy alcohol consumption, a diet high in red or processed meats, and insufficient intake of fruits and vegetables are risk factors for colorectal cancer.

13. What is the purpose of developing objectives for a program? a. Giving the speaker-specific topics to cover b. Providing direction for conducting the program c. Providing an enriching experience for participants d. Formulating a mission statement

ANS: B Objectives provide direction and provide the mechanism for evaluating specific activities in light of the total program. The formulation of a mission statement clarifies the values and overall purpose of the program and provides a framework for the goals and objectives that follow. The action steps will provide explicit actions to accomplish the objectives, such as giving the speaker-specific topics to cover. An enriching experience for the participants is accomplished through many things, more than the development of program objectives.

9. An occupational health nurse is assessing workplace hazards. Which of the following would be the best method for the nurse to use? a. Review incident reports. b. Walk through the worksite. c. Interview key employees. d. Read the Standard Industrial Classification (SIC) Code.

ANS: B One of the best methods an occupational health nurse can use in assessing workplace hazards is to walk through the worksite. Reviewing incident reports, interviewing key employees, and reading the SIC Code do not provide the nurse with as much information as walking through the worksite.

27. A leader controls group members through rewards and often keeps members in the dark about the goals and rationale behind prescribed actions. What type of leadership does this describe? a. Democratic b. Patriarchal c. Socialist d. Paternal

ANS: B Patriarchal is authoritative and permits one to control members through rewards and threats. Paternal leaders win respect and dependence through parent-like devotion. Democratic leadership is cooperatiterm-136ve in nature and promotes and supports members' involvement in all aspects of decision making and planning. Socialist leadership supports community ownership and involvement of the community.

4. To increase the immunization rates among children in the community, a nurse manager is considering holding immunization clinics on Saturdays one month before school begins. Which of the following describes what is happening? a. Formulating b. Planning c. Implementing d. Evaluating

ANS: B Planning is selecting and carrying out a series of actions to achieve stated goals. In the formulating phase, the problem is defined, and client need is assessed. The implementing phase requires obtaining and managing the resources required to operationalize the program in a way that is consistent with the plan. Evaluation is determining whether a service is needed and can be used, whether it is conducted as planned, and whether the service actually helps people in need.

12. A nurse is conducting a community assessment in a poor neighborhood. Which of the following is the nurse most likely to find? a. Fewer minority groups b. Increased exposure to environmental hazards c. More available health care services d. Lower rates of crime and substance abuse

ANS: B Poor neighborhoods are more likely to have exposure to environmental hazards. Lower socioeconomic neighborhoods have been linked with poorer general health status, chronic conditions, mortality, birth outcomes, disability, injury and violence, and other health indicators. They are more likely to have higher rates of crime and poorer access to health care services.

2. Which problem does a health department usually have the legal authority to investigate? a. Pandemics b. Unusual clusters of illness c. World trends of diseaseNURSINGTB.COM d. Cases of the common cold and pneumonia

ANS: B Powers of local government include surveillance of unusual clusters of illness. A health department does not have legal authority to investigate pandemics, world trends of disease, and cases of the common cold and pneumonia.

4. A nurse practitioner (NP) is employed in a primary health care setting. Which services are most likely provided in this setting? a. Primary health services and prison medicine services b. Public health and primary care services c. Family practice and geriatrics care d. Home health and school health services

ANS: B Primary health care includes public health and primary care services.

12. Which type of cancer is the second leading cause of cancer deaths in the United States? a. Gastrointestinal b. Prostate c. Skin d. Testicular

ANS: B Prostate is the most common non-skin cancer and second leading cause of cancer deaths in the United States. Testicular cancer is rare. Skin cancer and gastrointestinal cancer are not among the leading causes of cancer deaths.

18. A person recently returning from overseas is diagnosed with Q fever. Which of the following must occur? a. Reporting must occur on the federal level. b. The case must be reported to the state and local health departments. c. The patient must be isolated immediately. d. The patient's immunization record must be reviewed.

ANS: B Q fever is on the notifiable list and must be reported to the state and local health departments. Thus, it needs to be reported, but does not imply immediate isolation or an immunization review.

12. Which statement fits with those of the Liberal Democratic Theory? a. One should reject any idea that societies, states, or collectives of any form can be the bearers of rights or can owe duties. b. Inequalities result from birth, natural endowment, and historic circumstances. c. Everyone has a right to private property. d. Government should be limited.

ANS: B Rawls acknowledges that inequities are inevitable in society, but he tries to justify them by establishing a system in which everyone benefits, especially the least advantaged. This is an attempt to address the inequalities that result from birth, natural endowments, and historic circumstances. The other choices relate to libertarianism.

31. A nurse is planning to evaluate a group's progress toward a health goal. Which of the following components should be included in the evaluative process? a. Type of teaching strategy used b. Recognition of accomplishments in the group c. Conflict that occurred in the group d. The type of leadership in the group

ANS: B Recognition of accomplishments in the group and of the group is built into the evaluative process. The type of teaching strategies used, types of leadership, and conflict are not part of the evaluative process of the group.

25. Which is a requirement of the Superfund Amendment and Reauthorization Act (SARA)? a. Applying stricter rules to determine Superfund sites b. Sharing written disaster plans with key resources in the community c. Authorizing each industry to write their own disaster plan d. Evaluating the effectiveness of a written disaster plan

ANS: B SARA requires that written disaster plans be shared with key resources in the community, such as fire departments and emergency departments. SARA does not apply stricter rules, authorize each industry to write their own disaster plan, or evaluate the effectiveness of a written disaster plan.

23. A nurse is teaching a client diagnosed with gonorrhea how to prevent reinfection and further spread. Which of the following describes the action taken by the nurse? a. Primary prevention b. Secondary prevention c. Tertiary prevention d. Primary health care

ANS: B Secondary prevention focuses on early detection and prompt treatment of disease, injury, or disability. Primary prevention refers to those interventions aimed at preventing the occurrence of disease, injury, or disability. Tertiary prevention focuses on chronic care and rehabilitation. Primary health care refers to the first line of care provided to patients typically by a physician or other health care provider.

17. A nurse administers a rabies immunization post-exposure to an animal bite. Considering the interventions used with infectious disease, which of the following levels of prevention is being used? a. Primary prevention b. Secondary prevention c. Tertiary prevention d. Assessment

ANS: B Secondary prevention focuses on early detection and prompt treatment of disease, injury, or disability. Primary prevention seeks to reduce the incidence of disease by preventing occurrence. Tertiary prevention reduces complications through treatment and rehabilitation. Assessment refers to the systematic collection of data.

9. A nurse is educating a community women's group about the importance of monthly breast self-exam and annual mammograms. Which of the following best describes why this community health education is important? a. Increased screening will lead to a decreased incidence of breast cancer. b. Early detection of breast cancer typically leads to a better prognosis. c. The death rate of breast cancer continues to rise. d. The potential harm from screening needs to be discussed.

ANS: B Secondary prevention measures, including screening techniques, make a difference in the death rates for breast cancer. Early detection often means cure, whereas late detection typically results in an unfavorable prognosis. A potential harm from screening does not exist.

11. A nurse collects evidence from a victim following a sexual assault. Which of the following level of prevention is being used? a. Primary prevention b. Secondary prevention c. Tertiary prevention d. Assessment

ANS: B Secondary prevention occurs following the occurrence of injuries and crime. This includes direct care provided to victims and perpetrators as well as the collection of evidence. Primary prevention focuses on preventing violence and injuries before they occur. If disability, incarceration, or death occurs, tertiary prevention is applied in settings appropriate to address rehabilitation or identify factors that have put individuals at risk.

17. A nurse is examining social determinants of health. Which factor is the nurse looking at? a. Ethnicity b. Income c. Gender d. Marital status

ANS: B Social determinants of health are such factors as economic status, education, environmental factors, nutrition, stress, and prejudice that lead to resource constraints, poor health, and health risk. Ethnicity, gender, and marital status are not considered social determinants of health.

12. A community health nurse has recently become involved in surveillance. Which of the following describes the situation that the nurse is in? a. Educating clients about influenza immunizations b. Collecting information about occurrence of measles c. Evaluating the effectiveness of an HIV/AIDS prevention program d. Advocating for changes in the national disease reporting requirements

ANS: B Surveillance gathers the "who, when, where, and what"; these elements are then used to answer "why." Nurses are frequently involved in surveillance by collecting data, making diagnoses, investigating and reporting cases, and providing information to the public. Client education, program evaluation, and advocating for changes are not part of the surveillance process.

2. What is a benefit of systematic planning? a. Allowing nurses to oveNrsUeeRmSaIteNriGalsTpBre.sCenOteMd b. Ensuring that resources are used to address the needs of the community c. Linking local politicians with the community d. Educating student nurses about public health nursing

ANS: B Systematic planning ensures that available resources are used to address the actual needs of people in the community, and it focuses attention on what the organization and health provider are attempting to do for clients. Systematic planning does not allow nurses to be in charge, link politicians with the community, or educate student nurses about public health nursing.

18. A nurse is using telehealth technology in the home setting. Which of the following best describes the intervention that is being used by the nurse? a. Uses Web TV to teach clients about their health. b. Shares health information using electronic communications. c. Makes regular visits to clients to check the technology. d. Risks violation of the Health Insurance Portability and Accountability Act.

ANS: B Telehealth includes sharing health information between the client and clinicians using electronic communications. Telehealth may or may not include video technology for live interactions. Telehealth allows for monitoring health status and symptom recognition, providing education, increasing communication, and enabling clients to become active partners in their own care.

11. Testicular cancer is commonly found in which age group? a. 10 to 14 years b. 15 to 40 years c. 41 to 50 years d. 51 to 70 years

ANS: B Testicular cancer is commonly found in men 15 to 40 years old, with the peak incidence of age 33.

9. Which organization publishes the current scope and standards of home care practice? a. Centers for Medicare and Medicaid Services b. American Nurses Association (ANA) c. Robert Wood Johnson Foundation d. Visiting Nurse Association

ANS: B The ANA publishes scope and standards for Home Health Nursing and Hospice and Palliative Nursing. The Centers for Medicare and Medicaid Services control the regulations and reimbursement mechanisms for the majority of home care services. The Robert Wood Johnson Foundation provides grant funding for a variety of projects that support nursing. The Visiting Nurse Association is a home care agency.

15. An advanced public health nurse (APHN) is working as an administrator. Which of the following would the nurse most likely be doing? a. Engaging in consultation and problem solving with individuals, families, and the community b. Providing direct authority and supervision over the staff and client care c. Working with doctorate-prepared nurses on research projects d. Educating the public on current health care practices

ANS: B The APHN functioning in the role of administrator is often responsible for and has direct authority and supervision over the staff and client care. As a consultant, the APHN engages in consultation and problem solving with individuals, families, and the community. As a researcher, the APHN works with doctorate-prepared nurses on research projects. As an educator, the APHN educates the public on current health care practices.

2. Which part of the immunological system suffers the greatest damage resulting from HIV infection? a. Dendrite cells b. CD4+ T-lymphocytes c. Macrophages d. Monocytes

ANS: B The CD4+ lymphocytes are severely damaged in HIV infection. In 2008 the case definition for HIV infection was revised to include the HIV classification/staging system based on the number of CD4+ T-lymphocytes. The dendrite cells, macrophages, and monocytes are not as severely damaged as the CD4+ cells are.

11. Which concepts are used to evaluate client outcomes when using the Omaha System Problem Rating Scale for Outcomes? a. Problem, intervention, and outcome b. Knowledge, behavior, and status c. Knowledge, skill, and attitude d. Problem, category, and modifiers

ANS: B The Omaha System Problem Rating Scale for Outcomes uses the concepts of knowledge, behavior, and status.

20. A bioterrorism attack has occurred in the United States. Which program would be used to provide large quantities of NmedRicatIionGs to Bth.e CAmMerican public? a. Cities Readiness Initiative b. Strategic National Stockpile (SNS) c. Public Health Information Network (PHIN) d. Project BioShield

ANS: B The SNS has the capacity to provide large quantities of medicine and medical supplies to protect the American public in a public health emergency. The Cities Readiness Initiative aids cities in increasing their capacity to deliver medicines and medical supplies during a public health emergency. The PHIN helps ensure information access and sharing. Project BioShield is a program to develop and produce new drugs and vaccines against potential bioweapons.

6. A health educator is trying to change a client's attitudes about smoking. Which of the following domains would be used? a. Cognitive b. Affective c. Psychomotor d. Developmental

ANS: B The affective domain is used to attempt to influence what individuals, families, communities, and populations feel, think, and value. The cognitive domain includes memory, recognition, understanding, reasoning, and problem solving. The psychomotor domain includes the performance of skills that require some degree of neuromuscular coordination and emphasizes motor skills. Developmental domain is not one of the domains of learning.

2. Which agency delegates the authority of the local health department? a. The federal government b. The state c. Local authorities d. Regional governing boards

ANS: B The authority of the local health department is delegated by the state. The federal government, local authorities, and regional governing boards are not involved with delegating authority of the local health department.

8. A nurse is employed in an occupational health setting. Which of the following activities would be a primary role of the nurse? a. Caring for employees and their families b. Providing health promotion and emergency care c. Updating the Material Safety Data Sheets (MSDSs) d. Reporting communicable diseases

ANS: B The customary role of the occupational health nurse extends beyond emergency treatment and prevention of illness and injury and also includes the promotion and maintenance of health, overall risk management, care for the environment, and efforts to reduce health-related costs in business. The occupational health nurse does not typically care for families of employees, update MSDSs, or report communicable diseases.

12. A nurse is applying the steps of surveillance from the Minnesota Model of Public Health Interventions book. Which of the following describes the initial action that the nurse would take? a. Analyze data. b. Consider whether surveillance is appropriate to the situation. c. Evaluate the impact of the surveillance. d. Collect data.

ANS: B The first step in this model considers whether surveillance is appropriate. Analyzing data is the fifth step in this model. Evaluation of the impact of the surveillance is the last step of the model. Collecting data is the fourth step in this model.

17. A nurse is providing contraceptive counseling to a female. Which of the following goals does the nurse plan to work toward? a. Encourage the individual to choose abstinence. b. Ensure the individual is educated to make an informed choice about reproduction. c. Advocate for increased funding for reproductive services. d. Reduce the health risks of the individual.

ANS: B The goal of contraceptive counseling is to ensure that women have appropriate instruction to make informed choices about reproduction. TheOnurse should provide a nonjudgmental approach during counseling and allow the woman to choose the appropriate contraceptive method. Nurses do advocate for reproductive services for women, but that is not a goal of contraceptive counseling. Reduction in the health risks of the individual is a goal of preconceptual counseling.

1. What is the goal of local public health departments? a. Monitoring communicable diseases b. Improving the health status of communities c. Offering services for the uninsured d. Enforcing environmental codes and laws

ANS: B The goal of local public health departments is to safeguard the public's health and improve the health status of communities. The federal and state levels monitor communicable diseases, offer services for the uninsured, and enforce environmental codes and laws.

7. What is the time interval between invasion by an infectious agent and the first appearance of signs and symptoms of the disease called? a. Communicable period b. Incubation period c. Infectiousness d. Endemic

ANS: B The incubation period is the time interval between invasion by an infectious agent and the first appearance of signs and symptoms. The communicable period is the interval during which an infectious agent may be transferred directly or indirectly from an infected person to another person. Infectiousness is the measure of the potential ability of an infected host to transmit the infection to other hosts. Endemic refers to the constant presence of a disease within a geographic area or a population.

12. A nurse uses a key informant interview as part of completing a needs assessment. Which of the following describes the primary advantage of using this tool? a. Low cost. b. Provides a picture of services needed. c. Identifies the perspectives of many persons. d. Indicates the extent of services offered by existing programs.

ANS: B The main advantage of using a key informant as a needs assessment tool is that it would provide a picture of the services needed. The main advantage of focus groups is the low cost. The advantages of a community forum are the low cost and identification of the perspectives of large number of persons. The advantages of a survey of existing agencies are to know the extent of services offered in existing programs.

23. Which is most likely to be a major stressor for an advanced public health nurse (APHN)? a. Autonomy of practice b. Professional isolation c. Flexible work hours d. Collaboration

ANS: B The major stressor for an APHN is likely to be professional isolation, because they are likely to be hired in remote practice employment sites. Autonomy of practice in these sites attracts many NPs and APHNs. Long drives, long hours, lack of social and cultural activities, and lack of opportunity for professional development are often experienced.

1. Which statement about migrant and seasonal farmworkers is true? a. There are more than 10 million working in the United States. b. The majority are Mexican. c. Most are employed in the Northeastern states. d. Thirty-five percent are undocumented workers.

ANS: B The majority of migrant farmworkers are foreign-born (72%) and predominantly Mexican (67%). Estimates suggest there are approximately 1.0 to 1.8 million hired farmworkers. Five states account for 65% of all farmworkers: (1) California, (2) Florida, (3) Washington, (4) Oregon, and (5) North Carolina. Of the migrant and seasonal farmworkers, 52% have legal authorization to work in the United States.

21. A nurse is monitoring program activities, such as hours of service, number of providers used, number of referrals made, and amount of money spent to meet program objectives. Which of the following aspects of program evaluation is being used? a. Relevance b. Progress c. Efficiency d. Effectiveness

ANS: B The monitoring/tracking of program activities is an example of the progress aspect of program evaluation. Relevance is evaluating the need for the program. Efficiency is evaluating the relationship between program outcomes and the resources spent. Effectiveness is evaluation of the ability to meet program objectives and the results of program efforts.

2. A forensic nurse has collected trace evidence. Which of the following best describes what has been collected? a. Fingerprints b. Hairs c. Hand gun d. Autopsy report

ANS: B The most common types of trace evidence from victims of violence are hairs, fibers, and small pieces of material, such as fragments of metal, glass, paint, and wood.

6. A nurse is working with a family who has an income above the federal income guidelines but is unable to meet their living expenses. Which of the following terms best describes this family? a. Persistent poverty b. Near poor c. Impoverished d. Neighborhood poverty

ANS: B The near poor are persons and families whose income is above the federal income guidelines but insufficient to meet living expenses. Persistent poor refers to individuals and families who remain poor for long periods and who pass poverty on to their descendants. Neighborhood poverty refers to geographically defined areas of high poverty, characterized by run-down housing, high unemployment rates, and poorer health outcomes. Impoverished refers to those living in poverty.

26. An occupational health nurse is involved in disaster planning. Which of the following actions would be most appropriate for the nurse to take? a. Assess for possible disasters. b. Prevent injuries and death of workers c. Store Medical Data Sheets in a safe place. d. Collaborate with government authorities to plan disaster management.

ANS: B The occupational health nurse is a key player in occupational disasters. The goals of a disaster plan are to prevent or minimize injuries and deaths of workers and residents, minimize property damage, provide effective triage, and facilitate necessary business activities. A disaster plan requires the cooperation of different personnel within the company and community. There are not Medical Data Sheets rather MSDSs.

10. One case of smallpox occurs in a population in which it was previously eliminated. Which of the following best describes what has occurred? a. Endemic b. Epidemic c. Pandemic d. Infectivity

ANS: B The occurrence of one case of smallpox in a population in which it was previously eliminated is an epidemic. Endemic refers to the constant presence of a disease within a geographic area or population. Pandemic refers to an epidemic occurring worldwide and affecting large populations. Infectivity is the ability to enter and multiply in the host.

19. A nurse practitioner (NP) recently began employment in an emergency department. What would the primary role of the NP be in this setting? a. Triage clients. b. Manage non-emergent client problems. c. Assess medically uninsured clients. d. Provide health education about preventive health care.

ANS: B The primary role of the NP in emergency departments is usually to manage non-emergent uninsured clients. Provision oUf hSealthNedTucationOabout preventive care most likely would occur in an outpatient clinic, not in an emergency department. NPs can treat patients in this setting. Both medically insured and uninsured individuals will be seen in this practice setting.

21. A nurse is examining the future of disaster management. Which of the following should the nurse anticipate? a. Vague and unorganized future due to constant emergence of new disasters b. Increasing sophistication in technology and surveillance c. Lack of involvement by national and state officials d. Decreasing need for public health workers to provide care Public Health Nursing 10th Edition Stanhope Test Bank NURSINGTB.COM

ANS: B The terrorist event of September 11, 2001, and the anthrax cases have increased the awareness for the need to plan for disasters. Technology and surveillance will continue to advance. Disaster management continues to change and become more organized as learning occurs after each incident, producing progressive best practices. National and state officials are very involved in disaster management. PHNs are particularly critical members of the multidisciplinary disaster health team given their population-based nursing focus and specialty knowledge in epidemiology and community assessment skills.

22. A nurse is caring for a client who has been diagnosed with West Nile Virus. Which of the following types of illness does the client have? a. Foodborne b. Vector-borne c. Waterborne d. Zoonoses

ANS: B West Nile virus is carried by a mosquito, which is a vector. Foodborne illnesses are carried by food. Waterborne illnesses are transmitted through water. Zoonoses are infections that are transmitted from vertebrate animal to a human under natural conditions.

9. When would it be appropriate for a nurse to use a Geographic Information System (GIS)? a. Recording client data collected at a foot clinic b. Identifying areas with an increased incidence of lead poisoning c. Evaluating effectiveness of a farm safety program d. Scheduling health promotion programs in the community

ANS: B The use of a GIS allows the PHN to apply the principles of epidemiology into practice. GIS allows nurses to code data so that it is related spatially to a place on earth and is helpful in determining concentrated areas for incidence of disease and illness. An electronic health record (EHR) would be the most appropriate place to record client data from a foot clinic. Evaluation of effectiveness of a farm safety program would need to be done through some type of assessment or evaluation method—GIS would not be useful to collect this data. GIS is not a scheduling system and would not be helpful when scheduling health promotion programs in the community.

2. Why do acts of bioterrorism or natural disasters often have more casualties? a. They cause the most widespread destruction. b. Victims have little time to make evacuation preparations. c. Those with chronic conditions cannot escape in time. d. The early warning systems are not effective.

ANS: B The use of weapons of mass destruction or natural disasters, such as earthquakes, will often have more casualties because victims have little time to make evacuation preparations. The other answers are false.

6. The wide variations in health services and health status between certain population groups are identified by what term? a. Vulnerable population groups b. Health disparities c. Disadvantaged populations d. Risk markers

ANS: B The wide variations in health services and health status between certain population groups are called health disparities. Vulnerable populations are typically considered to be those who are at greater risk for poor health status and who have poor access to health care. Disadvantaged populations have fewer resources for promoting health and treating illness than does the average person in the United States. Risk describes that some people have a higher probability of illness than others.

20. A nurse refers to the Code of Ethics for Nurses or the Public Health Code of Ethics. Which of the following describes why the nurse has referred to this document? a. To provide answers for ethical dilemmas b. To guide professional practice related to ethics c. To increase moral leadership in ethics d. To find a framework for ethical decision making

ANS: B These codes provide general ethical principles and guide personnel in thinking about the underlying ethics of the profession. They do not provide answers for ethical dilemmas, only serve as a guide. They do not increase moral leadership, nor do they provide a framework for ethical decision making.

1. A nurse is utilizing the provisions created by the Ryan White HIV/AIDS Treatment Extension Act. Which of the following interventions is the nurse most likely performing? a. Increasing AIDS awareness in the community b. Determining available health care services for HIV-infected individuals c. Preventing the transmission of AIDS to children from their mothers d. Allowing persons in the final stages of HIV to die with dignity

ANS: B This act provides emergency services, services for early intervention and care, and drug reimbursement programs for HIV-infected individuals.

19. A nurse is planning to assist homeless clients at a local shelter with finding community resources to meet their health care needs. Which of the following actions should the nurse take before interacting with this population? a. Change personal views about homelessness. b. Understand the concepts of homelessness. c. Receive special training about where to locate homeless people. d. Have experience in mental health nursing and counseling.

ANS: B To implement effective nursing interventions for homeless clients, nurses need to be aware of their own beliefs and values about the homeless as well as understand the concepts of homelessness.

8. Which statement is true about health care services for migrant workers? a. Clinics are usually found within walking distance from work. b. Undocumented workers fear deportation when using clinics. c. Health care records are kept by workers to allow for continuation of services. d. Clinics are most often open on weekends and keep evening hours.

ANS: B Unauthorized individuals fear that securing services in a federally funded or state-funded clinic may lead to discovery and deportation. Clinics are usually a great distance from work and offer health services only during work hours. Health records do not typically follow the client.

2. A nurse is trying to provide effective nursing care to poor persons, families, and populations in a variety of settings. Which of the following actions should be taken by the nurse? a. Move to the area where people are living to understand their plight. b. Understand the concept of poverty on many levels. c. Take a trip abroad to seNe UhoRwSpIoNveGrtTy Bex.isCtsOinMother countries. d. Become fully immersed in the concept of poverty for at least a year.

ANS: B Understanding the concept of poverty with historical, social, political, economic, biological, psychological, and spiritual dimensions is the first step in providing effective care to this population. It is not necessary to move to the area, take a trip abroad, or become immersed in the concept of poverty to provide effective nursing care to this population.

22. A nurse is educating intravenous (IV) drug users about sharing equipment. Which of the following information would be appropriate for the nurse to include in this education? a. Tell the clients to throw away their equipment after one use. b. Educate the clients on using full-strength bleach on their drug paraphernalia for 30 seconds. c. Suggest limiting the number of people who share the equipment. d. Provide clean needles and syringes to whoever wants them.

ANS: B Using bleach on the needles and syringes is a way to decrease cross-contamination. This is the last-resort option. People who inject drugs are difficult to reach for health care services, so providing them education is important so that they can protect themselves and others as they most likely will not throw away their equipment or be selective about whom they share their equipment with. Providing needles and syringes does not provide the appropriate education to prevent the spread of disease.

9. A nurse is discussing the role of a vector in the spread of disease. Which of the following is the nurse most likely referring to? a. Contaminated water b. A tick c. A dirty needle d. An infected person

ANS: B Vectors are arthropods such as ticks and mosquitoes that transmit the infectious agent by biting or depositing the infective material near the host. Contaminated water, a dirty needle, and an infected person would be common vehicles.

5. A public health nurse is examining several issues within daily practice. Which of the following issues would be considered an ethical dilemma? a. Whether or not to establish a community health center in a rural area b. Allocating resources in a natural disaster c. Deciding to withdraw care on a hospice patient d. Applying the principles of Florence Nightingale in Bangladesh

ANS: B When resources are scarce, a dilemma may exist as to how to allocate them. Considering establishing a community health center may be a dilemma, but it probably does not involve ethics. Withdrawing care from a hospice patient would most likely not be an issue encountered by a public health nurse, as this represents community health nursing practice, not public health nursing practice. Applying the principles of Florence Nightingale would not be ethical issue.

2. Which are categories of classifying interventions according to the Omaha System Intervention Scheme? (Select all that apply.) a. Health education b. Case management d. Direct care services e. Skilled nursing care

ANS: B, C The Omaha System Intervention Scheme is comprised of four broad categories of interventions: (1) teaching, guidance, and counseling; (2) treatments and procedures; (3) case management; and (4) surveillance.

2. Which principles are included in the Scope and Standards of Public Health Nursing Practice? (Select all that apply.) a. The client or "unit of care" is the population. b. Primary prevention is given priority. c. All processes must include partnering with representatives of the people. d. Government agencies mandate requirements for public health. e. The focus of many interventions should be on secondary prevention. Public Health Nursing 10th Edition Stanhope Test Bank

ANS: B, C The eight tenets of public health nursing distinguish public health nursing from other nursing specialties. Primary prevention is given priority and all processes must include partnering with representatives of the people as part of these established principles.

2. A nurse is assessing for environmental health risks in the community. Which of the following approaches would be most appropriate for the nurse to use? (Select all that apply.) a. Asking legislators to provide a list of environmental pollutants in the area b. Developing a list of exposures associated with urban, rural, or suburban settings c. Assessing the risk by medium such as air, water, soil, or food d. Dividing the environment into functional locations: home, school, workplace, and community e. Recruiting community members to sign a petition to decrease pollution

ANS: B, C, D Developing a list of exposures associated with urban, rural, or suburban settings, assessing the risk by medium such as air, water, soil, or food, and dividing the environment into functional locations: home, school, workplace, and community are ways a nurse can assess the environment. Asking legislators for a list of pollutants or recruiting community members to sign a petition are not effective means to assess for environmental health risks.

2. A community health nurse is applying the Ethical Principles for Effective Advocacy. Which of the following best describes the action(s) of the nurse? (Select all that apply.) a. Act in the health care provider's best interest. b. Keep the client (group, community) properly informed. c. Maintain client confidentiality. d. Carry out instructions with diligence and competence. e. Provide advice to all members of the community.

ANS: B, C, D Keep the client (group, community) properly informed, maintain client confidentiality, and carry out instructions with diligence and competence are ethical principles for effective advocacy. The nurse should act impartially and offer frank, independent advice, which does not necessarily mean that advice should be provided to all members of the community.

3. Which are the objectives of a disease investigation? (Select all that apply.) a. Enhancing the knowledge of the nurse working in public health b. Controlling and preventing disease or death c. Identifying factors that contribute to the disease outbreak/event occurrence d. Implementing measures to prevent occurrences e. Participating in the core functions of public health

ANS: B, C, D The objectives of an investigation are to control and prevent disease or death, identify factors that contribute to the disease outbreak/event occurrence, and implement measures to prevent occurrences. Enhancing the knowledge of the nurse working in public health and participating in the core functions of public health are not objectives of an investigation.

1. Why is it difficult to know the exact number of homeless persons? (Select all that apply.) a. Homeless people refuse to provide demographic information. b. Many homeless people refuse to be interviewed. c. It is difficult to generalize from one location to another. d. Some persons experience short intervals of homelessness. e. Homeless individuals are most likely hiding within the community.

ANS: B, C, D Those who are homeless are hard to find since they may sleep in boxcars, on building roofs, in doorways, or under freeways or pedestrian overpasses; temporarily stay with friends or family; refuse to be interviewed or deliberately hide the fact that they are homeless; have only short or intermittent episodes of being without a home; or may not fit in a general category.

3. Which is considered a forensic environment? (Select all that apply.) a. Crime scene b. Courtroom c. Mental health nursing unit d. Prison e. Emergency department

ANS: B, D Forensic environments include courtrooms, jails, prisons, and psychiatric facilities for the criminally insane.

18. A nurse is providing preconceptual counseling to a woman. Which of the following supplements would be recommended to help prevent neural tube defects? a. Iron b. Calcium c. Folic acid d. Vitamin C

ANS: C 400 μg of folic acid are recommended daily. Supplements of iron and Vitamin C are not recommended during preconceptual counseling. Supplements of calcium may be recommended to prevent bone loss but not for preconceptual counseling.

3. A nurse responding to a natural disaster is most likely responding to what concern? a. Transportation accident b. Pollution c. Communicable disease epidemic d. Fire

ANS: C A communicable disease epidemic would be considered a natural disaster. Transportations accidents, pollution, and fire are considered human-made disasters.

15. A person diagnosed with syphilis presents with signs and symptoms of rash, sore throat, and muscle and joint pain. Which of the following stages of syphilis is the client most likely experiencing? a. Congenital b. Primary c. Secondary d. Tertiary

ANS: C A person with signs and symptoms of a rash, sore throat, and muscle/joint pain is experiencing the secondary stage of syphilis. Primary syphilis occurs when the bacteria produce infection in the form of a chance at the site of entry. Tertiary syphilis usually occurs several years after initial infection and is rare in the United States because the disease is usually cured in its early stages with antibiotics. Congenital syphilis is transmitted transplacentally.

20. A college student goes to the Student Health Center with an extremely swollen neck where the advanced practice nurse diagnoses the mumps. The student reports that a roommate also is experiencing malaise and a sore throat. Which of the following defines these two students? a. Mixed outbreak b. Common source c. Point source d. Propagated outbreak

ANS: C A point source outbreak involves all persons exposed becoming ill at the same time, during one incubation period. A mixed outbreak is a common source followed by secondary exposures related to person-to-person contact. A common source outbreak refers to a group exposed to a common noxious influence such as the release of noxious gasses. A propagated outbreak does not have a common source and spreads gradually from person to person over more than one incubation period.

14. A population needs assessment has been completed. Which of the following describes what was done? a. A health status report indicates many people are not using the health department because of long waits. b. The World Health Organization (WHO) encourages all travelers to be up to date on the immunizations when traveling abroad. c. A nurse assesses the health status of older adults with diabetes prior to initiation of a health education program. d. An agency uses a television advertisement to encourage healthy eating by the citizens in the community.

ANS: C A population needs assessment focuses on the characteristics of a specific population, its health needs, and the resources available to address those needs. A nurse assessing the health status of older adults with diabetes is an example of a population needs assessment. A health status report indicating that many people are not using the health department because of long waits does not focus on a specific population. The WHO encouraging all travelers to be up to date on the immunizations when traveling abroad does not include the resources available to address those needs. An agency using a television advertisement to encourage healthy eating by the citizens in the community does not focus on assessment of a specific population.

27. A nurse needs to collect baseline data before implementing which type of a study? a. Cost-accounting study b. Cost-benefit study c. Cost-effectiveness study d. Cost-efficiency study

ANS: C A study designed to measure the quality of a program as it relates to cost is a cost-effectiveness study. A cost-efficiency study is designed to examine the actual cost of performing program services and to focus on productivity versus cost. A study used to assess the desirability of a program by examining costs and benefits is a cost-benefit study. Cost-accounting studies are performed to find the actual cost of a program.

14. A nurse is using the principles of virtue ethics decision making. Which of the following describes the action that the nurse would take? a. Provide efficient and effective nursing care. b. Identify the meaningful facts in the situation. c. Seek ethical community support to enhance character development. d. Plan ways to restructure the social practices that oppress women.

ANS: C According to Aristotle, virtues are acquired and include interest in the concept of the good, including benevolence, compassion, trustworthiness, and integrity. One part of the process is seeking ethical community support to enhance character development. Nurses can demonstrate advocacy when providing efficient and effective nursing care. Identifying the meaningful facts in the situation is part of the ethical decision-making process. Planning ways to restructure the social practices that oppress women is part of the feminist ethics decision-making process.

16. A home care agency has applied for accreditation from the Joint Commission. What is the next step the agency will take? a. Attend a conference to learn more about the accreditation process. b. Schedule a site visit with the Joint Commission. c. Complete a self-study of the agency. d. Improve methods of documentation of client visits.

ANS: C After applying for accreditation, a lengthy self-study must be completed that addresses all aspects of the agency's operation. Following completion of the self-study, an accreditation team schedules a site visit. Attending a conference to learn more about accreditation should be done before applying. A site visit with the Joint Commission occurs after the self-study has been reviewed. Improving methods of documentation of client visits should be addressed before applying for accreditation.

13. A large amount of data related to hypertension rates in a community is collected and analyzed. What is the next step in the surveillance process? a. Collecting data from multiple valid sources b. Evaluating the impact on the surveillance system c. Interpreting the data and disseminating it to decision makers d. Asking political officials to finance a hypertension clinic

ANS: C After data is collected and analyzed, the findings must be disseminated. Collection of data is the step described in this scenario and the question asks what must be done next. Evaluating the impact on the surveillance system occurs after dissemination. Dissemination may occur to a broader audience than only political officials.

6. What effect does alcohol consumption have on migrant farmworkers? a. Provides relaxation and is important for migrant workers' well-being. b. Adds additional risks to pesticide exposure. c. Poses safety hazards for farmworkers. d. Allows migrant workers to sleep better, thus improving daytime productivity.

ANS: C Alcohol can contribute to farm-related injuries. Drug and alcohol use have been identified as a significant source of stress. Alcohol can also contribute to health problems, greater risk for human immunodeficiency virus (HIV), violence in camps/home sites, domestic violence, and decreased funds for personal and family needs.

13. A nurse is assessing persons arriving at an alternate care center following a disaster. Which of the following actions should the nurse take first? a. Assess the amount of equipment and medications needed for each client. b. Determine if the client has a psychological condition requiring special attention. c. Assess whether this type of facility is appropriate for the client. d. Determine if the client has a support system to assist with additional care needs.

ANS: C Alternate care centers may be used to shelter patients with medical needs designated as "non-ambulatory care/hospital overflow," for example, care of non-ambulatory patients with less intense medical needs. After determination of the appropriateness of the facility, the nurse can then provide medical care as needed. It should not be necessary to initially determine if the client has a psychological condition or calculate the amount of equipment or medications the client will need.

20. An occupational health nurse refers an employee to an employee assistance program. Which of the following problems would most likely be addressed? a. Obesity b. Smoking c. Alcohol abuse d. Lack of exercise

ANS: C Employee assistance programs are designed to address personal problems such as marital/family issues, substance abuse, or financial difficulties. These issues are addressed because they affect the employee's productivity. These efforts are cost-effective for businesses.

3. An advanced practice nurse (APN) uses knowledge and physical, psychosocial, and environmental assessment skills to respond to common health and illness problems. Which of the following best describes this individual? a. Clinical nurse leader b. Advanced public health nurse (APHN) c. Nurse practitioner (NP) d. Clinical nurse specialist

ANS: C An NP applies nursing knowledge with physical, psychosocial, and environmental assessment skills. An advanced practice nurse is a licensed professional nurse prepared at the master's level or doctoral level to take leadership roles in applying the nursing process and public health sciences to achieve specific health outcomes for the community. An APHN has a minimum of a master's degree and has specialized courses related to public health. Clinical nurse specialists influence care outcomes by providing expert consultation for nursing staffs and by implementing improvements in health care delivery systems. Clinical nurse leader is a master's degree-prepared generalist who functions at the institution's microsystem level and assumes accountability for health care outcomes for a specific group of clients within a unit or area.

2. Medicare-certified home health agencies place emphasis on what type of care? a. Chronic b. Distributive c. Intermittent d. Primary

ANS: C As part of the Conditions of Participation, Medicare-certified home health agencies must place an emphasis on intermittent care. Home care services that are provided must be intermittent and provide a skilled service. Primary care refers to the care that is often provided in a physician's office. Medicare-certified agencies provide care following an acute hospitalization or medical change; they do not provide long-term care for chronic illnesses.

3. A nurse fulfills the environmental health competency of "assessment and referral." Which of the following actions has the nurse most likely completed? a. Advocating for public policy changes b. Understanding policy framework and major pieces of legislation c. Completing an environmental health history d. Describing the scientific principles about environmental health

ANS: C Assessment and referral states that all nurses should be able to successfully complete an environmental health history, recognize the potential environmental hazards and sentinel illnesses, and make appropriate referrals for conditions with probable environmental causes. Advocating for public policy changes is part of advocacy. Understanding policy framework and legislation is part of legislation and regulation. Describing the scientific principles of environmental health is part of basic knowledge and concepts.

17. Public health administrators provide part-time translators to a health department serving indigent immigrants. Which of the following best describes what is being addressed? a. Policy development b. Quality c. Assurance d. Libertarian philosophy

ANS: C Assurance refers to the role of public health in making sure that essential community-oriented health services are available, which may include providing essential personal health services for those who would otherwise not receive them. Policy development refers to the need to provide leadership in developing policies that support the health of the population, including the use of the scientific knowledge base in making decisions about policy. Quality refers to providing the best care possible. Libertarian philosophy refers to the view that the right to private property is the most important right.

8. An advanced public health nurse(AIPN HGNT) is completing the process for certification renewal in the specialty. What time period has passed making renewal necessary? a. 1 year b. 3 years c. 5 years d. 7 years

ANS: C Certification for APHNs and NPs is for 5 years.

23. A nurse has been working on a regular basis with a group that demonstrates cohesion. Which characteristic is the nurse most likely to describe? a. Inclusion of several leaders. b. Demonstrates diverse attitudes and values. c. All members share a common goal. d. The group has an efficient number of members.

ANS: C Cohesion is the attraction between individual members and between each member and the group. Group effectiveness improves as members work together toward group goals while still satisfying the needs of individual members. A group with several leaders may have difficulty obtaining cohesion if each of the leaders has a different idea. Common attitudes and values among the group members will be important for the group to obtain cohesion. Efficiency does not play a role in developing cohesion.

17. An occupational health nurse has conducted a walk-through assessment and has identified potential hazards in the workplace. The nurse recognizes that it will be easiest to modify exposure to which hazard? a. Bacteria b. Aerosols c. Noise d. Burnout

ANS: C Controlling physical agents, such as noise, can usually be accomplished through engineering strategies and personal protective equipment. It is much harder to change biological agents (bacteria), chemical agents (aerosols), and psychosocial agents (burnout).

11. What does each state do with the information that it receives about notifiable diseases? a. Utilizes the information for surveillance purposes. b. Reports the information to the local branch of the World Health Organization (WHO). c. Transmits the data electronically, weekly, to the Centers for Disease Control and Prevention (CDC). d. Stores the information for state use only.

ANS: C Data is transmitted weekly to the CDC through the National Electronic Telecommunications System for Surveillance. States need to transmit the information to the CDC rather than only keeping its own records. The reports are sent to the CDC, not to the WHO.

3. A nurse is caring for a client who has designated someone else to make health care decisions when they are unable to do so. Which of the following is being used by the client? a. An advanced medical directive b. A living will c. A durable medical power of attorney d. The Patient Self-Determination Act

ANS: C Durable medical power of attorney is the legal way for a client to designate someone else to make healthcare decisions when he or she is unable to do so. An advanced medical directive includes both a living will and a durable medical power of attorney. A living will allows the client to express the wishes regarding the use of medical treatments in the event of a terminal illness. The Patient Safe Determination Act requires those providers receiving Medicare and Medicaid funds to give clients written information regarding their legal options for treatment choices if they become incapacitated.

15. A person is already homeless and is receiving shelter. Which type of housing is being used? a. Low income b. Supportive c. Emergency d. Adequate

ANS: C Emergency housing is a shelter for persons who are already homeless. These shelters typically are only open at night and in many cities only on extremely cold nights.

6. The nurse teaches food handlers to wash utensils after contact with raw meat. Which of the following best describes the focus of this education? a. Agent b. Host c. Environment d. Food handler

ANS: C Environmental factors facilitate the transmission of an infectious agent from an infected host to other susceptible hosts. Teaching food handlers to wash utensils after contact with raw meat is a prevention that focuses on the environment. An agent is described by its ability to cause disease and the nature and the severity of the disease. The four major categories of agents are: (1) bacteria, (2) parasites, (3) fungi, and (4) viruses. A food handler is an example of a host, which is a human or animal that can harbor an infectious agent.

6. What statement accurately describes the characteristics of epidemiology? a. It is a science that studies the poisonous effects of chemicals. b. It explains the association between learning disabilities and exposure to lead-based paint at the cellular level. c. It helps nurses understand the strength of the association between exposure and health effects. d. It is a method for tracking the prevalence of health outcomes.

ANS: C Epidemiology studies the incidence and prevalence of disease, helping nurses understand the strength of the association between exposure and health effects. Toxicology is the science that studies the poisonous effects of chemicals. Environmental surveillance provides data with which to track and analyze the incidence and prevalence of health outcomes. Epidemiology does not examine causes at the cellular level.

20. According to the Centers for Disease Control and Prevention's (CDC's) format for evaluation, what would occur during the focus of the evaluation design step? a. Writing the program description b. Specifying the indicators that will be used c. Describing the purpose for the evaluation d. Linking the conclusions to the evidence gathered

ANS: C Focusing the evaluation design step involves describing the purpose for the evaluation, the users who will receive the report, how it will be used, the questions and methods to be used, and any necessary agreements. Writing the program description occurs when describing the program. Specifying the indicators that will be used is part of gathering credible evidence. Linking the conclusions to the evidence gathered is part of justifying the conclusions.

6. A nurse completes a program evaluation to identify if the objectives of the program were met. Which of the following describes the action of the nurse? a. Assessment b. Evaluation c. Formative evaluation d. Summative evaluation

ANS: C Formative or process evaluation is used to assess whether objectives are met, or planned activities are completed. Assessment is completed to provide a clear understanding of the overall health status of a community, to identify populations at risk, and to document health needs. Evaluation is determining whether a service is needed and can be used, whether it is conducted as planned, and whether the service actually helps people in need. Summative or impact evaluation is used to assess program outcomes or as a follow-up to the results of the program activities.

14. A nurse is implementing an educational program about the importance of being physically active. Which of the following steps would the nurse complete first? a. Provide learning guidance. b. Present the stimulus. c. Gain the learner's attention. d. Ask learners to recall prior learning.

ANS: C Gaining the learner's attention must happen first before learning can take place. The following steps of implementation include telling the learners the objectives of the instruction, asking learners to recall previous knowledge, presenting the essential materials, helping the learners apply the information, encouraging learning to demonstrate what they have learned, and providing feedback to help learners improve their knowledge and skills.

15. A nurse assesses the factors that may place a client at risk for injury from a violent act by considering the role these factors play before, during, and after the violent act. The work of which theorist is being applied? a. Matrix b. Maslow c. Haddon d. Hammer

ANS: C Haddon developed a matrix when examining methods to identify the risk of injury. The matrix looks at factors related to personal attributes, vector or agent attributes, and environmental attributes before, during, and after an injury or death. Hammer recognized that in forensic nursing practice, the caring component must address caring for oneself as well as caring for clients. Maslow's theory focuses on clients' meeting their hierarchy of needs.

8. Which of the following characteristics indicates a man is at a higher risk for developing prostate cancer? a. Being of Caucasian descent. b. Has not had a prostate-specific antigen (PSA) test. c. Has a father or brother who has had prostate cancer. d. Has benign prostatic hypertrophy (BPH).

ANS: C Having a father or brother who has had prostate cancer places a man at higher risk for developing prostate cancer. Being of African American descent places a man at higher risk for developing prostate cancer. The PSA test is not accurate in terms of sensitivity or specificity, but it is used as a screening test. BPH can cause an elevated PSA test result.

5. A hospital is using surveillance. Which of the following describes the rationale for this action? a. To protect the public against isolated patients b. To eliminate pathogens from the environment c. To improve quality of care and outcomes d. To decrease the incidence of ventilator-acquired pneumonia

ANS: C Hospital surveillance is used to improve quality of care and outcomes. Reduction of the incidence of ventilator-acquired pneumonia is one reason why a hospital uses surveillance, but it does not address the entire issue. Hospital surveillance is unable to eliminate pathogens from the environment and cannot protect the public against isolated patients.

12. A nurse is providing direct client care while working in a correctional setting. Which of the following levels of prevention is being used? a. Primary prevention b. Secondary prevention c. Tertiary prevention d. Assessment

ANS: C If disability, incarceration, or death occurs, tertiary prevention is applied in settings appropriate to address rehabilitation or identify factors that have put individuals at risk. Primary prevention focuses on preventing violence and injuries before they occur. Secondary prevention occurs following the occurrence of injuries and crime. This includes direct care provided to victims and perpetrators as well as the collection of evidence.

10. Which is a characteristic of persons living in poverty? a. Longer life expectancy b. Simple health problems c. Higher rates of chronic illness d. Fewer hospitalizations

ANS: C Not only do persons living in poverty have higher rates of chronic illness, but less access to care. They have a shorter life expectancy, more complex health problems, and hospitalization rates greater than those for persons with higher incomes.

20. Which of the following statements about health disparities is accurate? a. African American women have a much higher incidence of breast cancer. b. Health disparities are the result of negligent health care practices. c. Health disparities vary among racial/ethnic groups. d. Men have more obstacles to receiving healthcare than women.

ANS: C Improvements have been made in some areas of health disparities among various populations. However, health disparities vary among each racial/ethnic group. African American women have much higher death rates from breast cancer, not necessarily a higher incidence. The awareness of health disparities remains low among the general public. These health disparities are typically not the result of negligent health care practices.

22. A nurse practitioner (NP) is investigating regulations for third-party reimbursement. Which information will the NP most likely discover? a. NPs are reimbursed when directly supervised by a physician. b. NPs only receive reimbursement when working in rural areas. c. NPs are reimbursed at 85% of the physician rate. d. NPs are unable to be reimbursed by Medicare and Medicaid.

ANS: C In 1997, with the passage of the national reconciliation spending bill, NPs can be directly reimbursed, regardless of geographic setting, at 85% of what a physician would have been paid (if the service is covered under Medicare Part B). NPs can have an independent practice and do not need direct supervision by a physician. NPs can receive reimbursement regardless of practice setting. NPs can be reimbursed by a variety of payer sources.

2. Which population would have been the focus of care for an occupational health nurse in the early 1900s? a. Injured workers b. All workers c. Families d. The community

ANS: C In the early days of occupational health nursing, the nurse's work was holistic and centered on the family. The care provided by Betty Moulder and Ada Mayo Stewart focused on both ill workers and their families.

16. Which statement regarding the effects of homelessness on health is accurate? a. Peripheral vascular disease and hypertension are lessened by this lifestyle. b. The incidence and virulence of infections are decreasing. c. Trauma is a significant cause of death and disability. d. Crowded living conditions result in decreased risk for exposure to infections.

ANS: C Intended and unintended injuries such as traumas are a significant cause of morbidity and mortality in the homeless. Those who are homeless are at greater risk for peripheral vascular disease and hypertension. Because of their poor living conditions, a minor skin injury or infections left untreated can result in widespread infection.

16. Which statement about chemical agents is accurate? a. Chemicals are not ordinarily found in the body tissues of the general population. b. Most chemicals have been studied to determine the effects of exposure on humans. c. Chronic exposure to low-level doses of workplace chemicals below standards constitutes a potential health risk. d. Human effects of chemical exposure are associated with single agents rather than with the interaction of agents.

ANS: C It is true that chronic exposure to low-level doses of workplace chemicals below standards constitutes a potential health risk. Most chemicals have not been studied epidemiologically to determine the effects of exposure on humans. As a consequence of general environmental contamination with chemicals from work, home, and community activities, a variety of chemicals have been found in the body tissues of the general population. Predicting human responses to chemical exposures are complicated because multiple chemicals often combine and interact to create a new chemical agent.

21. What is the most common vector-borne disease in the United States? a. Malaria b. Yellow fever c. Lyme disease d. Rocky Mountain spotted fever

ANS: C Lyme disease is the most common vector-borne disease in the United States, with over 30,000 confirmed cases and probable cases reported to CDC in 2012. Yellow fever and Rocky Mountain spotted fever are both vector-borne diseases but are not the most common. Malaria is most prevalent vector-borne disease worldwide.

29. A nurse is working with a group focused on improving the health of the community. Which of the following interventions should be used by the nurse? a. Making decisions for the group to move the process along b. Inviting business leaders to participate in the group process c. Maintaining members through recognition and encouragement d. Teaching topics that are of national importance

ANS: C Maintaining members is an important intervention to facilitate group process. The topics that are taught should be of importance to the group. The group should be part of the decision-making process. The membership of the group should be maintained, inviting new members to the group may hinder the processes of the group.

11. Which types of industries are noted for high degrees of hazards associated with the work? a. Data entry, animal rescue, and hospice b. Engineering, science, and laboratories c. Manufacturing, mining, and agriculture d. Aeronautics, plastics, and nursing

ANS: C Manufacturing, mines, construction, and agriculture are noted for their high degree of hazards associated with their work. However, no worksite is free of occupational health and safety hazards.

19. A nurse may need to use medical coercion with a client. Which of the following medical diagnoses would the client most likely have? a. Human immunodeficiency virus (HIV) b. Depression c. Schizophrenia d. Hypertension

ANS: C Medical coercion may be necessary in certain situations. These situations include incapacity to make decisions and making decisions that could result in harm to self or harm to others in the absence of interventions. For instance, mentally ill inmates may be medically coerced to take psychotropic medications against their will if the mental disorder is serious, the inmate is dangerous to himself, and the medication prescribed is in the inmate's best medical interest. Schizophrenia is the diagnosis most likely to require medical coercion.

1. Historically, which of the following population groups would have been least likely to receive community assistance? a. Widowed women b. Orphaned children c. Mentally ill d. Injured laborers

ANS: C Mentally ill individuals were considered undeserving poor and did not receive any community assistance. Widowed women, orphaned children, and injured laborers were deserving poor because their poverty was beyond their control.

13. What is the greatest single source of air pollution in the United States? a. Waste incineration b. Power plants c. Motor vehicles d. Molds

ANS: C Mobile sources such as cars and trucks are the greatest single source of air pollution in the United States. Waste incineration and power plants are major contributors after motor vehicles. Molds contribute to poor indoor air quality.

18. A nurse working at the individual level to reduce pollution in the environment would most likely be involved in what activity? a. Providing a tax incentive to factories that do not pollute b. Making laws related to allowed levels of pollution in the area c. Recycling paper, glass, cans, and plastic d. Moving to an area with less pollution

ANS: C Nurses can reduce pollution by doing their part, which can include choosing to recycle paper, glass, cans, and plastic. Providing a tax incentive and creating laws would not be completed at the individual level to reduce pollution in the environment. Moving to an area with less pollution would not help to improve the problem.

26. A nurse is caring for a client who has been diagnosed with a parasitic infection. Which of the following information should the nurse know when caring for this client? a. The medication to prescribe to treat these infections b. The nature and symptoms of all parasitic illnesses c. What specimens to collect and how and when to collect them d. Public policy about parasitic infections

ANS: C Nurses need to be cognizant about what specimens to collect, how and when to collect, and what laboratory techniques to use. Proper specimen collection is necessary so that the clinical diagnosis can be confirmed. It is not necessary for the nurse to know the signs and symptoms for all parasitic infections in order to provide care for the client. Public policy about parasitic infections is not important to know when providing care for the client.

21. A nurse is demonstrating advocacy in personal nursing practice. Which of the following actions best demonstrates this principle? a. Offering a smoking cessation program b. Screening for hypertension c. Lobbying for health care reform d. Conducting home visits

ANS: C Nurses should participate in implementing new directions for health care and help envision these new directions. Nurses can be an important voice in advocating for access to consistent, effective, and efficient health care for all. This is best accomplished by performing interventions at the population level.

4. A nurse is forced to evaluate personal beliefs, values, and knowledge about poverty. Which of the following experiences most likely explains the situation the nurse is experiencing? a. A client asking for a small amount of money at a clinic b. Setting up a homeless shelter c. Making a home visit to a home that is unkempt d. Reading about poverty in a textbook

ANS: C Nurses' behaviors in situations are influenced by their relationship with clients. It is important to evaluate clients and populations in the context of the environment to develop nursing interventions. Reading a book is not adequate. Setting up a homeless shelter or having a client ask for money does not force the nurse to self-reflect as much as when the nurse is in the client's environment.

11. A nurse is assessing the occupational health risks of a migrant farmworker. Which health risk is the nurse most likely to find? a. Denial of care when going to the emergency room b. Difficulty obtaining affordable health insurance c. Increased exposure to chemicals and work-related injuries d. Episodic violence from farm owners and employers

ANS: C Occupational health risks for migrant farmworkers include working conditions, exposure to chemicals, and using machinery. Denial of care and difficulty obtaining health insurance are not occupational health risks. Episodic violence from farm owners is not a known occupational health risk.

17. A nurse is providing primary prevention when working with the homeless. Which of the following best describes what is being implemented? a. Providing emergency housing aid b. Creating drug and alcohol treatment options c. Offering effective job training programs d. Evaluating comprehensive case management programs

ANS: C Offering effective job training programs is an example of primary prevention—attempting to prevent the problem before it occurs. Emergency housing is secondary prevention. Drug and alcohol treatment and comprehensive case management are tertiary preventions.

21. An advanced public health nurse (APHN) wants to establish an independent practice. Which is the best method to achieve this goal? a. Act as a legal expert. b. Learn complex assessment skills. c. Market a service package. d. Serve as a liaison for community programs.

ANS: C One of the best ways for an APHN to establish an independent practice is to market a service package, providing certain services for a specific population. Acting as a legal expert, serving as a liaison, and learning complex assessment skills would not be helpful in establishing an independent practice.

7. A nurse is using surveillance to collect outcome data. What information would most likely be collected? a. Number of clinic services which use evidence-based protocols b. Proportion of the population vaccinated against influenza c. Incidence of breast cancer in the population d. Probability of a bioterrorism attack occurring in the community

ANS: C Outcome data focus on change in health status; incidence rates are one example of this type of information. Process data focuses on what is done, such as services provided or protocols for health care delivery. An example of process data is collection of data about the proportion of the eligible population vaccinated against influenza in any 1 year.

14. Which is a principle of good practice for community partnerships? a. Partnerships are easily formed among those working in public health. b. Partnerships use their own roles to accomplish tasks. c. Partnerships are built on identified strengths and assets. d. Partnerships are loose associations among people.

ANS: C Partnerships are built upon identified strengths and assets, including areas that need improvement. This is one of the principles of good practice. An effective partnership is formed by balanced power and understanding among the partners. Partnerships use mutual trust and respect genuineness and commitment as part of their relationship.

19. What is the first step the nurse would take when addressing the problem of air pollution in the community? a. Setting standards b. Monitoring c. Permitting d. Compliance

ANS: C Permitting is a process by which the government places limits on the amount of pollution emitted into the air or water. Environmental standards may describe a permitted level of emissions, a maximum containment level, an action level for environmental cleanup, or a risk-based calculation. Monitoring is an ongoing process after an action has happened. Compliance refers to the processes for ensuring that permit/standard/regulatory requirements are met.

9. The case rate of new tuberculosis (TB) is highest among which ethnicity in the United States? a. African American b. Native American c. White American d. Asian American

ANS: D Of the new cases, 66% are foreign-born persons living in the United States, with Asians and Hispanics being the most common ethnic groups, representing 30% and 28% of national TB cases. The TB rates for African American, Native American, and White American are lower than the Asian and Hispanic populations.

22. A nurse is a political advocate for the migrant population. Which of the following best describes the action being taken by the nurse? a. Upholding immigration laws and deporting illegal aliens b. Passing laws to improve the living conditions of migrant farmworkers c. Educating communities about the health problems of migrant farmworkers d. Learning to speak Spanish for better communication with migrant farmworkers

ANS: C Political advocacy can include educating communities about these individuals. This education helps to provide legislators and policy makers at local, state, and national levels with the information needed to make policy decisions.

25. A community health nurse visits a homeless shelter to provide directly observed therapy (DOT) to several clients who have been diagnosed with TB. Which of the following best describes the rationale for this nursing intervention? a. Homeless clients do not care about their health. b. Homeless clients do not have access to medications. c. Poor adherence can result in drug resistance. d. These medications are so powerful, clients must be observed for reactions.

ANS: C Poor adherence has led to antibiotic-resistant strains. These clients may care about their health but may have difficulty adhering to the treatment regimen. Medications to treat TB are available to the homeless population. The concern with the antimicrobial treatment is with non-adherence, not with side effects.

22. When a nurse is evaluating the components of an educational program. Which of the following best describes the action that is being taken by the nurse? a. Short-term evaluation b. Educational product c. Process evaluation d. Long-term evaluation

ANS: C Process evaluation means looking at the components of an educational program. The educational product is the outcome of the educational process. Short-term evaluation evaluates the immediate effects of a health program. Long-term evaluation is geared toward following and assessing the status of an individual, family, community, or population over time.

19. What question should be answered through a program evaluation? a. Did the audience enjoy the program? b. Is a process evaluation also necessary? c. Are the problems it was designed to solve being solved? d. Was enough information given to change policy?

ANS: C Program evaluation is a means of documenting accountability. The question "Are the problems it was designed to solve being solved?" addresses the aspect of accountability. The other questions should not be the focus of program evaluation.

3. A home care nurse is employed by a proprietary agency. Which statement is true about such an agency? a. It is exempt from federal income taxes. b. It is governed by a board of directors. c. It is a profit-making agency. d. It is reimbursed primarily by tax funds.

ANS: C Proprietary agencies are profit-making agencies. Proprietary agencies are free-standing, for-profit agencies that are required to pay taxes. Many are part of large chains and now dominate the home care industry.

11. A nurse is implementing tertiary prevention strategies in the community. Which of the following best describes what the nurse is doing? a. Providing immunizations b. Conducting lead screening activities for children c. Providing case management services for clients with chronic illness d. Identifying and treating clients in a tuberculosis (TB) clinic

ANS: C Providing case management services for clients with chronic illness demonstrates tertiary prevention, as it is focused on treatment and rehabilitation. Providing immunizations is a primary prevention, and conducting lead screening activities and identifying and treating clients in a TB clinic are at the secondary level of prevention.

11. A nurse is developing a one-stop service to meet the needs of a vulnerable group. Which of the following would the nurse most likely create? a. Wrap-around services where mental services are linked b. Giving all immunizations on a single clinic visit c. Providing multiple services during a single clinic visit d. Providing free services to the medically indigent

ANS: C Providing multiple services during a single clinic visit makes services more responsive to the combined effects of social and economic stressors. Wrap-around services provide comprehensive health as well as social and economic services, so it would include more than the linkage of mental health services. Administering all immunizations or providing free services would not provide a one-stop shop for all needed services.

3. A nurse is working in a public health department. Which of the following would most likely be the focus of the care that is provided? a. Home visiting and hospice care b. Care of communicable diseases c. Health promotion d. Illness prevention

ANS: C Public health nurses (PHNs) employed in local, state, and federal agencies work together to identify, develop, and implement interventions that will improve and maintain the nation's health.

11. Requirements for disease reporting in the United States are mandated by what entity? a. The Centers for Disease Control and Prevention (CDC) b. Federal laws c. State laws and regulations d. The World Health Organization (WHO)

ANS: C Requirements for disease reporting in the United States are mandated by state laws and regulations. These diseases are then reported to the CDC. State public health officials collaborate with the CDC to determine which diseases should be nationally notifiable. The federal government and WHO are not involved in disease reporting.

13. A population-focused advanced public health nursing (APHN) activity is implementing a secondary prevention strategy. Which of the following best describes the intervention that is being completed? a. Flu immunizations at churches b. Education on need for early detection c. Screenings for blood pressure and cholesterol d. Follow-up clinics for diabetics

ANS: C Secondary prevention focuses on early detection and screening—screenings for blood pressure and cholesterol. Flu immunizations at churches and education on need for early detection are primary prevention strategies, and follow-up clinics for diabetes are a tertiary prevention strategy.

5. A nurse in the occupational health arena is implementing a secondary prevention strategy. Which of the following best describes the action that will be taken by the nurse? a. Providing education on safety in the workplace to prevent injury b. Working with workers who have diabetes to ensure they have appropriate medications c. Screening for hearing loss resulting from noise levels in the plants d. Ensuring that a person with cardiovascular disease attends a rehab program

ANS: C Secondary prevention occurs after a disease process has already begun—screening for hearing loss. Tertiary prevention includes those interventions aimed at disability limitation and rehabilitation from disease, injury, or disability—working with workers who have diabetes and ensuring that a person with cardiovascular disease attends rehab. Primary prevention is aimed at prevention of a disease before it occurs—providing education on safety in the workplace to prevent injury.

15. A nurse is assessing a worker who has had a health reaction to a "safe" low-level exposure. Which of the following most likely describes what has happened? a. An allergic reaction b. A compromised immune system c. Hypersusceptibility d. Malnutrition

ANS: C Several host factors appear to be associated with this hypersusceptibility: light skin, malnutrition, compromised immune system, glucose-6-phosphae dehydrogenase deficiency, serum alpha1-antitrypsin deficiency, chronic obstructive pulmonary disease, sickle cell trait, and hypertension. A worker who has a health reaction to "safe" low-level exposures is experiencing hypersusceptibility.

22. A nurse is promoting social justice. Which of the following actions would the nurse most likely take? a. Contacting lawmakers about environmental health issues b. Assisting at homeless shelters c. Advocating for policies to improve social conditions d. Serving on a local coalition to prevent obesity

ANS: C Social justice refers to providing equitable care and social supports for the most disadvantaged members of society. Nurses can function as advocates for policy changes to improve social, economic, and environmental factors that predispose vulnerable populations to poor health.

11. A nurse refers a client to an assistance program that requires the individual to find a job and/or enroll in job training program. Which program is being used? a. Aid to Families with Dependent Children (AFDC) b. Medicaid c. Temporary Assistance for Needy Families (TANF) d. Women, Infants, and Children (WIC)

ANS: C TANF requires participants to find jobs and/or enroll in job training programs. The current TANF program was formerly called AFDC. Medicaid provides health insurance to poor families but does not require participants to find jobs. WIC provides food vouchers entitling participants to free nutritious foods and infant formulas from local grocers but does not require participants to find jobs.

24. The nurse teaches the family of an AIDS client about managing symptomatic illness by preventing deteriorating conditions, such as diarrhea, skin breakdown, and inadequate nutrition. Which of the following best describes the action taken by the nurse? a. Primary prevention b. Secondary prevention c. Tertiary prevention d. Primary health care

ANS: C Tertiary prevention includes those interventions aimed at disability limitation and rehabilitation from disease, injury, or disability. Primary prevention refers to those interventions aimed at preventing the occurrence of disease, injury, or disability. Secondary prevention focuses on early detection and prompt treatment of disease, injury, or disability. Primary health care refers to the first line of care provided to patients typically by a physician or other health care provider.

21. A nurse works with a group of abused women to enhance their levels of self-esteem. Which of the following levels of prevention is being performed? a. Primary level of prevention b. Secondary level of prevention c. Tertiary level of prevention d. Health promotion

ANS: C Tertiary prevention includes those interventions aimed at disability limitation and rehabilitation from disease, injury, or disability. Primary prevention aims to prevent disease and illness before it occurs. Secondary prevention focuses on screening and early detection. Health promotion is not a level of prevention.

16. A nurse is developing a community-based exercise program for a group of women who have cardiovascular disease. Which of the following levels of prevention is being used? a. Primary prevention b. Secondary prevention c. Tertiary prevention d. Health promotion

ANS: C Tertiary prevention includes those interventions aimed at disability limitation and rehabilitation from disease, injury, or disability. Primary prevention focuses on prevention on disease before it occurs. Secondary prevention aims for early detection and screening. Health promotion is not a level of prevention.

6. The nurse provides direct care services to a stroke victim to avoid complications. Which of the following levels of prevention is being implemented? a. Primary prevention b. Secondary prevention c. Tertiary prevention d. Assessment

ANS: C Tertiary prevention includes those interventions aimed at disability limitation and rehabilitation from disease, injury, or disability. Thus, direct care for a stroke victim is focusing on limiting disability and encouraging rehabilitation. Primary prevention focuses on preventing the disease before it occurs. Secondary prevention focuses on early detection and screening. Assessment is not a level of prevention.

7. An employer offers a limited duty program after an employee has experienced a cumulative trauma injury. Which of the following levels of prevention is being implemented? a. Primary prevention b. Secondary prevention c. Tertiary prevention d. Assessment

ANS: C Tertiary prevention includes those interventions aimed at disability limitation and rehabilitation from disease, injury, or disability—limited duty program after injury. Primary prevention refers to those interventions aimed at preventing the occurrence of disease, injury, or disability. Secondary prevention occurs after a disease process has already begun.

15. A nurse advises a client who has HIV not to donate blood, plasma, or organs. Which of the following levels of prevention is being used? a. Primary prevention b. Secondary prevention c. Tertiary prevention d. Health promotion

ANS: C Tertiary prevention includes those interventions aimed at disability limitations and rehabilitation from disease, injury, or disability. Primary prevention focuses on prevention on disease before it occurs. Secondary prevention aims for early detection and screening. Health promotion is not a level of prevention.

16. The nurse provides footwear and gloves to leprosy clients to prevent trauma to their insensitive and deformed hands and feet. Which of the following best describes the intervention used by the nurse? a. Primary level of prevention b. Secondary level of prevention c. Tertiary level of prevention d. Primary health care

ANS: C Tertiary prevention reduces complications through treatment and rehabilitation. Primary prevention seeks to reduce the incidence of disease by preventing occurrence. Secondary prevention seeks to prevent the spread of infection and/or disease once it occurs. Primary health care is the essential health care services provided by physicians and other health care providers.

15. A community identifies cardiovascular disease as a priority need because its mortality rate is four times higher than the rate identified in Healthy People 2020. Which of the following describes the type of approach that was used? a. Pre-active b. Reactive c. Inactive d. Interactive

ANS: C The inactive approach defines the problem based on the existing health state of the population to be served. A pre-active approach projects a future need. A reactive approach defines the problem based on past needs. An interactive approach describes the problem using past and present data to project future population needs.

14. A client is self-employed as a mechanic and has no health insurance coverage. Which of the following best describes the legislation that will assist this client in obtaining health insurance? a. Balanced Budget Act NURSINGTB.COM b. Health Insurance Portability and Accountability Act (HIPAA) c. Patient Protection and Affordable Care Act (ACA) d. Social Security Act

ANS: C The ACA of 2010 provides the opportunity for all to purchase health insurance. The Balanced Budget Act shifted payment in home health care. The HIPAA was intended to help people keep their health insurance when moving from one place to another. The Social Security Act created the largest federal support program for elderly and poor Americans.

15. A nurse is providing information to a local newspaper about the presence of infectious diseases in the United States. Which of the following statements by the nurse is accurate? a. "It is the goal of the WHO to prevent the transmission of the plague by avoiding direct contact with inflicted individuals." b. "Rabies is easily spread by contact with animals." c. "The United States is a certified polio-free country." d. "The onset of tularemia is characterized by a distinct skin lesion often called a bull's-eye lesion."

ANS: C The Americas are certified polio free. The plague is a vector-borne disease and cannot be spread by direct contact with inflicted individuals. Rabies is a rare event because of the widespread vaccination of dogs in the 1950s. The Americas were certified as polio free in 1994. The onset of Lyme's disease is characterized by a bull's-eye lesion.

22. Which environmental law sets basic structure for regulating pollutants to United States waters? a. Safe Drinking Water Act b. Toxic Substance Control Act c. Clean Water Act d. Pollution Prevention Act

ANS: C The Clean Water Act sets basic structure for regulating pollutants to United States waters. The Safe Drinking Water Act authorized the EPA to establish safe standards of purity and required all owners or operators of public water systems to comply with primary standards. The Toxic Substance Control Act gives the EPA the ability to track the 75,000 industrial chemicals currently produced or imported into the United States. The Pollution Prevention Act focused industry, government, and public attention on reduction of the amount of pollution through cost-effective changes in production, operation, and use of raw materials.

8. A nurse is discussing eligibility for federal food assistance with a family. Which federal guideline would a nurse refer to when considering financial eligibility? a. Poverty Threshold Guideline b. Consumer Price Index (CPI) c. Federal Income Poverty Guideline d. Temporary Assistance to Needy Families (TANF)

ANS: C The Federal Income Poverty Guideline is issued by the U.S. Department of Health and Human Services and is used to determine if a person or family is financially eligible for assistance or services from various federal programs. The Poverty Threshold Guidelines are used primarily for statistical purposes. The CPI is a measure of the average change over time in the prices paid by households for a fixed market basket of consumer goods and services. TANF is a program, not a guideline, that assists families in need.

15. Employees working with hazardous chemicals have the "Right to Know" about the chemicals they are working with through the creation of which resource? a. Material Safety Data Sheet (MSDS) b. Consumer Confidence Report c. Hazard Communication Standard d. Environmental Protection Agency (EPA)

ANS: C The Hazard Communication Standard requires employers to maintain a list of all hazardous chemicals that are used on site. Each of the chemicals should have an associated chemical information sheet known as an MSDS, which is written by the chemical manufacturer. Consumer Confidence Reports summarize the results of the annual testing of the public water supply. The EPA sets forth policies, regulations, and public laws for the safety of the environment and the population.

10. Which was the first legislation that specifically required certain prevention programs for workers? a. Occupational Safety and Health Act b. National Institute for Occupational Safety and Health (NIOSH) c. Mine Safety and Health Act d. SIC Code

ANS: C The Mine Safety and Health Act was enacted in 1968. It was the first legislation that specifically required certain prevention programs for workers. The Occupational Safety and Health Act established OSHA and NIOSH to carry out the Act's purpose of ensuring "safe and healthful working conditions for working men and women." NIOSH examines potential hazards of new work technologies and practices. The SIC Code did not require certain protection programs for workers.

5. What is the main purpose of the National Center for Injury Prevention and Control? a. Assist victims of violence. b. Maintain records of the incidence of injuries. c. Coordinate the prevention of injuries, violence, and their consequences. d. Encourage collaboration among professionals from various disciplines.

ANS: C The National Center for Injury Prevention and Control was established to coordinate the prevention of injuries, violence, and their consequences. The work of this center includes providing grants that fund prevention programs, dissemination of research findings, and maintenance of a website, blogs, podcasts, and electronic newsletters. This funding helps to inform the public of the epidemiology of injury and to provide other resources for professionals.

17. What is the purpose of the National Incident Management System (NIMS)? a. Create a new branch of government that deals with bioterrorism. b. Establish a way for the Red Cross to carry out its mission. c. Develop a nationwide all-hazards approach to domestic incident management. d. Extend presidential power to act quickly upon acts of bioterrorism.

ANS: C The National Response Framework is a unified, all-discipline, all-hazards approach to domestic incident management. It is built upon scalable, flexible, and adaptable coordinating structures to align key roles and responsibilities linking all levels of government and organizations. The Homeland Security Act of 2002 created a new branch of government that deals with bioterrorism, the U.S. Department of Homeland Security. The NIMS did not impact the Red Cross or extend presidential power to act upon bioterrorism.

14. A community health nurse is using the Omaha System. Which of the following best describes the rationale for using this system? a. To define each health care provider's role b. To use nursing diagnoses in community health nursing practice c. To foster collaborative practice d. To provide practitioners a method for communication

ANS: C The goals of developing the Omaha System were to develop a structured and comprehensive system that could be both understood and used by members of various disciplines and to foster collaborative practice. Therefore, the Omaha System was designed to guide practice decisions, sort and document pertinent client data uniformly, and provide a framework for an agency-wide, multidisciplinary clinical information management system capable of meeting the daily needs of clinicians, managers, and administrators.

7. A public health nurse (PHN) aims to change behaviors such as poor hand washing skills and unsafe sex practices. Which of the following best describes the role that the nurse is using? a. Case manager b. Advocate c. Educator d. Role model

ANS: C The PHN works as an educator, teaching to the level of the client so that information received is information that can be used. The PHN who aims to change behaviors, such as poor hand washing skills and unsafe sex practices, is acting in the role of educator. The public health function of a nurse who assists a client in identifying the services needed the most at the least cost is a case manager. As an advocate, the PHN collects, monitors, and analyzes data and works with the client to identify and prioritize needed services, whether the client is an individual, a family, a community, or a population. A PHN discusses with a client which services are appropriate to meet her needs. A nurse serves as a role model for the client by being a person for the client to look up to.

10. A nurse has discovered that a client should be receiving Meals on Wheels. Which of the following steps of the nursing process is the nurse performing? a. Diagnosis b. Planning c. Assessment d. Implementation

ANS: C The Standards of Care and the Standards of Professional Performance both include the six steps of the nursing process: (1) assessment, (2) diagnosis, (3) outcomes identification, (4) planning, (5) implementation, and (6) evaluation. As a nurse performs an assessment, the nurse recognizes the current needs of the client.

12. Which legislation changed the availability of federal services accessible to certain immigrants to the United States? a. Migrant Health Act of 1962 b. Supplemental Security Income (SSI) c. Welfare reform legislation of 1996 d. Food stamps

ANS: C The Welfare reform legislation of 1996 changed the availability of federal services accessible to certain immigrants in the United States. The Migrant Health Act provides funds for primary and supplemental health services to migrant workers and their families. Because of the welfare reform legislation of 1996, many legal immigrants and unauthorized immigrants are ineligible for services such as SSI and food stamps.

4. What does the cognitive domain of learning include? a. Changes in attitudes and the development of values b. The performance of skills c. Memory, recognition, understanding, reasoning, and problem solving d. Memorization of one set of skills before moving on to the next

ANS: C The cognitive domain includes memory, recognition, understanding, reasoning, and problem solving. The affective domain includes changes in attitudes and the development of values. The psychomotor domain includes the performance of skills that require some degree of neuromuscular coordination and emphasizes motor skills. Memorization of skills is only one part of the cognitive domain.

5. Which statement accurately describes how nurse practitioner (NP) programs began? a. Started as a physician's assistant program. b. Were an offshoot of midwifery. c. Have not always required a master's degree. d. Began in Europe and didn't reach the United States until 1970.

ANS: C The early NP degree programs were often continuing education programs and certificate programs that did not require a master's degree or even a baccalaureate degree. NP programs began with a focus on well children. The NP movement began in 1965 at the University of Colorado.

15. A nurse is implementing a community health education program at a local church. Which of the following educational principles should the nurse implement when providing this education? a. Refer to trustworthy sources. b. Use an active voice. c. Create the best learning environment. d. Accentuate the positive health behaviors of the participants.

ANS: C The environment must be conducive to learning for educational programs to be effective. The environment should be free of distractions and consistent with the message. The other strategies may assist with the educational program but are not one of the major educational principles discussed in the textbook.

8. Which example contains the components necessary to form an epidemiologic triangle? a. Pesticides, water, food b. Lead, mercury, soil c. Trichloroethylene, water, infants d. Children under 12, elderly, temperature

ANS: C The epidemiologic triangle consists of an agent (chemical), host (community consisting of several variants), and environment (air, water, soil, etc.). The other examples do not contain an agent, host, and environment.

15. A nurse is trying to facilitate interprofessional collaboration. Which of the following actions should be taken by the nurse? a. Understand who oversees the client's care. b. Make appropriate referrals. c. Recognize what other professionals do and how they view their roles. d. Request assistance when performing complex skills.

ANS: C The factors for successful interprofessional functioning include the categories of knowledge, skill, and attitudes with subheadings including the understanding of the roles of each professional.

17. A nurse is completing the first phase of a risk assessment. Which of the following questions would the nurse most likely try to answer? a. Has the chemical been released into the environment? b. How much and by which route of entry can the chemical enter the body? c. Is the chemical known to be associated with a negative health effect? d. What is the prediction for potential harm?

ANS: C The first phase is determining if a chemical is known to be associated with negative health effects (in animals or humans): Is the chemical known to be associated with a negative health effect? The second step is determining if the chemical has been released into the environment: Has the chemical been released into the environment? The third step is estimating how much and by which route of entry the chemical might enter the human body: How much and by which route of entry can the chemical enter the body? The final step considers the previous steps: What is the prediction for potential harm?

25. A group member has taken on the role of the "gatekeeper." Which of the following actions would most likely be taken by this member? a. Seek and accept the authority or direction of others. b. Guide and direct the group activity. c. Control outsiders' access to the group. d. Focus the movement toward the main work of the group.

ANS: C The gatekeeper controls outsiders' access to the group. The follower seeks and accepts the authority or direction of others. The leader guides and directs group activity. The task specialist focuses or directs movement toward the main work of the group.

1. How are official agencies financed primarily? a. Charities b. Individual clients c. Third-party payers d. Tax funds

ANS: D Official agencies are financed primarily by tax funds. These agencies are typically operated by state, county, city, or other local government units, such as health departments.

18. Which program planning stage defines the problem based on the existing health status of the population to be served? a. Pre-active b. Reactive c. Inactive d. Interactive

ANS: C The inactive stage defines the problem based on the existing health status of the population to be served. A pre-active approach projects a future need. Identifying the need for more diabetes health educators based on the projected incidence of diabetes is an example of the pre-active approach. A reactive approach defines the problem based on past needs. An interactive approach describes the problem using past and present data to project future population needs.

l 18. What is the most chronic bloodborne infection in the United States? a. Hepatitis A b. Hepatitis B c. Hepatitis C d. HIV

ANS: C The most chronic bloodborne infection in the United States is hepatitis C. Hepatitis A virus is most commonly transmitted through the fecal-oral route. Sources may be water, food, feces, or sexual contact. The hepatitis B virus (HBV) is spread through blood and body fluids and, like HIV, is a bloodborne pathogen. HIV is transmitted through exposure to blood, semen, transplanted organs, vaginal secretions, and breast milk. Although hepatitis B and HIV are both bloodborne pathogens neither are the most chronic infection like hepatitis C.

23. A nurse has created the program's objective "to decrease the incidence of early childhood disease in Center County by 10% to 25% by providing immunization clinics in all schools before August and December 2018." Which of the following describes the operational indicator? a. Decrease b. Promoting immunization clinics c. 10% to 25% d. Before August and December 2018

ANS: C The operational indicator is 10% to 25%; decrease is the action verb; promoting immunization clinics is the purpose; and before August and December 2018 is the time frame.

7. What is the primary cause of vulnerability? a. Race b. Age c. Poverty d. Illness

ANS: C The primary cause of vulnerability is poverty. Race, age, and illness may cause populations to be vulnerable, but these factors are not the primary cause.

6. A nurse is caring for a client who has been diagnosed HIV positive. Which of the following roles of the nurse would be the most important for the nurse to implement? a. Advocate, lobbying for AIDS research b. Counselor, discussing implications of future sexual activity c. Role model, providing supportive care d. Policy maker, addressing laws governing privacy rights of HIV-positive persons

ANS: C The priority role of the nurse should be role model, as nurses must be role models because many HIV-positive patients are stigmatized. The need to be an advocate and policy maker does not address the individual needs of the client. The nurse can counsel the client while providing supportive care.

21. A nurse is conducting an occupational health assessment. Which of the following best describes the rationale for this assessment? a. It is required by the Occupational Safety and Health Act (OSHA). b. It is required by National Institute for Occupational Safety and Health (NIOSH). c. It helps to identify agent and host factors that place employees at risk. d. It helps to educate workers about potential hazards.

ANS: C The purpose of this assessment is to become knowledgeable of the work processes and the materials, the requirements of various jobs, the presence of actual or potential hazards, and the work practices of employees. This assessment allows the nurse to identify agents and host factors that place employees at risk. An occupational health assessment is not required by any organization.

7. There are two medically indigent clients in the clinic who have come to get their monthly supply of free insulin. There is only enough for one client. Which of the following actions would the nurse take first? a. Identify all options. b. Determining who arrived first. c. Gather additional information. d. Act and assess decisions made.

ANS: C The steps of the ethical decision-making framework are to first identify the ethical issues and dilemmas, then place them within a meaningful context, obtain all relevant facts, reformulate ethical issues and dilemmas if needed, consider appropriate approaches to actions or options, make decisions and take action, and evaluate decisions and action.

13. A nurse is assessing host factors as part of the epidemiologic triad. Which of the following factors would the nurse most likely assess? a. Crowding b. Shift work c. Worker's family d. Chemical exposure

ANS: C The worker's family is considered a part of the host factor. Crowding and shift work are environmental factors, and chemical exposure is the agent factor.

6. A nurse is providing home care for several elderly clients in the community. Which of the following situations would most likely be indicative of elder abuse? a. A daughter refuses to visit her mother due to work commitments. b. A child runs around a grandparent's house breaking items. c. A young man repeatedly steals money from his grandmother. d. An older person demands that the family come for dinner.

ANS: C Theft or mismanagement of money or resources is an element of abuse. Recognition of abuse includes these other possibilities as well: willful infliction of physical pain or injury, inflection of debilitating mental anguish and fear, or unreasonable confinement or the depriving of services. The other examples do not describe these elements of abuse.

18. A nurse is caring for a homeless population. Which of the following characteristics should the nurse anticipate as a need of this population? a. Need more nursing care than other vulnerable groups. b. Have no desire to seek medical care. c. Have even fewer resources than poor people who have adequate housing. d. Are living in despair with no hope or resilience.

ANS: C Those who are homeless have even fewer resources than poor people who have adequate housing. Homeless and marginally housed people must struggle with heavy demands as they try to manage daily life because their resources are limited. These individuals must cope with finding a place to sleep at night and a place to stay during the day or moving frequently from one residence to another, as well as finding food, before even thinking about health care.

1. Which statement about eating disorders is correct? a. Individuals with anorexia frequently complain about weight loss. b. Purging is associated with anorexia. c. Most women with bulimia are concerned with the shape and weight of their body. d. Bulimia has more medical complications than anorexia.

ANS: C Those with bulimia are usually concerned with the shape and weight of their body. Those with anorexia view themselves as normal or overweight, purging is associated with bulimia, and anorexia is considered to have more complications than bulimia.

19. A public health nurse (PHN) would like to increase the immunization coverage of infants and toddlers. Which of the following strategies would be appropriate for the nurse to use? a. Read the Mortality and Morbidity Weekly Report to learn about vaccinations. b. Require that children have all their immunizations before going to public school. c. Track children known to be at risk for under-immunization. d. Lead teams of health care workers to enforce laws related to immunizations.

ANS: C Tracking children known to be at risk for under-immunization is a function of PHNs who work in health departments where immunizations are given and tracked. Reading the Mortality and Morbidity Weekly Report requiring that children have all their immunizations before attending school, and leading teams of health care workers would not be as effective in increasing immunization coverage for this population.

23. A community health nurse investigates an outbreak of pinworm at a local daycare center. To minimize the spread of infection, which of the following suggestions would the nurse provide to the daycare workers? a. Close the daycare until all surfaces are cleaned. b. No action is necessary because it is easily treated with oral vermicides. c. Using good hand washing is important to prevent the transmission. d. Every child in the daycare should be treated because they all are probably infected.

ANS: C Transmission of pinworm occurs through the fecal-oral route, so good hand washing after toileting is essential. It is not necessary to treat all children or close the daycare. It is necessary that action be taken, as without any action being taken, the pinworm outbreak will continue.

12. A nurse is caring for a client who has been diagnosed with a bacterial sexually transmitted infection (STI). Which of the following infections does this client most likely have? a. Trichomonas b. HIV c. Syphilis d. Genital warts

ANS: C Trichomonas, HIV, and genital warts are caused by viruses. Syphilis is caused by bacteria.

12. A public health nurse (PHN) is working with uninsured individuals. Which of the following is the most appropriate action for the nurse to take? a. Providing direct care for this population in times of acute illness b. Advocating for federal policy changes for this population c. Assisting this population to access necessary health care services d. Encouraging lifestyle changes for this population

ANS: C e populations by acting as a bridge between these populations and the resource needs for this at-risk group by approaching health care providers on behalf of individuals seeking medical/health services and keeping the needs of this population on the political agenda.

1. How would a registered nurse become a forensic nurse? (Select all that apply.) a. No additional education is needed after obtaining an undergraduate nursing degree. b. Complete additional course work in police science. c. Enroll in a certificate program for forensic nursing. d. Select a graduate program that offers a track in forensic nursing. e. Complete a nurse practitioner program.

ANS: C, D There are three primary routes for training in forensic nursing. Additional skills and knowledge can be gained through continuing education courses or basic concepts introduced in generalist education. Certificate programs and graduate programs offer additional opportunities to specialize in forensic nursing.

28. A nurse collects data about productivity of staff members to evaluate a program. Which type of study is the nurse completing? a. Cost-accounting study b. Cost-benefit study c. Cost-effectiveness study d. Cost-efficiency study

ANS: D A cost-efficiency study is designed to examine the actual cost of performing program services and to focus on productivity versus cost. A study designed to measure the quality of a program as it relates to cost is a cost-effectiveness study. A study used to assess the desirability of a program by examining costs and benefits is a cost-benefit study.Cost-accounting studies are performed to find the actual cost of a program.

24. What is the purpose of adult day health centers? a. Provide support of life until death occurs. b. Provide assistance to people who may need help with activities of daily living (ADLs). c. Provide treatment for those needing rehabilitation. d. Provide respite care relief for caregivers.

ANS: D Adult day health care is for individuals whose mental or physical function requires them to obtain more health care and supervision. It serves as more of a medical model than the senior center, and often individuals return home to their caregivers at night. Hospice provides support of life until death occurs. Assisted living provides assistance to people who may need help with ADLs. Long-term care facilities provide treatment for those needing rehabilitation.

25. An elderly person is living independently in a single room in a full-service life care community. Which of the following terms best describes this living arrangement? a. Adult daycare b. Home care c. Nursing home d. Assisted living

ANS: D An elderly person living independently in a single room in a full-service life care community is a client of an assisted living facility. Adult day health care is for individuals whose mental or physical function requires them to obtain more health care and supervision; it serves as more of a medical model than the senior center, and often individuals return home to their caregivers at night. Nursing home facilities provide treatment for those needing rehabilitation. Home care provides individual and environmental assessments, direct skilled care and treatment, and short-term guidance for individuals in the home.

20. A city sets the standard of an acceptable level of emissions or a maximum contaminant level allowed for factories. Which concept is being applied? a. Controlling pollution b. Waste minimization c. Land use planning d. Environmental standard

ANS: D An example of an environmental standard is an acceptable level of emissions or a maximum contaminant level allowed. Although this does help to control pollution, environmental standard is best applied in this example. Setting a standard for emissions does not impact waste minimization or land use planning.

1. What occurs during an act of biological terrorism? a. The intentional release of hazardous chemicals into the environment b. Environmental and occupational exposure to biological toxins c. Immunity to toxins related to repeated exposures d. An intentional release of viruses, bacteria, or other toxins

ANS: D An intentional release of viruses, bacteria, or other toxins would be described as biological terrorism. Chemical terrorism is the intentional release of hazardous chemicals into the environment intended for harming or killing. Surveillance is used to monitor environmental and occupational exposures. Immunity to toxins after repeated exposures does not occur during biological terrorism.

18. A nurse prepares for a presentation to a group of adults using strategies appropriate for adult learning. Which of the following concepts is being applied? a. Authoritarianism b. Learning style c. Pedagogy d. Andragogy

ANS: D Andragogy is a term and model developed by Malcolm Knowles to describe learning strategies for adult learners. In pedagogy the teacher assumes full responsibility for making decisions about what will be learned, and how and when it will be learned. In authoritarianism the leader controls group movement and progress through interpersonal power. Learning style refers to how someone learns best.

12. A nurse is using the educational process of selecting appropriate educational methods when planning a community health program. Which of the following steps of the nursing process does this action most resemble? a. Assessment b. Evaluation c. Implementation d. Planning

ANS: D Assessment would be identifying educational needs, evaluation would be evaluating educational process, and implementation would be implementing the educational plan. Planning is similar to selecting appropriate educational methods.

2. Which statement about migrant and seasonal farmworkers is true? a. The economic status of migrant workers has improved over the last decade. b. This is a transient population that does not need nursing care. c. Most migrant workers are eager to accept health care services. d. Availability of food in the United States depends on these individuals.

ANS: D Availability and affordability of food in the United States depends on these individuals, yet their economic and social status has not changed significantly over the past decades. Most migrant workers do not seek out health care services.

19. A nurse is working to remove barriers to receiving health care. Which of the following actions is the nurse most likely to support? a. Discrimination against certain groups b. Treatment of pets at the same facility c. Provision of free food at a food bank d. Providing services for a rural population by using a mobile clinic

ANS: D Barriers to access are policies and financial, geographic, or cultural features of health care that make services difficult to obtain or so unappealing that people do not want to seek care. Examples of removing barriers include providing extended clinic hours, low-cost or free health services for people who are uninsured or underinsured, transportation, mobile vans, and professional interpreters, which can help improve access to care.

5. A nurse is teaching a client about how to complete a wound dressing change. Which of the following conditions must be met before learning will occur? a. Must be able to memorize the instructions, relay this information to a partner, and demonstrate the dressing change. b. Must master the dressing change at the time it is taught, repeat the demonstration for the nurse, and teach another person. c. Must be able to speak the language of the nurse, have time to practice the dressing change, and master the dressing change in a short time. d. Must have the necessary ability, a sensory image of how to carry out the dressing change, and an opportunity to practice the dressing change.

ANS: D Before psychomotor learning occurs, the learner must have the necessary ability, a sensory image of how to carry out the skill, and an opportunity to practice the skill. This is the only option that contains all of the necessary requirements of the learner.

2. A nurse is applying the knowledge and processes of ethics to the examination of ethical problems in health care. Which of the following describes the actions of the nurse? a. Values b. Morality c. Ethics d. Bioethics

ANS: D Bioethics applies the knowledge and processes of ethics to the examination of ethical problems in health care. Values are beliefs about the worth or importance of what is right or esteemed. Morality is shared and generational societal norms about what constitutes right or wrong conduct. Ethics is a branch of philosophy that includes both a body of knowledge about the moral life and a process of reflection for determining what persons ought to do or be regarding this life.

18. An employee has come into contact with a biological agent. Which consideration should be made by the nurse? a. The biological agent may cause accidents in the work environment. b. The biological agent is commonly found in the agriculture industry. c. The biological agent may potentiate stress in the workplace. d. The biological agent indicates exposure to bacteria, viruses, fungi, or parasites.

ANS: D Biological agents are living organisms whose excretions or parts are capable of causing human disease, usually by an infectious process. Biological hazards are common in workplaces such as health care facilities and clinical laboratories where employees are potentially exposed to a variety of infectious agents, including viruses, fungi, and bacteria.

6. The growing multiculturalism of American society can contribute to ethnicity conflicts when considering what? a. Providing care to different cultural groups b. Aligning individual values with the cultural norms c. Ethnic groups overburdening the health care system d. The greater community's values being jeopardized by specific ethnic values

ANS: D Callahan offered perspectives on judging diversity and suggests a thoughtful tolerance and some degree of moral persuasion (not coercion) for ethnic groups to alter values so that they are more in keeping with what is normative in American culture. Providing care to different cultural groups should not produce an ethnicity conflict. Individual alignment with cultural norms would make it less likely that an ethnicity conflict would occur. Ethnic groups using the health care system will not cause it to be overburdened or result in an ethnicity conflict.

5. Which of the following disorders has been identified as an occupational health risk for agricultural workers? a. Stroke b. Heart disease c. Diabetes d. Cancer

ANS: D Cancer is an identified but not well-documented health problem for migrant farmworkers associated with their exposure to chemicals. Other identified health risks are musculoskeletal injuries, traumatic injuries, respiratory problems, dermatitis, infectious diseases, and eye problems.

9. Community preparedness is being used to plan for a disaster. Which of the following tasks is being implemented? a. Assembling emergency supplies b. Understanding the workplace disaster plan c. Taking a disaster training course d. Developing an evacuation plan to remove individuals from danger

ANS: D Community preparedness involves developing an evacuation plan to remove individuals from danger. Assembling emergency supplies is an example of individual preparedness. Understanding the workplace disaster plan is an example of preparedness in the workplace within the community. Taking a disaster training course is an example of professional preparedness.

9. A nurse operates a school-based clinic in a local school where multiple providers and disciplines offer care to children, making it easier for children to access health care. Which of the following best describes this approach? a. Advocacy b. Wrap-around services c. Social justice d. Comprehensive services

ANS: D Comprehensive services are health care services that focus on more than one health problem or concern. Stationary or mobile clinics that provide a wide array of health promotion, illness prevention, and illness management services in migrant camps, schools, and local communities are examples of this. Wrap-around services describe a system in which comprehensive health services are available and social and economic services are "wrapped around" these services. Advocacy refers to actions taken on behalf of another. Social justice describes justice with respect to the concepts of egalitarianism and equality.

3. A nurse uses the scope and standards for forensic nursing practice to differentiate this practice from other nursing specialties. Which parameter best describes the practice of forensic nursing? a. Assessment includes assessing the community for potential violence. b. Identification of outcomes of care is accomplished by collaborating with healthcare professionals. c. Coordination of care involves education and other strategies to prevent injuries well after the occurrence. d. Evaluation of outcomes relates to program evaluation.

ANS: D Coordination of care relevant to forensic outcomes often involves education and other strategies to prevent injuries well after the occurrence, perhaps after the death of an individual who has been injured. Assessment includes the collection of forensic data, identification of realistic outcomes of care is most commonly performed in collaboration with a team of professionals outside of nursing and medicine, and evaluation of outcomes may involve dissemination of results to the client and the community.

26. What is the purpose of performing a cost-accounting study? a. To detect what percentage of cost the clients recover b. To measure how long it takes for clients to see a provider c. To note how many cases are associated with other major risk factors d. To find the actual cost of a program

ANS: D Cost-accounting studies are performed to find the actual cost of a program. Cost-accounting studies do not detect what percentage of clients recover, measure how long it takes for clients to see a provider or note how many cases are associated with other major risk factors.

10. Which statement describes what occurs during the preparedness stage of disaster management? a. Heightened inspection and increased security in the community b. Incorporation of provision for pets into local disaster plans c. Purchase of personal protective equipment for all citizens d. Assembly of disaster kits for the home, workplace, and car

ANS: D Disaster kit assembly for the home, workplace, and car, especially by nurses, occurs during the preparedness phase. Heightened inspection and increased security in the community is part of the prevention activities. Purchase of personal protective equipment for all citizens and incorporation of provision of pets into local disaster plans are not realistic plans for preparedness.

4. A forensic nurse assesses a child who has injuries that indicate possible abuse. What is the best way to document the assessment findings? a. Tape recorded statements from the child b. Digital photographs of injuries c. Video recording to record statements and show injuries d. Written documentation and photography

ANS: D Documentation of all forms of violence should include written documentation as well as photography.

14. What information is included in a case definition? a. The precise point of contact b. Laboratory confirmation U c. Source of contamination d. Clinical symptoms

ANS: D Each case has a unique set of criteria based on what is known about the disease. It may include clinical symptoms, laboratory values, and epidemiologic criteria. The precise point of contact, laboratory confirmation, and source of contamination are not part of a case definition.

19. A nurse is working for a public health department as the community health educator. Which of the following is a potential barrier that a nurse may experience? a. Working with clients with a low literacy level b. Memorizing the information that is to be taught c. Having a limited vocabulary d. Lacking necessary knowledge

ANS: D Educators may lack knowledge about how to gain participation. Participation can be fostered by asking open-ended questions, inviting participation, and planning small-group activities whereby a person responds based on the group rather than presenting his own information. Clients with low literacy levels and limited vocabularies are learner-related barriers, not a barrier of the educator. Memorizing the information to teach may cause problems with the presentation but would not be a barrier for the educator.

21. A facility is inspected after it has obtained a permit. The inspection is completed for the purpose of observing whether the plans submitted in the permit application are being implemented as approved. Which of the following environmental protection strategies is being applied? a. Controlling pollution b. Waste minimization c. Land use planning d. Environmental monitoring

ANS: D Environmental monitoring would be an inspection of a facility after a permit is obtained to observe whether the plans submitted in the permit application are being implemented as approved. Permitting is an important step in controlling pollution. Waste minimization and land use planning are activities aimed at prevention of pollution.

20. A nurse is implementing an intervention at both the primary and tertiary levels of prevention. Which of the following best describes the nursing intervention? a. Providing emergency shelter housing b. Offering physical and mental health services c. Developing a targeted case management program d. Establishing a needle exchange program

ANS: D Establishing a needle exchange program could be considered both primary (preventing a problem before it occurs by helping clients avoid disease transmission) and tertiary (working with a population who already has a "problem" of drug abuse). Providing emergency shelter housing and offering physical and mental health services are both secondary and tertiary prevention interventions. Developing a targeted case management program is secondary prevention.

22. What is the purpose of the Occupational Safety and Health Act (OSHA)? a. Educating occupational health and safety professionals b. Identifying and research occupational health and safety hazards c. Distributing research findings relevant to occupational health and safety d. Educating employers about occupational health and safety

ANS: D OSHA educates employers about occupational health and safety. NIOSH identifies and researches occupational health and safety hazards, distributes research finding relevant to occupational health and safety, and educates occupational health and safety professionals.

15. Which statement about feminist ethics is correct? a. Feminists include only women in their worldview. b. Persons who ascribe to feminist ethics are passive and wish to pursue their ideals through the legislative process. c. Feminists believe that men should not be nurses. d. Women's thinking, and moral experiences are important and should be taken into account.

ANS: D Feminist theory ascribes to the idea that women's thinking, and moral experiences are important and should be considered. A feminist perspective leads us to think critically about connections among gender, disadvantage, and health as well as the distribution of power in public health processes. Feminists are women and men who hold a worldview advocating economic, social, and political equality for women that is equivalent to that of men.

24. A nurse recognizes the importance of group norms when planning community health education. Which of the following provides the best explanation for why this is important? a. Allows for creativity and variety among group members. b. Influences members' perception of community. c. Are helpful in evaluating the effectiveness of the group. d. Maintains the group through various supports to members.

ANS: D Group norms serve to maintain the group through various supports to members. They are the standards that guide, control, and regulate individuals and communities. They suggest what a group believes is important, what it finds acceptable or objectionable, or what it perceives as of no consequence. Group norms may limit the creativity and variety among the group members. Members' perception of the community could be influenced by group norms, but this is not why this consideration should be made when planning community health education. Group norms are not used to evaluate the effectiveness of the group.

13. After reviewing the objectives of Healthy People 2020, the public health nurse (PHN) determines the focus for the programming for communicable diseases in the community. Which of the following would the nurse most likely select as the focus? a. Encouraging community members to engage in healthy behaviors b. Identifying high-risk sexual behaviors among community members c. Monitoring the sexual activity of adolescents d. Providing information about the hazards of multiple sexual partners

ANS: D Healthy People 2020 lists communicable disease as an area of focus, including levels of HIV, acquired immunodeficiency syndrome (AIDS), and sexually transmitted infections. One method to reduce the risk of acquiring a communicable disease is by the PHN providing information on the hazards of multiple sexual partners and street drug use. Encouraging community members to engage in healthy behaviors does not specifically address communicable diseases. Identifying behaviors and monitoring sexual activity are not appropriate focuses for programming.

2. An older person is in the last stages of dying. Which type of care would be the best for this client? a. Home health b. Assisted living c. Nursing home d. Hospice

ANS: D Hospice would be the best form of care for a person in the last stages of dying. Home health care provides individual and environmental assessments, direct skilled care and treatment, and short-term guidance for individuals in the home. Assisted living offers a wide variety of living choices, from a single shared room to a full-service, life-care community. Nursing homes provide care to meet the health care needs of those needing rehabilitation, as well as for those needing a permanent supportive residence.

7. A nurse is working in a public health center. A patient who has been newly diagnosed as HIV positive comes for counseling. By law, which of the following actions must be taken by the nurse? a. Give antiviral medications to the patient. b. Ask the person to name all his or her sexual contacts. c. Refer the patient to the social worker for case management. d. Report the HIV-infected person to the state health department.

ANS: D It is mandatory to report HIV cases by name in all states, although not all states require viral load and CD4 counts. There are not laws in place that mandate administration of medications, naming of sexual contacts, or referral to case management.

21. A nurse is planning to provide culturally effective care to Mexican migrant farmworkers. Which of the following strategies would the nurse use? a. Learn how to speak Spanish to improve communication. b. Understand that traditional beliefs and practices are more common among the well-educated population. c. Consult the female head of household about health care matters. d. Recognize that cultural backgrounds of the workers will vary depending on their place of origin.

ANS: D It would be incorrect to assume that all Mexicans have the same cultural beliefs. The nurse must remember that beliefs and practices differ between regions and localities of a country, and among individuals. Mexico is a multicultural country; therefore, the cultural backgrounds of Mexican immigrants vary depending on their place of origin.

1. A community health nurse is caring for a client with methicillin-resistant Staphylococcus aureus (MRSA). Which of the following should the nurse be aware of? a. Persons with MRSA usually have a chronic illness. b. MRSA is a hospital-acquired infection and not often seen in the community. c. Vancomycin-resistant Staphylococcus aureus (VRE) follows MRSA. d. MRSA is becoming more common in the community.

ANS: D MRSA is being seen more and more in the community with outbreaks frequently associated with school athletic programs and prison populations. MRSA is still largely a health care-associated infection, but it is becoming more common. VRE was found before MRSA. MRSA is not associated with chronic illness.

4. How have nurses historically learned to identify a possible relationship between environmental chemical exposures and their potential harm? a. Extrapolation by toxicologists b. Biomonitoring c. Completing chemistry courses d. Observing signs and symptoms in clients

ANS: D Nurses have historically made discoveries related to chemical exposure when people presented with signs and symptoms related to known chemical toxicity. Extrapolation by toxicologists and biomonitoring are modern methods. Completing a chemistry course does not show how to identify these possible relationships.

28. What characteristics should the nurse expect when working with an established education group? a. The group membership will change from week to week. b. The members all have the same interests. c. They prefer lectures rather than demonstrations. d. The group already has operating methods that have been successful.

ANS: D Nurses working with established groups should know that this type of group has membership ties and an existing structure that has proven to be successful. An established group would have a stable group membership. Learning occurs better in all groups with demonstrations instead of lectures. In an established group, they may be together because of a common interest, but may not share all the same interests.

6. A community coalition monitors the increasing obesity rate of children in their schools. Based on this data, they consider a variety of programming options which may possibly help decrease this trend. What was the purpose of conducting this surveillance? a. Protect the children from diseases that affect obese children. b. Teach parents that obesity will not be tolerated in this community. c. Educate children on surveillance techniques. d. Demonstrate that new clinical and effective protocols need to be developed.

ANS: D Ongoing surveillance in a community can lead to new clinical and effective protocols to address an issue. Ongoing surveillance makes it possible to have ongoing monitoring in place to ensure that disease and event patterns improve rather than deteriorate. This surveillance cannot protect children from diseases, teach parents that obesity will not be tolerated, or educate children on surveillance techniques.

21. Which of the following factors has the largest impact on health disparities among all populations? a. Ethnicity b. Education level c. Lifestyle choices d. Poverty

ANS: D Poverty is a strong and underlying current factor that affects all special groups. Other factors that may also influence disparities are: education, insurance status, segregation, immigration status, health behaviors and lifestyle choices, health care provider behavior, employment, and the nature and operation of the health system in the community.

18. Which setting is a forensic nurse most likely to be employed? a. Physician's office b. Urgent care center c. School-based health center d. Medical examiner's office

ANS: D Practice arenas for forensic nurses include health care facilities such as private clinics and emergency departments, criminal justice centers for victims of crime, medical examiner offices, police departments, correctional facilities, and mental health centers.

16. An advanced public health nurse (APHN) has recently been hired by a public health department. Which of the following best describes what the APHN will most likely be doing in this role? a. Diagnosing health problems b. Managing chronic disease c. Limiting disability d. Conducting community assessments

ANS: D Practicing within the role of clinician, the APHN is involved in conducting community assessments; identifying needs of populations at risk; and planning, implementing, and evaluating population-focused programs to achieve health goals. Diagnosing health problems, managing chronic disease, and limiting disability are roles of a nurse practitioner (NP).

19. Which statement is discussed in the Code of Ethics for Nurses with Interpretive Statements? a. The profession of nursing is responsible for making political statements and supporting nurse-friendly candidates for office. b. The nurse's primary focus is on acute bedside nursing, followed by community health care to promote seamless care. c. The nurse owes duty prNimaRrilyIto tGhe pBh.ysCiciaMn to strive to protect health, safety, and the rights of the patient. d. The profession of nursing is responsible for articulating nursing values, for maintaining the integrity of the profession, and for shaping social policy.

ANS: D Provision 9 of the Code of Ethics for Nurses with Interpretive Statements discusses the need for the nursing profession to address national and global health concerns as well as be involved with shaping policies through political action. The Code of Ethics for Nurses with Interpretive Statements does not address the practice setting for nursing care. According to the Code of Ethics for Nurses with Interpretive Statements, public health should achieve community health in a way that respects the rights of individuals in the community, not owing duty primarily to the physician. Public health should seek the information needed to implement effective policies and programs that protect and promote health.

14. A nurse is implementing a primary prevention strategy. Which of the following activities would the nurse most likely implement? a. Urine screening for pesticide exposure b. Tuberculosis (TB) skin testing c. Treatment of lead poisoning d. Reduction of pesticide exposure

ANS: D Reduction of pesticide exposure is an example of primary prevention. Urine screening and TB skin testing are secondary prevention, and treatment of lead poisoning is tertiary prevention.

19. A nurse responding to a disaster places the priority on which disaster response? a. Cleaning up the environment b. Handling the stress reaction of the victims c. Bringing in as many aid workers and nurses as possible d. Reestablishing sanitary barriers and focus on basic needs

ANS: D Reestablishing sanitary barriers and focusing on water, food, waste disposal, vector control, shelter, and safety are the first goals. After this has been established, the nurse can address the cleanup of the environment, the stress reactions of the victims, and the need to bring in additional workers.

10. A public health nurse (PHN) is organizing a multidisciplinary team to address the issue of water pollution in the community. Which group of professionals would be invited to address this issue? a. Physicians, water sanitation workers, and occupational therapists b. Pharmacologists, radiologists, and epidemiologists c. Nurse practitioners, pharmacologists, and environmentalists d. Geologists, meteorologists, and chemists

ANS: D Scientists who study how pollutants travel in air, water, and soil are geologists, meteorologists, and chemists. The other professionals are not experts in the area of water pollution in the community.

18. A nurse is providing seconNdaUryRpSreIvNenGtiTonBw.hCenOwMorking with the homeless. Which of the following best describes what is being implemented? a. Employer incentives b. Safe sex education c. Comprehensive case management d. Soup kitchens

ANS: D Secondary preventive services target persons on the verge of homelessness as well as those who are newly homeless. An example of secondary prevention is a soup kitchen. Employer incentives and safe sex education are primary prevention strategies; comprehensive case management is a tertiary prevention strategy.

7. A nurse surveys a program's participants after completion of a wellness program to determine if the participants have increased their involvement in daily exercise. Which of the following describes the action of the nurse? a. Assessment b. Evaluation c. Formative or process evaluation d. Summative or impact evaluation

ANS: D Summative or impact evaluation is used to assess program outcomes or as a follow-up to the results of the program activities. Formative or process evaluation is used to assess whether objectives are met or planned activities are completed. Assessment is completed to provide a clear understanding of the overall health status of a community, to identify populations at risk, and to document health needs. Evaluation is determining whether a service is needed and can be used, whether it is conducted as planned, and whether the service actually helps people in need.

6. A nurse experiencing a stress reaction while working with the survivors of a disaster is likely to experience what problem? a. Making effective decisions to assist with care coordination of the survivors b. Sharing with the supervisor of the disaster the details of the care that was provided c. Crying after returning home at the end of the day d. Refusing to follow orders

ANS: D Symptoms that may signal a need for stress management assistance include the following: being reluctant or refusing to leave the scene until the work is finished; denying needed rest and recovery time; feelings of overriding stress and fatigue; engaging in unnecessary risk-taking activities; difficulty communicating thoughts, remembering instructions, making decisions, or concentrating; engaging in unnecessary arguments; having a limited attention span; and refusing to follow orders.

23. A nurse is planning and implementing care for vulnerable populations. Which of the following would be the most appropriate action for the nurse to take? a. Setting up multiple clinics in a wide geographic area b. Advising legal consultants on a variety of issues c. Making laws to protect the homeless d. Teaching vulnerable individuals strategies to promote health

ANS: D Teaching vulnerable individuals, families, and groups strategies to prevent illness and promote health is one of the ways nurses provide care for these populations. Setting up clinics, advising legal consultants, and making laws do not address the direct care provision role of the nurse.

18. A nurse is developing and coordinating an emergency response plan for a community. With which of the following organizations would the nurse collaborate? a. American Red Cross (ARC) b. National Disaster Medical System (NDMS) c. Centers for Disease Control and Prevention (CDC) d. Emergency Management Agency (EMA)

ANS: D The EMA is responsible for developing and coordinating emergency response plans in a defined area. The ARC works with communities in the preparedness, response, and recovery phases of a disaster. The CDC studies the effect that disasters have on population health and continuously develops new prevention strategies. The NDMS provides nurses the opportunity to work on specialized disaster response teams.

5. A woman needs to take some time off from work to care for her invalid mother. Which of the following health policies allows her to take an extended leave from work to care for a family member? a. Personal Responsibility and Work Opportunity Reconciliation Act b. Women's Health Equity Act (WHEA) c. Temporary Assistance for Needy Families (TANF) d. Family and Medical Leave Act (FMLA)

ANS: D The FMLA provides job protection and continuous health benefits where applicable for eligible employees who need extended leave for their own illness or to care for a family member.

7. A homicide has occurred in a community. To whom would this information need to be reported? a. United States Public Health Service b. World Health Organization (WHO) c. Centers for Disease Control and Prevention (CDC) d. National Violent Death Reporting System (NVDRS)

ANS: D The NVDRS was initiated by the CDC to develop a national state-based surveillance system that could identify the cause and manner of death, leading to prevention efforts by pulling together data on violent deaths, unintentional firearm injury deaths, and deaths of undetermined intent. The United States Public Health Service and WHO have the goal of reducing injury, but homicides are not reported to these organizations.

13. A nurse is using the Omaha System Problem Classification Scheme. Which of the following situations is the nurse most likely experiencing? a. Measuring client outcomes b. Collaborating with other professionals c. Providing health education d. Organizing a comprehensive assessment

ANS: D The Omaha System Problem Classification Scheme is used to organize a comprehensive assessment according to four priority domains. It's simple and concrete terms are used to organize a comprehensive assessment, an important standard of nursing practice. It does not measure client outcomes, promote collaboration with other professionals, or provide health education.

12. After conducting a comprehensive client assessment, the nurse would organize information about the client's medication regimen into which domain of the Omaha System Problem Classification Scheme? a. Environmental b. Psychosocial c. Physiological d. Health-related behaviors

ANS: D The Omaha System Problem Classification Scheme uses four domains to organize client information. The Health-related behaviors domain includes information related to patterns of activity that maintain or promote wellness, promote recovery, and decrease the risk of disease. The environmental domain includes material resources and physical surroundings both inside and outside the living area. The psychosocial domain includes patterns of behavior, emotion, communication, relationships, and development. The physiological domain includes functions and processes that maintain life.

2. The ability of an agent to produce a severe pathologic reaction is known by what term? a. Antigenicity b. Invasiveness c. Toxicity d. Virulence

ANS: D The ability of an agent to produce a severe pathologic reaction is known as virulence. Antigenicity is the ability to stimulate an immunological response. Invasiveness is the ability to penetrate and spread throughout a tissue. Toxicity is the ability to produce a poisonous reaction.

14. An advanced public health nurse (ALPHAGAN) provides other nurses with information about how to improve client care. Which of the following roles is being performed? a. Researcher b. Administrator c. Educator d. Consultant

ANS: D The consultant role is being practiced when an APHN provides other nurses with information on improving client care. As a researcher, the advanced practice nurse conducts their own investigations and collaborates with doctorate-prepared nurses, answering questions related to nursing practice and primary health care. As an administrator, the advanced practice nurse may be responsible for and have direct or indirect authority and supervision over the organization's staff and client care. As an educator, the advanced practice nurse provides formal and informal teaching.

Effective Care Environment: Management of Care 17. A nurse is implementing basic program planning. Which of the following would occur during the formulating stage? a. The client and provider analyze available solutions. b. The best plan for funding is presented to administrators. c. The provider group identifies solutions. d. The client identifies problems.

ANS: D The first step in the planning process involves the identification of problems, which occurs in the formulating stage. The provider group identifies solutions during the conceptualizing phase. The available solutions are analyzed during the evaluating phase. The best plan is presented and chosen during the implementing phase.

19. A community health nurse would like to reduce the prevalence of sexually transmitted diseases (STDs) in the community. Which of the following activities would most likely be used? a. Establish immunization clinics to prevent sexual transmitted infections (STIs). b. Educate people with human immunodeficiency virus (HIV) about the mode of transmission. c. Explain to women that HIV is transmitted to women usually by intravenous drug abuse. d. Develop an STD clinic to increase community access to services.

ANS: D The goal of Healthy People 2020 is to promote responsible sexual behaviors, strengthen community capacity, and increase access to quality services to prevent STDs and their complications. Nursing activities should align with these goals through development of an STD clinic to increase community access to services. Immunization clinics will not prevent the spread of most STDs. Education without access to services will not assist with reducing the problem.

11. What is the most common reportable infectious disease in the United States? a. Gonorrhea b. Syphilis c. Herpes d. Chlamydia

ANS: D The most common reportable infectious disease in the United States is chlamydia. Gonorrhea is the second most commonly reported infectious disease. Syphilis and herpes are not among the most common reportable infectious diseases in the United States.

13. A nurse is implementing a primary prevention strategy for migrant workers. Which of the following activities would the nurse most likely implement? a. Caring for persons afflicted with pesticide exposure b. Setting up clinics for persons with HIV c. Setting up clinics for persons with HIV d. Educating about prevention of such illnesses as HIV and infectious diseases

ANS: D The primary level of prevention often includes education for the prevention of diseases, measures to reduce pesticide exposure, and immunizations. Caring for persons afflicted with pesticide exposure, setting up clinics for persons with HIV, and doing research in the field related to heat stress are examples of tertiary prevention.

7. The nurse is teaching a new diabetic client how to give himself an insulin injection. Which of the following domains would be used? a. Developmental b. Cognitive c. Affective d. Psychomotor

ANS: D The psychomotor domain includes the performance of tasks that require some degree of neuromuscular coordination and emphasizes motor skills. Developmental domain is not one of the domains of learning. The cognitive domain includes memory, recognition, understanding, reasoning, and problem solving. The affective domain is used to attempt to influence what individuals, families, communities, and populations feel, think, and value.

6. A public health nurse (PHN) assists a client in identifying the services needed the most at the least cost. Which of the following best describes the role that the nurse is using? a. Role model b. Advocate c. Counselor d. Case manager

ANS: D The public health function of a nurse who assists a client in identifying the services needed the most at the least cost is a case manager. As an advocate, the PHN collects, monitors, and analyzes data and works with the client to identify and prioritize needed services, whether the client is an individual, a family, a community, or a population. A PHN discusses with a client which services are appropriate to meet her needs. As a counselor, the nurse is a person for the client to talk with to discuss feelings and concerns. A nurse serves as a role model for the client by being a person for the client to look up to.

19. A nurse collecting morbidity data would refer to which source for applicable information? a. Vital statistics reports b. Birth records c. Death certificates d. Specialized disease registries

ANS: D There are many sources of morbidity data (rate of disease incidence), including specialized disease registries. Vital statistics reports, birth records, and death certificates are part of mortality data.

17. Which of the following groups would be most likely to receive an injection of prophylactic immune globulin for possible exposure to hepatitis A? a. Persons who have had direct contact with blood b. Those who ate at the same restaurant as the person with hepatitis A c. All healthcare workers d. All those who had household or sexual contact with persons with hepatitis A

ANS: D Those who have been in close contact with persons who develop hepatitis A should receive immune globulin. The other groups do not describe populations in direct contact with the person who has hepatitis A.

14. A nurse is caring for an individual who has been a victim of trauma. Which of the following most likely describes the client that the nurse is caring for? a. A middle-class, Caucasian female b. A middle-class, African American male c. A low-income, Caucasian male d. A low-income, African American female

ANS: D Trauma victims are overrepresented in minority, disenfranchised, and disadvantaged groups. Where inequities of resources and education exist, health disparity, violence, and other crimes rise, and accidental injuries occur more often.

21. A nurse is investigating a potential bioterrorism attack. What evidence would a nurse find if a bioterrorism attack occurred? a. Many cases of influenza b. A case of cholera in a local politician following a trip to India c. Dengue fever in a group of students who just returned from a mission trip d. An unusual illness in a population

ANS: D Unusual illness in a population should trigger an investigation that may signal a covert bioterrorism attack. Cases of influenza, cholera, and dengue fever in a group would not be considered unexpected illnesses.

9. Focus groups are used to determine the method of cleanup for contaminated groundwater in a neighborhood. Which of the following is a disadvantage of this needs assessment tool? a. Technically demanding b. Expensive c. Bias of leaders d. Time consuming

ANS: D Using focus groups to determine the method of cleanup for contaminated groundwater in a neighborhood would be very time consuming. Disadvantages of surveys are that they are expensive and technically demanding. Key informants have the disadvantage of the bias of the leaders.

5. A nurse is working with a 17-year-old pregnant cocaine addict who is homeless. Which of the following best describes this client? a. At risk b. A special population c. A Healthy People 2020 target group d. A vulnerable individual

ANS: D Vulnerable individuals often have multiple risk factors. Vulnerable populations of concern to nurses are persons who are poor or homeless, have special needs, pregnant teens, migrant workers and immigrants, individuals with mental health problems, people who abuse addictive substances, persons who have been incarcerated, people with communicable diseases and those who are at risk, and persons who are HIV positive or have hepatitis B virus or sexually transmitted infections (STIs). Risk is an epidemiological term meaning that some people have a higher probability of illness than others.


Conjuntos de estudio relacionados

Christian Values and Biblical Faith (CVBF) - Midterm

View Set

Lab: Half-Life Assignment: Reflect on the Lab

View Set

Anatomy 2: Digestive System Test

View Set

Top 50 SQL Interview Questions & Answers

View Set

DOC1 Chapter 12 Grading Systems, Marking, and Reporting

View Set